You are on page 1of 25

2023학년도 6월 고2 전국연합학력평가 정답 및 해설

[4~7] 화법 / 작문 9. [출제의도] 자료 활용의 적절성 파악하기


• 국어 영역 • [자료 1]에서 가치 소비에 대한 관심이 높아지는 현
4. [출제의도] 글쓰기 계획 파악하기
상을 확인할 수 있다. 그러나 [자료 2]를 활용하여
(가)의 1문단 ‘학생 자치활동 활성화를 위해’에서 ㄱ
소비자와 기업이 상호 의존적인 관계라고 설명할 수
을, 3문단 ‘학생회에서 회의를 개최하여’에서 ㄴ을 확
정 답 없다. 이러한 내용은 (나)와도 관련 없다.
인할 수 있다.
[오답풀이] ① [자료 1]의 설문 조사 결과를 활용하
[오답풀이] ㄷ의 학생자치실을 구축하며 발생할 수
1 ⑤ 2 ④ 3 ⑤ 4 ① 5 ③ 여 1문단에서 가치 소비를 들어본 학생과 가치 소비
있는 문제와 해결 방안, ㄹ의 학생자치실 활용과 관
6 ③ 7 ④ 8 ② 9 ④ 10 ① 에 대해 모르는 학생의 비율을 수치로 나타낼 수 있
련한 회의의 결과와 후속 회의의 주제는 (가)에서
11 ④ 12 ② 13 ② 14 ⑤ 15 ② 다.
16 ① 17 ② 18 ② 19 ④ 20 ③ 확인할 수 없다.
② [자료 2]에서 기업이 폐기물을 줄이거나 포장재를
21 ④ 22 ③ 23 ⑤ 24 ④ 25 ③ 5. [출제의도] 말하기 방식 파악하기 최소화한다는 내용을 활용하여, 가치 소비를 추구하
26 ⑤ 27 ③ 28 ⑤ 29 ⑤ 30 ① 는 움직임에 맞추어서 기업이 실천하고 있는 사례로
[B]에서는 학생 2의 발언에 대해 학생회장이 ‘구체적
31 ⑤ 32 ① 33 ④ 34 ⑤ 35 ②
으로 어떤’, ‘어떻게’라는 추가 설명을 요구하고 있다. 4문단에 보충할 수 있다.
36 ① 37 ④ 38 ① 39 ② 40 ①
[A]에서는 해당 내용을 확인할 수 없다. ③ [자료 3]에서 불매 운동이 타인의 권리를 침해할
41 ③ 42 ① 43 ② 44 ③ 45 ⑤
[오답풀이] ① [A]와 [B]에서는 모두 상대의 발언 수 있다는 내용을 활용하여 3문단에 미닝 아웃의 부
내용에 긍정적으로 반응하고 있다. 작용으로 다룰 수 있다.
② [A]와 [B]에서는 모두 상대의 발언 내용을 되묻 ⑤ [자료 2]에서 환경보호를 지향하는 제품을 구매하
해 설 지 않는다. 면 지구 환경이 좋아질 수 있으며, [자료 3]에서 미
④ [A]와 [B]에서는 모두 상대의 발언 취지를 확인 닝 아웃이 사회와 환경에 긍정적 영향을 끼칠 수 있
하지 않는다. 다고 언급하였다. 이를 활용하여 4문단에서 가치 소
[1~3] 화법 비가 바람직하게 전개되었을 때 얻을 수 있는 효과
⑤ [A]에서는 학생 1~3의 발언을 요약하여 정리하고
있지만, [B]에서는 확인할 수 없다. 를 언급할 수 있다.
1. [출제의도] 말하기 방식 파악하기
‘오늘 이곳에서 ~ 관람하면 좋겠습니다.’를 보면, 청중 6. [출제의도] 학생의 반응 분석하기 10. [출제의도] 조건에 맞게 표현하기
에게 바라는 바를 언급하며 강연을 마무리하고 있다 글의 흐름에 따라 바람직한 가치 소비가 사회에 미
㉢의 학생자치실 사용을 예약하도록 하는 방안은 학
(5문단). 치는 영향력을 표현하고 있으며, 비유법과 설의법을
생자치실 사용 대상을 확대하기로 한 것을 고려한
[오답풀이] ① 강연을 하게 된 소감을 밝히는 부분 사용하고 있다. ‘시냇물이 모여 강물이 되듯이’, ‘큰
결정임을 알 수 있다.
은 없다. 흐름’은 비유법, ‘만들 수 있지 않을까?’는 설의법이다.
[오답풀이] ① (가)에서 설문 조사를 학교 누리집과
② ‘국제식물학회’라는 단체는 언급되어 있지만, 전문 [오답풀이] ② ‘소비해야 하지 않을까?’라는 설의법을
누리소통망(SNS)에서 실시한다고 하였다(4문단).
가의 견해를 인용한 부분은 없다. 사용했으나, 비유법을 사용하지 않았으며 바람직한
② (가)에서 학생회에서 회의할 때만 사용하여 학생
③ 강연에 대해 청중이 요청하는 바는 없다. 가치 소비가 사회에 미치는 영향을 표현하지 않았다.
회실에 대해 잘 모르는 학생이 많다고 하였다(2문단).
④ 강연자가 청중에게 질문은 하지만, 청중이 강연자 ③ 바람직한 가치 소비가 사회에 영향을 미칠 수 있
④ (가)에서 공간 구성 방안에 대한 사전 회의 계획
에게 질문하지는 않는다. 음을 표현하였고, ‘물방울이 바위를 뚫듯이’라는 비유
을 찾을 수 없고, (나)의 회의 과정에서 추가로 논의
된 내용임을 알 수 있다. 법을 사용하고 있다. 그러나 설의법은 사용하지 않았다.
2. [출제의도] 강연 계획 확인하기
⑤ (가)에서 해당 사업이 학생 참여형 학생자치실 ④ ‘마음의 거울’이라는 비유법과 ‘있지 않을까?’라는
학명의 끝에는 명명자의 이름을 붙이는데, 이는 생략
구축 사업임을 확인할 수 있다(1문단). 설의법을 사용하고 있다. 그러나 개인의 소비 생활을
할 수도 있다. 하지만 학명에 명명자 이름을 표기하
성찰하자는 의미를 나타낼 뿐이며 바람직한 가치 소
는 기준은 언급되어 있지 않다(4문단). 7. [출제의도] 쓰기 맥락 이해하기 비가 사회에 미치는 영향을 표현한 것은 아니다.
[오답풀이] ① 청중에게 익숙한 소재인 무궁화를 예
기사문에서 필자의 의견이나 학생 참여가 중요하다 ⑤ 개인의 소비가 사회에 영향을 미치고 그것이 자
로 제시하고 있다(1문단).
는 것을 파악할 수 있는 내용은 확인할 수 없다. 신에게도 영향을 주게 된다는 점을 언급하여 바람직
② 식물에 학명(學名)을 붙이는 방법을 알아본다며,
[오답풀이] ① 다음 회의 날짜, 회의 주제 등의 내용 한 가치 소비의 필요성을 표현하고 있다. ‘부메랑처
강연의 목적을 밝히고 있다(1문단).
을 확인할 수 있다. 럼’이라는 비유법을 사용하였으나 설의법은 사용하지
③ 이명법의 개념과 그와 관련된 정보(속명, 종소명
② 학생자치실을 회의와 모임, 학생회 주최 행사 등 않았다.
등)를 제공하고 있다(3, 4문단).
에 활용하자는 내용을 확인할 수 있다.
⑤ 무궁화 학명의 라틴어 발음을 우리말(히비스커스
③ 접이식 가벽 설치, 이동형 수납장 배치 등의 내용
시리아커스 엘)로 표기하여 시각 자료로 제시하고 [11~15] 문법
을 확인할 수 있다.
있다(1문단).
⑤ 기사문에 인용된 학생의 발언을 통해 열의 있는
11. [출제의도] 관형사와 관형어 이해하기
3. [출제의도] 듣기 과정 점검하기 회의 분위기를 확인할 수 있다.
두 명사가 나란히 올 때 앞의 명사는 관형어로 쓰이
ⓔ에서 ‘무궁화와 히비스’의 그림을 제시하지 않은 지만, 관형사로 품사가 바뀌는 것은 아니다.
아쉬움을 느끼고 있음을 알 수 있다. 따라서 강연 자 [8~10] 작문 [오답풀이] ① 관형사는 고정된 형태로 쓰인다(3문단).
료의 준비 부족을 언급하였다고 볼 수는 있다. 하지 ② 관형사와 관형어는 모두 체언을 꾸며 준다(1문단).
만 강연 내용의 신뢰성 여부를 따지는 것은 아니므 8. [출제의도] 글쓰기 계획 파악하기 ③ 관형사 외에도 관형어로 쓰일 수 있다(2문단).
로 적절하지 않다. (나)에서 가치 소비를 접한 경험을 언급하지 않았다. ⑤ 관형사형 어미가 결합한 ‘예쁜’의 품사는 형용사
[오답풀이] ① 학명을 짓는 이유를 모르고 있다가 [오답풀이] ① ‘미닝 아웃’을 ‘미닝’과 ‘커밍 아웃’으 이다(4문단).
알게 되었으므로, 새로운 지식을 수용하며 들었다고 로 분석하여 독자가 미닝 아웃의 의미를 쉽게 받아
볼 수 있다. 들이도록 하고 있다(3문단). 12. [출제의도] 관형어 파악하기
② 동물의 속명과 종소명을 찾아본다고 하였으므로, ③ 누리소통망(SNS) 게시, 제품 구매 유도, 불매 의사 ㄴ에서 관형어는 ‘모든’과 ‘달리는’이며, 이들의 품사
강연 내용과 관련하여 더 알고 싶은 점을 떠올리며 표현 등 미닝 아웃의 여러 형태를 제시하여, 독자가 는 각각 관형사와 동사이다.
들었다고 볼 수 있다. 글을 이해하는 데 도움이 되도록 하고 있다(3문단). [오답풀이] ① ㄱ에서 관형어는 ‘새’, ‘어머니의’이며
③ 속명과 종소명에 대한 설명을 공통점과 차이점으 ④ 가치 소비의 개념, 실천 사례, 의의에 대해 다룰 이들의 품사는 각각 관형사, 명사 + 조사이다.
로 나누어 정리하였으므로, 세부 정보들의 관계를 확 것임을 제시하여 독자가 뒤에 이어질 내용을 추측하 ③ ㄷ에서 관형어는 ‘친한’이며 이것의 품사는 형용
인하며 들었다고 볼 수 있다. 도록 하고 있다(1문단). 사이다.
④ 자신이 마신 경험이 있는 차에서 히비스커스를 떠 ⑤ 소비자의 바람직한 가치 소비가 장기적으로 계속 ④ ㄹ에서 관형어는 ‘우리’, ‘가던’이며 이들의 품사는
올려 무궁화와 관련성이 있을 것이라 여기므로, 강연 되면 사회에 선한 영향력을 미칠 것이라고 그 의의 각각 대명사, 동사이다.
내용을 자기 경험과 관련지어 들었다고 볼 수 있다. 를 언급하여, 독자가 가치 소비에 지속적으로 관심을 ⑤ ㅁ에서 관형어는 ‘대부분의’, ‘여름’이며 이들의 품
갖도록 하고 있다(4문단). 사는 각각 명사+조사, 명사이다.

1
2023학년도 6월
전국연합학력평가 정답 및 해설 고2

13. [출제의도] 음운의 변동 이해하기 16. [출제의도] 표현상의 특징 이해하기 20. [출제의도] 소재의 기능 파악하기
‘뚫는’은 ‘뚫는 → [뚤는] → [뚤른]’에서 ‘ㅎ’이 탈락하 (가)는 암행어사인 화자와 백성의 질문과 대답으로 ㉡은 자연을 즐기며 거문고 곡조에 맞춰 부르는 노
고 ‘ㄴ’이 ‘ㄹ’로 교체된다. 이때 ‘ㅎ’의 탈락으로 인해 내용이 전개되고 있다. 5~8행에서 암행어사인 화자 랫소리이다. ㉢은 작가의 추억 속에 있는 ‘엿장수’가
음운은 1개가 줄어든다. 가 백성들에게 질문과 제안을 하고 있고, 9~17행은 내던 가위 소리로, 가위 소리가 들리기 시작하면 마
[오답풀이] ① ‘국밥 [국빱]’에서 ‘ㅂ’은 ‘ㅃ’으로 교체 그에 대한 백성들의 대답으로 구성되어 있다. 그에 을의 분위기가 분주하게 변화하던 모습을 확인할 수
된다. 이때 음운 개수에는 변화가 없다. 비해 (나)는 화자가 일방적으로 서술하고 있으므로 있다(3문단).
③ ‘막내[망내]’에서 ‘ㄱ’은 ‘ㅇ’으로 교체된다. 이때 문답 구조를 확인할 수 없다. [오답풀이] ① ㉠은 세금으로 인해 힘들어하는 백성
음운 개수에는 변화가 없다. [오답풀이] ② (가)에 드러나는 공간으로는 ‘당진’과 들의 울음소리로, 현재 상황에 대한 슬픔을 드러내는
④ ‘물약 → [물냑] → [물략]’에서 ‘ㄴ’이 첨가되고 이 ‘구렁’이 있는데, 화자가 지향하는 가치가 어사로서의 화자의 소리가 아니다.
‘ㄴ’이 ‘ㄹ’로 교체된다. 이때 ‘ㄴ’이 첨가되므로 음운 임무를 수행하여 백성들의 고통을 덜어 주고 싶다는 ② ㉢에서 엿장수의 불만은 드러나지 않는다.
은 1개 늘어난다. 것이라 본다면, 백성들이 현실을 도피하려는 공간인 ④ ㉠과 ㉡은 모두 화자의 과거 경험을 떠올리게 하
⑤ ‘밟힌[발핀]’에서 ‘ㅂ’과 ‘ㅎ’이 결합하여 ‘ㅍ’으로 ‘구렁’이 이 가치를 드러내는 공간이라 보기 힘들다. 는 기능을 하지 않는다.
축약된다. 이때 두 음운이 결합하여 하나의 음운이 ③ (가)의 ‘어린 자식 등에 업고 자란 자식 손에 끌 ⑤ ㉡과 ㉢이 들린 이후의 상황은 부정적이지 않다.
되었으므로 음운은 1개 줄어든다. 고’, ‘어디로서 좇아오며 어디로 가려는고’와 (나)의 그러므로 ㉡과 ㉢이 긍정적인 상황에서 부정적인 상
‘산화는 물의 픠고, 물 산의 운다’, ‘산수의 병이 황으로의 반전을 유발하는 것은 아니다.
14. [출제의도] 단어의 의미 이해하기
되고 금가의 벽이 이셔’에서 유사한 통사 구조를 반
‘굵다 ’은 ‘물체의 지름이 보통의 경우를 넘어 길
복하여 운율을 형성하는 것을 확인할 수 있다.
다.’라는 의미이고 ‘두껍다 ’은 ‘두께가 보통의 정도 [21~25] 독서(인문)
④ (가)의 ‘전년의 이천여 호 금년의 칠백 호’에서
보다 크다.’라는 의미이므로, ‘두꺼운 손가락’은 ‘굵은
구체적인 수치를 활용하여 고을의 상황 변화를 드러 <출전> 니체의 철학(재구성)
손가락’으로 쓰는 것이 적절하다.
내고 있음을 확인할 수 있다. 개관 : 이 글은 초월적 가치를 토대로 삶의 의미를
[오답풀이] ① 각 단어는 모두 2개 이상의 서로 관
⑤ (가), (나)에는 계절감을 느낄 수 있는 시어가 사 찾으려 한 전통 형이상학에 반대한 니체의 철학을
련된 의미를 가지므로 다의어이다.
용되지 않았다. 소개하고 있다. 니체는 현실적 삶 자체를 긍정하며,
② ‘열차의 기적 소리가 가늘게 들려왔다.’에서 ‘가늘
17. [출제의도] 외적 준거에 따라 작품 감상하기 각자의 삶을 주체적으로 살아갈 것을 강조하였다. 더
다’는 ‘소리의 울림이 보통에 미치지 못하고 약하다.’
높은 것으로 나아가고자 하는 욕망인 ‘힘에의 의지’
라는 의미이므로, 이 문장은 ‘가늘다 ’의 용례로 볼 9~17행에서 백성들은 자신들이 당진 출신이지만 세
와 경쟁을 통해 자신의 성장을 도모하는 ‘아곤’을 옹
수 있다. 금과 관리의 폭정으로 인해 차라리 죽는 게 편하다
호한 니체의 철학은, 현실적인 삶을 그 자체로 긍정
③ ‘그 책은 수요층이 두껍다.’에서 ‘두껍다’는 ‘층을 싶어 떠나는 것이라 말하고 있으므로 ‘고향으로 돌아
할 수 있는 철학적 토대를 마련하였다는 의의를 가
이루는 사물의 높이나 집단의 규모가 보통의 정도보 가려는 백성’이라는 표현은 적절하지 않다.
진다.
다 크다.’라는 의미이므로, 이 문장은 ‘두껍다 ’의 [오답풀이] ① 헌 누더기를 입은 백성들의 모습을
용례로 볼 수 있다. ‘차마 보지 못’하겠다고 한 것에서, 구강의 따뜻한 마 21. [출제의도] 내용 전개 방식 파악하기
④ ‘나뭇가지가 가늘다.’에서 ‘가늘다’는 ‘물체의 지름 음씨를 확인할 수 있다, 니체 철학의 핵심 개념인 ‘힘에의 의지’(3문단)와 ‘아
이 보통의 경우에 미치지 못하고 짧다.’라는 의미이 ③ 도망한 이에게 부과된 세금까지 부담해야 하는 곤’(4문단)을 설명하고 있으나, 사례를 들지는 않았
므로, 이 문장은 ‘가늘다 ’의 용례로 볼 수 있다. 백성들의 고통을 확인할 수 있다. 으므로 적절하지 않다.
④ 백성들의 문제를 해결하기 위해 ‘급히급히 고개를 [오답풀이] ① 니체는 전통 형이상학이 초월적 가치
15. [출제의도] 중세 국어의 특징 이해하기
넘어가자’는 말에서, 암행어사로의 임무에 최선을 다 를 토대로 삶의 의미를 찾고자 한 것에 반기를 든
㉡에서 현대 국어 ‘내가’는 ‘나’의 이형태인 ‘내’와 주
하려는 마음가짐을 확인할 수 있다. 철학자이다(1문단).
격조사 ‘가’가 결합된 형태이고, 중세 국어의 ‘내’는
⑤ 백성들이 고을을 떠나고 있음에도 곡식을 거두는 ② 니체 철학과 전통 형이상학의 차이점은 밝혔으나
‘나’와 주격조사 ‘ㅣ’가 결합된 형태이다. 따라서 주격
데만 힘쓰고 있다는 내용의 발언을 통해, 지방 관리 (1, 2문단), 공통점은 밝히지 않았다.
조사가 생략되었다는 말은 적절하지 않다.
들에 대한 비판적 태도를 확인할 수 있다. ③ 니체 철학의 변천 과정은 글에 드러나지 않는다.
[오답풀이] ① ‘ㆍ’가 쓰인 것으로 보아, 현대 국어에
⑤ 니체 철학은 현실적인 삶을 그 자체로 긍정할 수
쓰이지 않는 모음이 사용되고 있음을 확인할 수 있다. 18. [출제의도] 작품의 내용 이해하기
있는 철학적 토대를 마련하였다는 의의를 가진다(5
③ 현대 국어에서 ‘같은가’로 쓰인 것으로 보아, 중세 <2수>에서 귀향이 지체된 이유가 임금님의 은혜가
문단).
국어에는 이어적기가 사용되었음을 확인할 수 있다. 커서라고 말하고 있으므로, ‘미안함’은 적절하지 않다.
④ 현대 국어에서 ‘이르더니’로 쓰인 것으로 보아, 중 [오답풀이] ① ‘십 년’ 관리 생활을 하다 돌아온 ‘고 22. [출제의도] 세부 정보 파악하기
세 국어에는 두음법칙이 적용되지 않았음을 확인할 향’에서 ‘인사(人事)가’ 달라진 것을 보고, ‘세간존몰 현실적 욕구를 무가치한 것으로 보거나 삶을 개선의
수 있다. 을’ 슬퍼하는 부분에서 확인할 수 있다. 대상으로 본 것은 모두 전통 형이상학의 입장이다(2
⑤ 현대 국어에서 ‘생각지’로 쓰인 것으로 보아, 중세 ③ ‘세상의 어즈러운 긔별을 나 몰라 로라’에서 문단).
국어에는 구개음화가 일어나지 않았음을 확인할 수 확인할 수 있다. [오답풀이] ① 전통 형이상학에서는 현실 너머에 보
있다. ④ ‘산수를 희롱’하면서 ‘건듯’ 부는 ‘청풍’과 돌아오 편적 진리로 이루어진 참된 세계가 있다고 여겼다(1
는 ‘명월’을 감상하는 데서 자연과 어우러지는 모습 문단).
을 확인할 수 있고, 이는 자연에 대한 친근감을 드러 ② 전통 형이상학에서는 이성적 사유를 통해 초월적
[16~20] 문학(고전 시가 / 현대 수필)
내는 것으로 볼 수 있다. 가치를 추구하고자 하였다(1문단).
<출전> (가) 구강, 「북새곡」 / (나) 신교, 「귀산음」 / ⑤ ‘초로인생’이 ‘아이 놀고 어이 랴’에서 확인할 ④ 니체는 ‘강자를 넘어서려고 하는 의지’를 옹호하
(다) 안도현, 「엿장수 생각」 수 있다. 였다(5문단).
개관 : (가)는 구강이 암행어사의 임무를 띠고 추운 ⑤ 니체는 삶을 긍정한다는 것은 삶이 마주하는 어
19. [출제의도] 외적 준거에 따라 작품 감상하기
겨울에 북관, 즉 함경도를 지나며 경험한 일을 기록 려움을 잘 극복하고 성장하고자 하는 태도를 의미한
추억으로 남아 있던 엿장수가 사라진 것에 대한 안
한 장편 가사이다. 암행어사가 쓴 유일한 국문시가 다고 보았다(5문단).
타까움은 드러나 있지만, 그로 인해 기존 세대가 변
작품으로, 백성들의 삶의 현실, 관리들의 실정, 북관
화를 받아들이지 못한다는 내용은 지문에서 확인할 23. [출제의도] 세부 정보 추론하기
의 풍경과 그에 대한 감상이 드러나 있으며, 작품의
수 없다. 전통 형이상학에서는 절대적 가치를 삶의 궁극적인
끝부분에는 어사로서 살았던 삶에 대한 회고의 심정
[오답풀이] ① 엿장수를 기다리던 아이들의 모습을 목적으로 여기고, 개별적인 삶을 이 기준에 따라 재
이 서술되어 있다. (나)는 신교가 벼슬살이를 마치고
그 심정이 잘 드러나도록 묘사하고 있다(1문단). 단하려고 하였다. 반면 니체는 현실적인 삶 그 자체
고향으로 돌아와 고향의 산천을 바라보며 느낀 감상
② 엿장수의 가위 소리가 들리면 엿장수에게 뛰어가 가 목적이며, 가치 평가의 출발점이라고 보았다. 그
을 서술한 8수의 연시조이다. (다)는 글쓴이가 어린
던 다양한 모습을 생동감 있게 그려내고 있다(3문단). 의 시각에서 전통 형이상학은, 가치 평가의 기준이어
시절에 엿장수와 관련해서 겪었던 다양한 경험을 바
③ 고철, 함석 조각, 빈 병 등 엿으로 바꿔 먹었던 야 할 삶을, 절대적 가치를 기준으로 한 평가 대상으
탕으로, 그때를 추억하고 그리워하는 마음에서 쓴 수
물건들이 언급되고 있다(4문단). 로 만든 것으로 볼 수 있다.
필이다.
⑤ 엿장수에 대해 작가가 생각하고 있는 가치를 언
급하며, 독자들에게 질문을 던지고 있다(6문단). 24. [출제의도] 핵심 개념 비교하기

2
2023학년도 6월
전국연합학력평가 정답 및 해설 고2

니체는 ‘힘에의 의지(Ⓐ)’를 자연스러운 것으로 수용 른 아들이 있는지 묻는 행동을 통해, 교도관들이 ‘종 [오답풀이] ① 범인은 좌위 에서, 즉 7번 염색체
할 때 현재의 자신을 극복하고 새로운 가치를 창조 이쪽지’와 관련된 사실을 전달하는 것을 불편해 함을 하단부 21.11번 염색대에서 염기 서열 ‘GATA’가 각
할 수 있다고 보았다(3문단). 반면 쇼펜하우어는 ‘삶 알 수 있다. 각 여덟 번과 네 번 반복되는 상동 염색체를 가지고
에의 의지(Ⓑ)’를 다스려야 고통의 굴레에서 벗어날 [오답풀이] ① ‘종이쪽지’는 어머니가 쓴 것이 아니다. 있다.
수 있으며, 참된 행복을 위해서는 이 의지를 완전히 ② 어머니가 면회가 늦어진 것에 대해 항의하지는 ② 상동 염색체는 부계와 모계에서 각각 하나씩 물
버리는 것이 필요하다고 보았다. 않았다. 려받는다(2문단). 좌위 에서 부계와 모계에서 물려
[오답풀이] ①, ② 쇼펜하우어는 ‘삶에의 의지(Ⓑ)’가 ③ 어머니와 아들 사이에 갈등이 있다고 보기 어렵 받은 두 염색체의 STR 반복 횟수가 ‘6’으로 동일하
영원히 고통을 발생시킨다고 보았다. 안정적인 삶을 고, 교도관들이 그것을 해소하려는 것도 확인할 수 다.
유지하는 힘이나 더 행복하게 만드는 힘으로 보았다 없다. ③ ‘을’의 ‘DNA 프로필’을 만들 때 <보기>와 같이
는 것은 적절하지 않다. ④ 교도관들과 어머니의 대화 내용에서 교도관들이 20개의 좌위가 활용된다. <보기>에서 좌위 과 좌
③ ‘삶에의 의지(Ⓑ)’는 살고자 하는 맹목적인 욕망 아들과 아는 사이인지는 드러나지 않는다. 위 은 5번 염색체에서 분석이 이루어지므로, ‘을’의
이다. 따라서 쇼펜하우어가 타인과의 비교를 전제로 DNA 프로필을 만들기 위해서는 5번 염색체가 최소
한 욕망으로 보았다는 것은 적절하지 않다. 두 번은 활용된다.
[29~32] 독서(과학)
⑤ 니체는 ‘힘에의 의지(Ⓐ)’를 내면의 힘과 능력을 ④ 상동 염색체 한 쌍은 부계와 모계에서 각각 하나
더 높은 차원으로 발휘하고자 하는 의지라고 보았다 <출전> STR 분석법(재구성) 씩 물려받지만, 그 위치는 왼쪽이나 오른쪽으로 특정
(3문단). 따라서 니체가 ‘힘에의 의지’를 최소한 가짐 개관 : 이 글은 과학수사 시 범인 추정 및 피해자 확 할 수 없다. 좌위 에서 ‘AGAT’ 반복 횟수는 3과 5
으로써 고통에서 벗어날 수 있다고 보았다는 것은 인 등에 사용되는 ‘STR 분석법’에 대해 설명하고 있 이고, 좌위 에서 ‘AGAT’ 반복 횟수는 2와 7이므로,
적절하지 않다. 다. 30억 개의 염기 서열로 이루어진 DNA는 특정 모계에서 받은 염색체의 ‘AGAT’ 반복 횟수의 합은
구간에서 ‘짧은 염기 서열이 연쇄적으로 반복(STR)’ 가장 큰 값인 5와 7의 합을 넘어설 수 없다.
25. [출제의도] 구체적 사례에 적용하기
해서 나타나는 특징을 보여 주는데, 이 특징에 착안
니체의 ‘아곤’은 서로 간의 비교를 통해서 자신의 힘 32. [출제의도] 단어의 사전적 의미 파악하기
하여 동일인 여부를 판별하는 방법이 STR 분석법이
을 평가하고 더 상승시키기 위해 필요한 것이다(4문 ‘어떤 일에 대한 의견이나 느낌’은 ‘생각’에 대한 사
다. STR 분석을 위해 염색체 내 특정된 위치를 ‘좌
단). 따라서 ‘을’이 ‘갑’을 배려한 것을 ‘갑’의 능력을 전적 의미이다. ‘추정’의 사전적 의미는 ‘추측하여 판
위’라 하며, 현재 우리나라에서는 20개의 좌위를 표
확인하려는 것으로 보는 것은 적절하지 않다. 정함’이다.
준으로 하여 과학수사에 활용하고 있다.
[오답풀이] ① 자신이 뛰어넘고자 하는 ‘강자’는 자
신을 자극하고 발전시키는 선의의 파트너가 된다(4 29. [출제의도] 세부 정보 이해하기
[33~38] 독서(사회)
문단). ‘늘 1, 2위를 다투는’ ‘갑’과 ‘을’은 서로에게 STR 분석법은 30억 개의 염기 중 짧은 염기 서열이
뛰어넘고자 하는 존재이며, 그렇기에 이들은 서로에 연쇄적으로 반복해서 나타나는 특정 구간을 대상으 <출전> (가) 독점기업의 이윤 추구 과정(재구성) /
게 선의의 파트너가 된다고 볼 수 있다. 로 한다. (나) 공정거래법의 이해(재구성)
② ‘강자’들 각각의 삶이 자신의 상승을 위해 ‘힘에의 [오답풀이] ① DNA는 아데닌, 구아닌, 사이토신, 타 개관 : (가)는 경제학적 관점에서 완전경쟁시장과 독
의지’를 중심으로 경합하기도 한다(4문단). ‘갑’과 이민 네 종류의 염기로만 구성되기 때문에, 사람의 점시장의 차이점을 소개한 후, 독점시장에서 독점기
‘을’의 라이벌전이 펼쳐지는 ‘○○ 올림픽’은, ‘갑’과 DNA를 구성하는 염기 종류는 동일하다(2문단). 업이 ‘가격결정자’로서 이윤을 극대화하는 가격을 결
‘을’의 힘에의 의지가 맞서 겨루는 장으로 볼 수 있다. ② 상동 염색체는 모양과 크기가 동일한 염색체가 2 정할 수 있음을 말하고 있다. 독점기업은 이윤 극대
④ 강자들 사이에서도 힘의 차이에 따라 승패가 존 개씩 쌍으로 존재한다(2문단). 화를 위해 가격과 생산량을 조절하는데, 최적 생산량
재하며, 위계가 형성된다(4문단). 1위인 ‘을’에게 ‘금 ③ STR 분석을 위해 먼저 해야 할 것은 분석하려는 과 수요자들의 최대 지불 용의 지점을 찾아 수요자
메달’이, 2위인 ‘갑’에게 ‘은메달’이 주어진 것은 이러 염색체 내의 위치를 특정하는 것이다(4문단). 가 최대로 지불할 수 있는 금액을 최종 시장가격으
한 힘의 차이에 따른 위계를 반영한 것으로 볼 수 ④ 좌위는 모두 염기 서열의 STR이 나타나는 구간 로 결정한다. 또한 독점은 시장가격의 상승을 유발하
있다. 으로 지정되어 있다(4문단). 여 수요자에게 부정적 영향을 끼치고, 시장의 비효율
⑤ 강자들 사이에서도 위계가 존재하지만, 좀 더 나 성을 유발한다. (나)는 ‘공정거래법’이라고도 불리는
은 사람이 되고자 노력하였음을 서로 인정하므로, 강 30. [출제의도] 세부 정보 추론하기
‘독점규제 및 공정거래에 관한 법률’과 관련된 내용
자와 상대적 약자 간의 힘의 위계는 상호 존중의 형 짧은 염기 서열이 연쇄적으로 반복해서 나타나는 특
을 소개하고 있다. 공정거래법은 사업자의 시장 지배
태로 드러난다(4문단). ‘갑’과 ‘을’이 서로의 노력을 징은 DNA의 특정 구간에서만 나타난다(3문단). 따
적 지위 남용과 사업자들의 부당한 공동행위 등 불
인정하고 ‘서로에게 박수를 보낸’ 모습은 강자인 ‘을’ 라서 이러한 특징이 나타나지 않는 구간에는 STR
공정한 경쟁 제한 행위를 규제하고 있다. 공정거래법
과 상대적 약자인 ‘갑’ 간에 상호 존중의 형태로 힘 분석법을 사용할 수 없다.
을 위반한 경우, 공정거래위원회는 해당 사업체에 대
의 위계가 드러난 것으로 볼 수 있다. [오답풀이] ② 부계와 모계에서 물려받은 상동 염색
한 시정 조치와 과징금 부과를 통해 자유로운 경쟁을
체 한 쌍은 모양과 크기가 동일하다(2문단). 따라서
촉진하고 경제의 균형 있는 발전을 도모하고 있다.
상동 염색체의 동일한 위치에서는 부계와 모계에서
[26~28] 문학(현대 소설) 받은 염색체의 염색대 번호가 동일하다. 33. [출제의도] 세부 정보 파악하기
③ 동일인에서 채취한 경우, 혈흔이나 모발 등 샘플 (가)는 독점기업의 이윤 추구 방법과 관련된 독점기
<출전> 한승원, 「어머니」
이 다르다 하더라도 DNA는 동일하다. 따라서 동일 업의 가격과 생산량 결정 과정을 설명하고 있고, (나)
개관 : 이 작품은 일제 강점기와 해방 직후를 배경으
인에서 채취한 서로 다른 샘플에서는 좌위가 같으면 는 공정거래법의 내용을 설명하면서 공정한 거래를
로 그 시대를 살아가는 어머니의 한과 자식에 대한
염기 서열의 반복 횟수도 동일하게 나타난다. 저해하는 행위를 유형별로 나누어 설명하고 있다.
깊은 사랑을 다루고 있다. 어머니는 감옥살이를 하는
④ ‘4q31.3’인 좌위에서는 짧은 염기 서열 ‘CTTT’가 [오답풀이] ① (가)에서 시장구조를 바라보는 다양
자식을 면회하기 위해 필요한 여비를 마련하려고 애
반복되고 있다(4문단). 따라서 STR 분석법의 대상이 한 관점과 (나)에서 공정거래법에 대한 상반된 관점
쓴다. 그 과정에서 과거부터 지금까지의 일을 회상하
되는 특정 구간에 네 종류의 염기가 모두 반복되는 은 확인할 수 없다.
며 아들을 만나러 가는 심정을 애절하게 그리고 있다.
것은 아니다. ② (가)에서 독점의 이익 추구 과정은 나타나 있으
26. [출제의도] 서술상의 특징 파악하기 ⑤ 우리나라를 비롯한 여러 나라에서 20개의 좌위를 나, 독점이 필요한 이유는 드러나지 않는다. (나)에
이야기 외부의 서술자가 ‘어머니’에 초점을 두고 사 표준으로 하여 과학수사에 동일하게 활용하고 있다 서 부당한 독점 행위를 해결하기 위한 사례도 확인
건과 인물에 대한 생각을 서술하여 ‘어머니’의 내면 (5문단). 따라서 20개의 좌위를 사용하는 국가 간에 할 수 없다.
을 잘 드러내고 있다. 는 분석 결과를 공유할 수 있다. ③ (가)에서 균등한 소득 분배를 위한 경제학적 대
책은 확인할 수 없다. (나)에서 제시된 공정거래법은
27. [출제의도] 외적 준거에 따라 감상하기 31. [출제의도] 구체적 사례에 적용하기
경쟁을 제한하는 행위를 규제하는 대책이다.
어머니가 막내아들을 면회하기 위해 필요한 여비를 큰 20개의 좌위에 대한 두 샘플의 결과가 동일하게 나
⑤ (나)에서 독점 행위를 규제하는 제도의 문제점은
아들들이 마련해주지 않아서 원망을 한 것이지, 경제 올 때, 두 샘플이 동일인의 것일 확률이 100%에 가
확인할 수 없다.
적 어려움이 자식들 때문이라고 탓하는 것은 아니다. 깝다(5문단). <보기>와 ⑤번 선지를 비교해 보면
좌위 의 결괏값은 일치하지만, 좌위 의 결괏값은 34. [출제의도] 세부 정보 파악하기
28. [출제의도] 세부 내용 이해하기
서로 다르다. 따라서 모든 좌위의 결괏값이 일치하는 ㉠ 완전경쟁시장은 많은 수의 수요자와 공급자가 존
어머니를 앞에 두고 교도관끼리 말을 속닥거리고, 다
것은 아니므로, ‘을’을 범인으로 확정할 수는 없다. 재하므로 개별 공급자와 수요자가 가격에 영향을 미

3
2023학년도 6월
전국연합학력평가 정답 및 해설 고2

치기 어렵다. 반면에 ㉡ 독점시장은 한 기업이 독점 되었다. ③ 계도가 부인에게 자신의 외모가 소 학사와 닮은
적으로 재화를 공급하므로 시장지배력을 가진다(1, 2 ④ 지문 ⓓ의 ‘불리다’는 ‘무엇이라고 가리켜 말해지 것이 우연이라고 말한 것은 부인이 자신의 할머니임
문단). 거나 이름이 붙여지다.’의 의미로 사용되었으나, 선지 을 몰랐기 때문으로 볼 수 있다.
[오답풀이] ① ㉠ 완전경쟁시장에서 기업은 가격수용 의 ‘불리다’는 ‘이름이나 명단이 소리 내어 읽히며 대 ④ 천도화가 피었다가 사흘 후에 지는 것은 집에 경
자로서 시장에서 결정된 가격을 그대로 받아들인다 상이 확인되다.’라는 뜻으로 사용되었다. 사스러운 일이 생길 것임을 예고하는 것으로 계도가
(2문단). ⑤ 지문 ⓔ의 ‘이루어지다’는 ‘어떤 대상에 의하여 일 부인 집에 들른 날에 핀 천도화가 사흘 후에 진 것
② ㉡에서 독점기업이 생산량을 늘려나가면 한계수 정한 상태나 결과가 생기거나 만들어지다.’의 의미로 은 계도가 소 학사의 아들, 즉 부인의 손자임을 암시
입은 가격보다 낮아진다(3문단). 사용되었으나, 선지의 ‘이루어지다’는 ‘몇 가지 부분 한 것으로 볼 수 있다.
③ 시장의 유일한 공급자인 독점기업이 생산량을 줄 이나 요소가 모여 일정한 성질이나 모양을 가진 존 ⑤ 서준은 소 학사를 공격하고 그의 가족이 헤어지
이면 시장가격이 상승하고, 반대의 경우 시장가격이 재가 되다.’라는 뜻으로 사용되었다. 게 만든 적대자이므로, 소 학사의 아들인 계도가 서
하락한다(3문단). 준에게 양육된 것은 서사적 긴장감을 유발한다고 볼
④ ㉠은 진입장벽이 존재하지 않아 누구나 들어와 수 있다.
[39~42] 문학(고전 소설)
경쟁할 수 있는 시장구조이다(1문단).
<출전> 작자 미상, 「소학사전」
35. [출제의도] 구체적 사례에 적용하기 [43~45] 문학(현대 시)
개관 : 이 작품은 소 학사가 해적의 공격으로 가족과
독점기업 ‘갑’이 생산량을 Q1에서 Q2로 늘리면서 제
헤어졌다가 다시 만나는 과정을 보여 주고 있다. 적 <출전> (가) 정끝별, 「현 위의 인생」 / (나) 나희덕,
품의 가격을 P2에서 P1으로 낮춰 공급하면, 해당 독
대자에 의해 길러진 주인공의 아들이 적대자를 징계 「뿌리로부터」
점기업이 독점을 통해 얻고 있던 이윤은 사라진다.
하고 가족 상봉을 성사시키는 과정을 다양한 암시 개관 : (가)는 인생을 ‘현’에 빗대어 세상의 고단함
[오답풀이] ① 독점기업 ‘갑’의 최적 생산량 수준은
모티프의 활용이나 조력자의 등장과 같은 흥미 요소 속에서도 마음을 다스리며 아름다운 소리를 내기 위
한계수입곡선과 한계비용곡선이 교차하는 Q1 지점
를 풍부하게 활용하여 전개하고 있다. 해 노력하는 화자의 모습이 드러나 있는 작품이다.
이다.
(나)는 존재의 근원인 뿌리로부터 벗어나 새로운 길
③ 독점기업 ‘갑’의 이윤 추구 과정에서 한계수입이 39. [출제의도] 세부 내용 이해하기
을 찾아 나서는 화자의 모습이 드러나는 작품이다.
한계비용보다 높으면 생산량을 증가시켜야 한다. 거문고에 새겨진 글자를 확인하자고 제안한 사람은
화자는 안정된 뿌리로부터 벗어나 불확실하고 위험
④ 독점기업 ‘갑’이 제품을 판매할 때 얻게 되는 단 계도가 아니라 부인이다.
하지만 새로운 길을 찾으면서 한 걸음 더 성장하기
위당 이윤은 제품의 가격으로 설정한 P2에서 한계비 [오답풀이] ① 부인의 시아버지, 즉 소 승상의 아버
위해 노력하는 모습을 보인다.
용인 P1만큼을 뺀 값이 된다. 지가 신선에게서 천도화 나무를 얻어 뒤뜰에 심었다.
⑤ P1은 최적 생산량 Q1에 대응하는 가격이지만, 이 ③ 부인은 비자의 말을 들은 후에 외당에 나가 문틈 43. [출제의도] 표현상의 특징 파악하기
윤이 극대화되는 지점은 아니다. 독점기업은 최적 생 으로 공자(계도)의 상을 살펴보았다. (가)에는 ‘소리’, (나)에는 ‘뿌리’라는 시어를 반복하
산량을 Q1으로 결정한 후, 가격결정자로서 수요자들 ④ 부인은 둘째 아들이 큰아들을 찾아 나간 지 여러 여 각 시의 주제를 강조하고 있다.
의 최대 지불 용의 수준(수요곡선)을 고려하여 해당 해가 지났지만, 역시 소식이 끊어져 두 아들이 돌아 [오답풀이] ① (가), (나) 모두 공간의 이동은 나타
제품의 가격을 P2로 결정한다. 오기를 기다렸다. 나지 않는다.
⑤ 계도는 부인 집 후원에 피었던 천도화가 사흘 후에 ③ (나)는 명사로 시를 마무리하고 있지만, (가)는
36. [출제의도] 세부 정보 추론하기
지는 것을 직접 보고 월봉산 노인의 말을 떠올렸다. 동사로 시를 마무리하고 있다.
독점기업은 이윤 극대화를 위해 제품의 시장가격을
④ (가)는 청유형 종결 어미를 활용하고 있지만,
한계비용보다 높게 설정하는데, 시장가격의 상승으로 40. [출제의도] 인물의 심리 파악하기
(나)는 활용하고 있지 않다.
한계비용보다 지불 용의가 낮은 수요자들은 소비를 ㉠에서 부인은 남녀가 유별하다는 유교 규범에도 불
⑤ (가), (나) 모두 색채어가 사용되지 않았다.
포기하게 된다. 따라서 상호 이득이 될 수 있었던 거 구하고 계도를 만나보고자 한다.
래의 기회는 줄게 되고, 시장의 비효율성을 유발할 [오답풀이] ② ㉡에서 부인은 계도가 아들과 닮았기 44. [출제의도] 작품의 의미 추론하기
수 있다. 따라서 공정거래법에서는 독점으로 인한 경 에 만나기를 청했다는 자신의 입장을 설명하고 있다. 화자는 자신이 ‘비명을 노래’하면 청자에게 ‘속삭임’
제적 손실을 해소하기 위해 시장의 진입 제한을 막 ③ ㉢에서 계도는 천도화가 핀 것을 신기해 하며 그 으로 받아 달라고 말할 뿐, 자신을 반성하는 부분은
고, 기업 간 공정한 경쟁을 촉진한다. 것이 사흘 후에 질 것인지 확인하고자 한다. 확인할 수 없다.

37. [출제의도] 구체적 사례에 적용하기 ④ ㉣에서 부인은 계도가 가진 거문고가 아들의 것
45. [출제의도] 외적 준거에 따라 감상하기
임을 알아보고 아들을 그리워하며 한탄하고 있다.
공정거래법에서는 부당한 공동행위를 위한 사업자 화자는 뿌리로부터 멀어지면서 한 단계 성장하는 것
⑤ ㉤에서 계도는 거문고가 자기 가문의 기물임을
간의 합의만 있으면, 비록 그것이 실행되지 않더라도 이지, 자신이 ‘뿌리로부터 온 존재’임을 인정하면서
한 번 더 말함으로써 부인의 오해를 풀고자 한다.
부당한 공동행위가 성립한 것으로 본다(3문단). 한 단계 성장하는 것은 아니다.
[오답풀이] ① A사는 자신의 시장 지배적 지위를 남 41. [출제의도] 발화 의도 이해하기
용하여 경쟁사업자인 B사의 사업 활동을 부당하게 [B]에서는 거문고의 내력을 밝혀 말함으로써 그것이
배제하고 있다. 자기 가문의 기물이라는 계도의 주장을 반박하고 있
② B사와 거래하지 않는 조건으로 A사가 국내 PC 지만, [A]에서는 사물의 내력을 제시하고 있지 않다.
제조업체들과 합의한 것은 배타조건부 거래에 해당 [오답풀이] ① [A]와 [B]에서는 모두 인물의 심정
한다. 을 직접 드러내지만 상대의 행동을 유도하고 있지는
③ 공정거래법에서는 명시적인 형태와 묵시적인 형 않다.
태 모두 경쟁 제한 행위가 될 수 있다고 본다. ② [A]와 [B]에서는 모두 과거에 있었던 일을 제시
⑤ 공정거래법에서는 독과점 자체를 금지하지는 않 하여 상대에게 자신의 입장을 설명하고 있다.
으나, 이로 인해 일정한 폐해가 초래되는 경우 이를 ④ [A]에서는 자신에 대한 정보를 제공하지만 상대
규제하는 폐해규제주의를 취하고 있다. 의 협조를 요청하고 있지는 않으며, [B]에서는 자신
에 대한 정보를 제공하고 있지 않다.
38. [출제의도] 단어의 문맥적 의미 파악하기
⑤ [A]에서는 상대의 특정한 행동을 언급하지만 상
지문 ⓐ와 선지 ①의 ‘취하다’는 모두 ‘어떤 일에 대
대의 입장을 이해하고 있지는 않으며, [B]에서는 상
한 방책으로 어떤 행동을 하거나 일정한 태도를 가
대의 행동을 언급하고 있지 않다.
지다.’라는 뜻으로 사용되었다.
[오답풀이] ② 지문 ⓑ의 ‘나누다’는 ‘여러 가지가 섞 42. [출제의도] 외적 준거에 따라 감상하기
인 것을 구분하여 분류하다.’의 의미로 사용되었으나, 승상이 부인의 꿈에 나타나 손자가 올 것이라 말하
선지의 ‘나누다’는 ‘즐거움이나 고통, 고생 따위를 함 는 것은 부인이 손자인 계도를 만나게 됨을 예언한
께하다.’라는 뜻으로 사용되었다. 것이라고 볼 수 있다.
③ 지문 ⓒ의 ‘말하다’는 ‘어떤 사정이나 사실, 현상 [오답풀이] ② 부인이 아들의 소식을 여러 해 동안
따위를 나타내 보이다.’의 의미로 사용되었으나, 선 알지 못한 것은 가족이 헤어진 상황으로 볼 수 있다.
지의 ‘말하다’는 ‘평하거나 논하다.’라는 뜻으로 사용

4
2023학년도 6월
전국연합학력평가 정답 및 해설 고2

대하여 대칭이동한 그래프를 나타낸 함수는 (ⅱ)  ≥  일 때


• 수학 영역 •   
  이다. 따라서         

 
 
 
 
  


 ×  ×    ×  
×  ×
 
9. [출제의도] 삼각함수 사이의 관계를 이용하여 함숫값
계산하기   × 

정 답 
  

 
 

sin    cos    이고 sin     이므로 에서  
×  ×    이고
 
1 ③ 2 ② 3 ① 4 ④ 5 ④      
cos     sin         이다.  
6 ③ 7 ② 8 ⑤ 9 ④ 10 ⑤     ×     이므로 이를 만족시키는 자연수
11 ① 12 ① 13 ① 14 ③ 15 ⑤     은 존재하지 않는다.
      이므로 cos     이다.
16 ⑤ 17 ③ 18 ④ 19 ① 20 ②   따라서 (ⅰ), (ⅱ)에 의해    ,    이므로
21 ⑤ 22  23  24  25        이다.

26  27  28  29  30  sin    
따라서 tan         15. [출제의도] 삼각함수의 그래프를 이용하여 문제
cos    
 해결하기

해 설   cos      이므로 점 A 의 좌표는
10. [출제의도] 삼각함수의 그래프 이해하기   
   이다.    ≤  ≤   에서 직선    와
함수    sin    의 주기가  이고    이므로   
1. [출제의도] 지수 계산하기   
    에서    함수    cos     의 그래프가 만나는 두 점의
         
              
  
함수    sin    의 그래프가 점    를 지나  좌표는 방정식  cos       의 실근과 같다.
2. [출제의도] 로그 계산하기   
log       
 log    log     므로    sin    에서      ⋯ ①  cos       , cos      에서
log       
     또는   이므로 점 와 C 의 좌표는
 B
함수    sin    의 그래프가 점    을
3. [출제의도] 부채꼴의 넓이 계산하기 
부채꼴의 넓이를  , 중심각의 크기를  , 반지름의 
지나므로    sin    에서       ⋯ ②
 


각각     ,    이다.
   
  삼각형 ABC 는 정삼각형이므로
길이를  라 하면     이므로
 ①과 ②를 연립하면    ,    이 되어   

AC × sin    ×    
      
   ×  ×       ×  ×    ×  ×    이다.
   

따라서    
4. [출제의도] 삼각함수가 포함된 방정식 이해하기 11. [출제의도] 사인법칙 이해하기 

 삼각형의 세 각  ,  ,  에 대응하는 선분의 길이 16. [출제의도] 로그함수의 그래프를 이용하여


방정식  sin      에서 sin    이므로
 를 각각  ,  ,  라 하자.        이므로 명제 추론하기
  sin    sin     sin  이다.
      일 때, 방정식  sin      의 해는 ㄱ.    일 때, P    , Q   이므로
    
사인법칙에 의해        
 sin  sin  sin     ×   ×    (참)
   이다. 
 이므로 ㄴ.    일 때, P    , Q   이므로
sin   sin   sin    
5. [출제의도] 상용로그의 성질 이해하기 
 sin   sin   sin     ×   ×   

log   log  ×    log    

이고 상용로그표에서 log    이므로
log    이다.





    일 때, P     , Q     이므로
    

수 ⋯   

   

    ×    ×   


 

⋮ ⋮ ⋮ ⋮ ⋮ 따라서    ×    (참)


12. [출제의도] 지수함수의 그래프 이해하기
 ⋯    ㄷ.  이 아닌 모든 실수  에 대하여
함수       의 역함수의 그래프가
 ⋯    P     , Q     이다.    일 때    이므로
점        를 지나므로       이다.
 
 ⋯              에서    이다.    ×  ×        
 
13. [출제의도] 지수부등식 이해하기 
          

6. [출제의도] 코사인법칙 이해하기 
(ⅰ)    ≥  이고  
 ≥  일 때
       
코사인법칙에 의해  ≥  에서  ≥  이고   ≥  에서  ≤ 

따라서       
  
           
 

BC  
AB  
AC   × 
AB × 
AC × cos  이다. 이를 만족시키는 정수 는 존재하지 않는다.
 
 (ⅱ)    ≤  이고     ≤  일 때 이므로,  의 값이 증가하면  의 값도 증가한
      ×  ×  ×      
  ≤  에서  ≤  이고   ≤  에서  ≥  다. (참)

이므로 BC  이다. 이므로   ≤  ≤  이다.
17. [출제의도] 삼각함수의 정의를 이용하여 식의 값
따라서 (ⅰ), (ⅱ)에 의해 정수  의 개수는  이다.
7. [출제의도] 지수함수의 그래프 이해하기 구하는 문제 해결하기
함수        의 그래프의 점근선이 직선 14. [출제의도] 거듭제곱근의 성질 이해하기
OP 
점 P    (  )에 대하여     이므로
   이므로    이다. 이 함수의 그래프가     

  
 
 
 
 

점    를 지나므로       에서    이다.  
× ×  ×    ×  에서 sin    , cos    이다.
  
   
  
따라서       
과  이 자연수가 되어야 하므로  은  의  
  cos   sin          에서
8. [출제의도] 로그함수의 그래프 이해하기      

배수이다.
함수   log     의 그래프를  축의 방향으로         ,    
(ⅰ)    일 때
 
 만큼 평행이동한 그래프를 나타낸 함수는
 

 
   이므로    이고 점 P 는 P    이다.  
 

  ×    이므로

   
  log     이고 


× 

×    ×  에서  
  
   

함수   log     의 그래프를 직선    에    
   이다. 따라서  OP    
  

5
2023학년도 6월
전국연합학력평가 정답 및 해설 고2

   
18. [출제의도] 지수함수가 포함된 방정식 문제 해결하기 
(ⅳ)    일 때,           을 만족    일 때,   ≤  ≤   에서   이다.
  
두 직선    ,    와 곡선      이 만나는 시키는 자연수  은  ,  이다.  
   일 때,   ≤  ≤   에서
점을 각각 구하면 ①    ,    일 때,  ,  는 자연수이므로  
    에서   log    이고      에서 
   
          을 만족시키는     이다.
  log   이므로 점 D , F 의 좌표는 각각  
순서쌍   는    이다.  
log       , log     이다. ②    ,    일 때,  ,  는 자연수이므로    일 때,   ≤  ≤   에서   이다.
 
두 곡선     과      은  축에 대하여 
 따라서  ≤  ≤  에서  이 무리수인  은  ,
대칭이므로 점 E , G 의 좌표는 각각           을 만족시키는
 이다. 함수  의 주기는  이므로 자연수 
  log     ,  log    이다. 순서쌍   는    이다.

에 대하여
삼각형 CFG 의 넓이는 
   
(ⅴ)  ≥  일 때,          에서  ≤  ≤  ⋯ ①
   


log     log  ×      log  


 이 자연수이므로  ≥  이고  
    ≤  ≤     ,
이고 사각형 ABED 의 넓이는  
 ≥    이다.
           


  log    log       ×      ≤≤  ⋯ ②
그러므로           을 만족시키는  
   log     자연수  는 존재하지 않는다. ①, ②에서  의 최댓값은 서로 같다.
이다.      이고 삼각형 CFG 의 넓이와 사각형 따라서 (ⅰ)~(ⅴ)에서    의 최댓값은 따라서     이다.
ABED 의 넓이가 같으므로      이다.    ,    이므로
  log      log      [다른 풀이]  이하의 자연수  에 대하여  가 무리수가
log      log    되도록 하는 모든  의 값의 합은
log    에서    ,         
               이다.
 
log     이다. 따라서     
 log     ( ,  은 서로소인 자연수),    에 22. [출제의도] 지수 계산하기

 
19. [출제의도] 삼각함수를 활용하여 넓이 추론하기  
서    ,    ( 는  이상의 자연수)라 하자.   
 ×     ×     ×    ×   

∠CAB   이므로 ∠COB   이다. 삼각형 POB    이므로


 23. [출제의도] 로그함수가 포함된 방정식의 해 계산하기
가 이등변삼각형이고 ∠OQB   이므로          ,    ,  ,  이다.
 진수 조건에 의해     이고
(ⅰ)    일 때,    이고    ,  ,  ,  이므로
점 Q 는 선분 PB 의 중점이고 ∠POQ   이다. 

선분 PO 와 선분 QD 가 평행하므로
       ≤     
(ⅱ)    일 때,    ,  이고    ,  이므로 


log       에서     
     이므로
삼각형 POB 와 삼각형 QDB 는 닮음이다.    이다.
        ≤     
따라서 QD  이고 ∠QDB   이므로
(ⅲ)    일 때,    ,  이고    이므로 24. [출제의도] 삼각함수의 그래프의 주기 이해하기

   ×  ×  × sin     
       ≤       
 
 함수   cos   의 주기는   

(ⅳ)  ≥  일 때,    인 자연수  가 존재  
이다. CQ 
 CO 
QO 이므로 

 하지 않으므로 (가)와 (나)를 만족시키는 자연수  와
   × 
PQ × 
CQ sin  ×    cos    는 존재하지 않는다.
  
함수   tan  의 주기는   
   
이다.    ,   ,   cos  이므로 따라서 (ⅰ)~(ⅳ)에서    의 최댓값은  이다. 


   





 ×   ×     ×  × 

  

 21. [출제의도] 삼각함수의 그래프를 활용하여 최댓값
추론하기
두 함수의 주기가 같으므로   


따라서   
20. [출제의도] 로그의 성질을 이용하여 최댓값을 구하는   
문제 해결하기   
  

 
  sin  
  25. [출제의도] 삼각함수의 그래프 이해하기

  
조건 (나)에서 log     ( 과  은 서로소인 자
 

 


O


함수   cos    의 그래프가 점    
             
 
 
                    
             를 지나므로

연수)라 하면    이다.  


  


    cos        ×  cos   
  함수    의 주기는  이다.  
log    에서     이고, 조건 (가)에서
   이다. 따라서       이므로    이다.
    일 때,  ≤  ≤   에서   이다.
  
          이다. 26. [출제의도] 지수법칙을 이용하여 문제 해결하기
  
(ⅰ)    일 때,          을 만족    일 때,  ≤  ≤   에서   이다.       라 하면
  
시키는 자연수  은 존재하지 않는다.
       ≥   ×     이므로  ≥  이다.
    일 때,   ≤  ≤   에서   이다.
   

(ⅱ)    일 때,          을 만족       에서       이고  ≥  이므로
  
시키는 자연수  은  이고,  ,  는 자연수이므로    일 때,   ≤  ≤   에서   이다.    이다.
   

                
          을 만족시키는 순서쌍    일 때,   ≤  ≤  에서   이다.
 
    
  는    ,    ,    ,    이다.
        
    일 때,   ≤  ≤   에서
(ⅲ)    일 때,           을 만족  
27. [출제의도] 로그의 정의를 이용하여 집합 문제
시키는 자연수  은  ,  이다.   
    이다. 해결하기
①    ,    일 때,  ,  는 자연수이므로  
   집합  의 원소는 모두 자연수이므로  ,  ,  는

    일 때,   ≤  ≤   에서   이다.
         을 만족시키는
    모두  ( 는 자연수)의 꼴이다.
순서쌍   는    ,    이다.         이므로     이고
   일 때,   ≤  ≤   에서   이다.
②    ,    일 때,  ,  는 자연수이므로          ,       log   이다.
    집합  의 모든 원소의 합이  이므로
    일 때,   ≤  ≤   에서   이다.
          을 만족시키는   
log     이고      이다.
순서쌍   는    ,    이다.

6
2023학년도 6월
전국연합학력평가 정답 및 해설 고2

따라서 집합  의 모든 원소의 합은  
DF 
DF 함수    의 그래프와 직선    의 교점의
삼각형 CDF 에서      ⋯ ②
       이다. sin   개수가  이므로   이다.
sin 

28. [출제의도] 삼각함수가 포함된 방정식 문제 해결하기   
이 성립한다.
자연수  에 대하여  ≤  ≤  일 때,  에 대한   DF
  이고   
   이므로 DE
 
   

방정식 sin      의 실근의 합은

 

①에서      sin      cos 
      

         에서    가 되어 조건을 만족시키지


함수   sin  의 그래프가 직선    과 
 ②에서      sin  이므로 않는다.

만나는 점의  좌표의 합과 같다. 함수   sin  의 (ⅱ)    일 때
두 식의 양변을 제곱하여 연립하면
주기는  이다. 함수    은  의 값이 증가할 때  의 값도 증가
        sin    cos   
(ⅰ)    일 때, 함수   sin  의 그래프가 한다. 따라서 함수  는

   
직선    과 만나는 점의  좌표는  ,  이므로         또는  ≥    일 때  의 값이
 
따라서    ,    이므로      이다. 증가하므로  의 값도 증가하고,
  이다.
[다른 풀이]  ≤    또는  ≤      일 때  의 값이
  감소하므로  의 값도 감소한다.
∠AED   라 두면 ∠AFD     이고
∠BED     , ∠CFD   이다. 따라서 함수    의 그래프는 다음과 같다.
삼각형 BDE 와 삼각형 DCF 의 외접원의 반지름의
길이를 각각  ,  라 하면 사인법칙에 의해
O       
  
BD 
BD
  삼각형 BDE 에서     
sin    sin 
  sin  ⋯ ①
BD   


CD
(ⅱ)    일 때, 함수   sin  의 그래프가 삼각형 DCF 에서   
sin 

직선     과 만나는 점의  좌표를  ,  ,  ,  
CD  sin  ⋯ ②
 

   
라 하면    ,    에서  이다. ①, ②에 의해 
BD 
CD       이다.    
   
 
AB 
AC  , ∠BAC   에서  BC 
 이므로 함수    의 그래프와 직선    의 교점의 개
 
수가  이므로   이다.
  
CD  이다.
   
 

DF  라 두면 
 
   
  CF    가 되어
       
O 삼각형 DCF 에서 코사인법칙에 의해
 


 


   
          ×  cos 
  

 
(ⅲ)    일 때, 함수   sin  의 그래프가 
 따라서    ,    이므로      이다.
직선    과 만나는 점의  좌표를 ′ , ′ , ′ ,  ′

30. [출제의도] 지수함수의 그래프 추론하기      
′  ′  ′   ′ 
이라 하면    ,    에서          
    
    에서    이다.
 
 이다. 

   ≥  함수    의 그래프와 직선    의 서로 다른


 이며          일 때   이고, 세 교점의  좌표를  ,  ,  라 두면(    )
  ≤    또는  ≥    일 때 ≥  이다.   이므로 다음과 같은 세 가지 경우로 나눌
 따라서  는 수 있다.

               

①    일 때,    이므로
O ′  ′ ′  ′                  

 
    ≤    또는  ≥   
  


        ≤    따라서    이므로      이다.
②    일 때,       이다.
(ⅳ)  ≥  일 때, (ⅱ), (ⅲ)과 같은 방법으로  이            
③    일 때,    이므로



짝수일 때   ,  이  이 아닌 홀수일 때       ≤                  
  이다. 따라서        
                   ≥   
따라서     이므로      이다.
이다.
(ⅰ)      일 때 (ⅰ), (ⅱ)에 의해    이므로     이고
29. [출제의도] 사인법칙을 이용하여 선분의 길이 구하는
①, ②, ③에서 모든  의 값의 합은       
문제 해결하기 함수    은  의 값이 증가할 때  의 값은 감소
이다.
  한다. 따라서 함수  는
DF  , ∠CDF   라 하면 ∠BDE     이고
        또는  ≥    일 때  의 값이
    
AE DE 이므로 BE   AE   DE 이다. 증가하므로  의 값은 감소하고,
같은 방법으로  ≤    또는  ≤      일 때  의 값이

AF 
DF 이므로 
CF   
AF   
DF 이다. 감소하므로  의 값은 증가한다.
삼각형 BDE 와 삼각형 DCF 의 외접원의 반지름의 따라서 함수    의 그래프는 다음과 같다.
길이를 각각  ,  라 하면 사인법칙에 의해   
 
DE 
DE
삼각형 BDE 에서      ⋯ ①

sin   
  
sin 
     

7
2023학년도 6월
전국연합학력평가 정답 및 해설 고2

M: While taking a walk along the beach, I was so W: We have several kinds. Take a look at this display.
• 영어 영역 • inspired by the sound of the waves. M: All right, thank you. [Pause] Oh, how much are
W: That explains why the melody sounds very these? These would be perfect for my children.
refreshing. W: They’re 50 dollars each. They’re anti­fog, which
M: I thought of you as soon as I wrote it. I think it guarantees clear vision for fast swimming.
정 답
will go very well with your voice. M: I see. They’re a little bit expensive, though.
W: I like the songs you’ve composed so I’ve always W: In that case, there are cheaper ones with
1 ① 2 ⑤ 3 ① 4 ⑤ 5 ④ wanted to sing one of your songs. basically the same function. How about these?
6 ② 7 ① 8 ③ 9 ⑤ 10 ③
M: I’m flattered. Shall we get started? They cost 40 dollars each.
11 ② 12 ③ 13 ① 14 ① 15 ②
16 ⑤ 17 ② 18 ① 19 ① 20 ② M: Sounds better. I’ll buy two pairs of these. I also
4. [출제의도] 내용 불일치 고르기(그림)
21 ④ 22 ② 23 ④ 24 ⑤ 25 ④ need a swimming cap for myself.
W: Charles, come and look at this photo. My mom
26 ④ 27 ⑤ 28 ③ 29 ④ 30 ③ W: I recommend this new cap. It uses high­quality
took it last weekend.
31 ① 32 ③ 33 ③ 34 ⑤ 35 ④ materials and costs only 20 dollars.
36 ② 37 ③ 38 ④ 39 ③ 40 ① M: Oh, you went to that park that I really want to
M: Okay. Then I’ll buy two pairs of swimming
41 ⑤ 42 ⑤ 43 ④ 44 ② 45 ③ visit. The square sign here says Riverside Park.
goggles and one swimming cap.
W: Right. I went there on a picnic with my family.
W: We also have brand­new swimsuits in our store.
The weather was terrific.
Are you interested?
해 설 M: That’s wonderful! Why are those two balloons
M: No, thanks. These are enough.
tied to the tree?
W: Sounds good. We have a promotion this week, so
W: They’re in celebration of the second anniversary
you can get a 10% discount on the total amount.
1. [출제의도] 담화의 목적 고르기 of the park’s opening.
M: That’s great. Here’s my credit card.
M: Hello, I’m Jason Adams. I’ll be teaching M: How pretty! And you’re holding a book.
chemistry this semester. It’s very nice to meet W: Yes. It’s the book that you gave me as a 7. [출제의도] 5W1H(이유) 고르기
you all. Before we start, let me tell you the present on my birthday. That book was so M: Stella, what are you doing?
most important things to remember when you interesting. W: Well, I need to move out of my apartment, so
write your reports. You should use brief
M: I’m so glad to hear that. Actually, I was worried I’m searching for a moving company.
sentences to make them clear. If your reports
about whether you would like it. M: Why? Is your contract almost over?
are full of long and complex sentences, they’ll
W: It was a great choice! And look at this W: Not yet. I’ve been here for just five months,
be hard to understand. Also, be careful not to
star­patterned mat. The park gave it out for and I still have seven months left.
use difficult words without any explanations.
free as a second anniversary souvenir.
You may think it makes your reports look M: Oh, did you have some problem with your
professional, but the readers may be confused M: Cool. By the way, I wonder why that ball is on neighbors?
and you won’t be able to convey the intention of the table.
W: Not at all. They are all kind and I get along
your writing. You need to keep these things in W: My brother brought it to play soccer. with them.
mind when you write your reports. M: I see. You must’ve had a good time on the picnic. M: I heard you got a new job recently. Is it far
2. [출제의도] 의견 고르기 5. [출제의도] 할 일 고르기 from here?
W: No, it’s within five minutes’ walking distance.
W: Hey, Michael. What happened? You look tired. [Telephone rings.]
Actually, the issue is that my father is sick.
M: Hi, Claire. I can’t sleep well. My daughter has M: Hello, this is Sky Print Shop.
M: Oh, I’m sorry to hear that.
been up crying the past few nights. W: Hi, this is Lisa Morris. I’m a teacher from
W: It would be hard for my mother to take care of
W: It must be hard. What do you think is wrong? Evergreen High School. Could I speak with the
him alone, so I decided to move in with my
M: My wife and I have been trying to get her to manager, Mr. White?
parents to help look after my father.
sleep alone in her room. M: Speaking. How may I help you?
M: I hope your father gets better soon.
W: Ah, she might feel afraid because she’s not used W: Well, I checked the cover design of our club
W: Thank you for your concern.
to sleeping alone yet. Why don’t you give her activity book and I’d like to change it a bit. I’m
something she likes when she sleeps? afraid it seems too simple. 8. [출제의도] 세부 내용 고르기(언급 유무)
M: You mean, like blankets or stuffed animals? M: I see. How about adding student pictures or the W: Hi, Paul. What’s new?
W: Exactly! My son had the same problem for a school mascot?
M: Not much. I’m looking at the poster for the
while, but after sleeping with his favorite toy W: Sounds good. I think putting the school mascot Blueway Spelling Bee Competition.
robot, he’s come to sleep well on his own. on the cover will make it look better.
W: The spelling bee competition? What’s that?
M: So, do you think it really helped him feel more M: No problem. Could you send me the image
M: It’s a competition where participants are asked
comfortable and sleep better? later? Then I’ll add it.
to spell various words. The Blueway Library
W: I believe so. I’ve heard that young children’s W: Okay. Also, I emailed the introduction page of hosts it every year.
favorite items generally give them psychological the book to you this morning. Did you get a
W: That sounds interesting! Let me have a look.
comfort. chance to check it out?
Oh, it says any Blueway Library member can
M: Okay, I’ll give it a try. Thanks for your advice. M: Yes, I did. It looks pretty good. Are there any take part.
other changes you want to make?
3. [출제의도] 대화자의 관계 고르기 M: Yeah. I’m planning to enter the competition
W: No, that’s all. Do you think I can get the books because I want to test my spelling skills. Do
M: Sally, welcome to my studio. It’s good to see you. by next week? you want to join, too?
W: Thanks for inviting me, Chris. M: Of course. If you send the image file of the W: Sure, why not? How do we sign up for the contest?
M: I hear your new song everywhere these days. I mascot, we can add it to the cover, and then
M: We should email the application by Thursday.
like it a lot. start printing right away.
Here’s the email address on the bottom.
W: Oh, do you? I think it’s been popular since it W: I’ll send it to you now. Thank you!
W: Got it! Oh, they are only accepting 50 people.
was used in a TV commercial.
6. [출제의도] 숫자 정보 파악하기 M: Then we should hurry!
M: You must be thrilled. Did you listen to the file I
sent you? W: Good afternoon, sir. May I help you?
9. [출제의도] 담화 내용 불일치 고르기
W: Yes, I did. And I love it. How did you come up M: Yes, please. I’m looking for swimming goggles
W: Hello, I’m Grace, a manager of Dream Marine
with such a sweet and beautiful melody? for my kids.
Museum. This year, our museum has a special

8
2023학년도 6월
전국연합학력평가 정답 및 해설 고2

program, called Ocean World. The topic of the W: It’s yellow and it has a wooden handle. There’s through blue logos. So, to design your brand
program is deep­sea exploration. Through this a big picture of a flower on it. effectively, use colors wisely!
program, children will become interested in the M: Okay, I’ll check if it’s in the Lost and Found box
16. [출제의도] 긴 담화 듣기(담화의 주제 파악하기)
plants and fish living deep in the ocean. It runs right now... [Pause] Oh! We have an umbrella
from 10 a.m. to 6 p.m. this Saturday, October that looks like yours. Is this it? 17. [출제의도] 긴 담화 듣기(담화의 언급 유무 고르기)
28. This event will be held on the first floor.
W: Wow, that’s mine!
You can watch videos of deep­sea exploration 18. [출제의도] 글의 목적 고르기
M: Wait. Here’s a note that says a lady found it and
on a huge screen in the central hall. There will 친애하는 부모님께,
brought it to us.
be a photo booth where you can take selfies 학생의 잠재력을 극대화하는 데는 학교에 규칙적으로
wearing various masks of ocean creatures. In W:
출석하는 것이 필수적입니다. 최근에, 우리는 전 학년에
addition, you can make fish­shaped cookies with 14. [출제의도] 긴 대화의 적절한 응답 고르기 걸쳐 승인되지 않은 결석 수에 대해 우려하고 있습니
a chef. Anyone can participate in Ocean World 다. 저는 부모로서의 당신의 역할이 학교 결석을 승인
W: Fred, I got the blueberry juice we ordered.
for free without having to make a reservation. I 하는 것이라는 것을 더 명확히 하고 싶습니다. 학부모
hope to see you all there. M: Thanks, Kathy. Oh, it has a paper straw. Don’t
들은 결석 첫날로부터 7일 이내에 결석에 대한 설명을
they have a plastic one?
학교에 제공해야 합니다. 7일의 기간 내에 설명이 주어
10. [출제의도] 세부 내용 고르기(도표) W: What’s wrong with a paper straw? 지지 않을 경우, 학교는 결석을 정당하지 않은 것으로
W: Honey, here’s a flyer about smart security M: It can get soaked and fall apart while drinking. 학생부에 기록할 것입니다. 반드시 학부모 포털 사이트
cameras. They’re on sale! So I prefer a plastic straw. 에 들어가서 자녀가 결석할 때마다 사유를 등록해 주십
M: Cool! I’ve been thinking about buying one to W: But plastic straws can harm the environment. 시오. 자녀가 불이익에 처하지 않도록 모든 결석을 승
protect our house before we’re away on M: Protecting the environment is all good, but my 인해 주십시오. 협조해 주셔서 대단히 감사합니다.
vacation. drink tastes bad with paper straws. 진심으로,
W: Same here. Which one do you want to get? W: If so, there are other types of straws to enjoy 교감, Natalie Brown
M: Hmm... I don’t think we can spend more than your drink that cause less harm to the 19. [출제의도] 심경 변화 고르기
$500 on a security camera. environment.
밖에서 호버 엔진의 윙윙거리는 소리가 들리자마자
W: You’re right. We’ve already spent a lot for our M: I didn’t know there were other kinds. Can you Ester는 일어섰다. “편지,”라고 외치며 그녀는 계단을 세
vacation. name some of them? 칸씩 뛰어내려 가 문을 확 열었다. 비가 쏟아지고 있었지
M: There are two types of connectivity options, W: Have you heard of a silicone straw? 만 그녀는 빗속으로 뛰어나갔다. 그녀는 우체통을 마주하
wired and wireless. M: No, I haven’t. It sounds interesting. 고 있었다. 안에는 뜯지 않은 편지 한 통이 들어 있었다.
W: I prefer wireless so that we can install it W: There are also straws made of glass or metal. 그녀는 이것이 그녀가 간절히 기다리고 있는 것임에 틀림
anywhere we want. They’re reusable and don’t affect the taste of 없다고 확신했다. 망설임 없이 그녀는 봉투를 뜯어서 열
M: Good idea. We need one that can be installed your drink. 었다. 그녀는 종이를 꺼내 펼쳤다. 편지에는 ‘우리 회사에
indoors and outdoors as well. 지원해 주셔서 감사합니다. 우리는 당신을 인턴십 프로그
M:
W: I agree. Now we have two options left. Well... 램에 초대하고 싶습니다. 우리는 당신을 곧 뵙기를 기대
The one with sound detection looks attractive to 15. [출제의도] 상황에 적절한 응답 고르기 합니다.’라고 쓰여 있었다. 그녀는 펄쩍펄쩍 뛰며 다시 편
me. M: Nick and Annie are partners for an art project. 지를 내려다보았다. 그녀는 이 소식을 가족들에게 빨리
They are supposed to make a presentation on the 전하고 싶었다.
M: I heard that sound detection is not as accurate
as motion detection. life of Vincent van Gogh. However, while 20. [출제의도] 필자의 주장 고르기
W: Really? Then, we should definitely get one with researching and working on their project, they
신기술의 도입은 지속 가능한 발전에 긍정적인 영향
motion detection. realized that some of the content they found was
과 부정적인 영향을 분명히 미친다. 기술 자원을 잘 관
conflicting. For example, there was a disagreement
M: Okay. Let’s buy this one. 리하려면 그것들을 충분히 고려해야 한다. 원자력과 농
about when exactly Van Gogh started painting.
업과 같은 분야의 기술 발전은 환경적 이익뿐만 아니라
11. [출제의도] 짧은 대화의 적절한 응답 고르기 Nick gathered his information from the books in
환경이나 인간의 건강에 대한 위험이 어떻게 기술 발전
M: Jenny, thanks for helping me prepare my speech the library, while Annie collected hers from the
에 수반될 수 있는지에 대한 예를 제공한다. 새로운 기
for the graduation ceremony. Internet. Nick and Annie are not sure which one
술은 또한 심오한 사회적 영향을 끼친다. 산업혁명 이
to trust. Nick wants their presentation to be as
W: There was not much to help. You were already 후 기술의 발전은 직장에서 요구되는 기술의 본질을 변
precise as possible, so he thinks that they should
well­prepared. 화시켜 고용 패턴에 영향을 미치며 특정 유형의 일자리
ask their professor for help with deciding which
M: I’m still worried. Do you really think people will 를 창출하고 다른 유형의 일자리는 소멸시켰다. 신기술
source is more reliable. In this situation, what
understand what I’m trying to say? 은 긍정적이고 부정적인, 모든 잠재적 영향들에 대해
would Nick most likely say to Annie?
W: 평가되어야 한다.
[16~17] 21. [출제의도] 함축적 의미 파악하기
12. [출제의도] 짧은 대화의 적절한 응답 고르기
W: Welcome back to Tips for Entrepreneurs. Last 추수감사절 칠면조와 같은 몇 가지 전해 내려오는 풍
W: Steve, do you know where my earphones are? I time, we talked about this year’s trendy colors. 속을 제외하고, 북미의 토착 요리는 원주민들과 같은 불
need them right now but can’t find them Now let’s talk about how to use them in design 행한 운명을 맞이했다. 확실히, 우리는 여전히 지역 특
anywhere. for your brand. Understanding the art of using 색 음식을 가지고 있지만, Carolina 바비큐는 거의 확실
M: I saw Dad taking them this morning for his colors can help create more effective and 히 California 토마토를 소스에 넣을 것이고, Louisiana
business trip. memorable designs. For example, using red for 검보도 인도네시아 양식 새우를 포함할 것이다. 만약 이
W: Oh, no! That means I can’t use them until this important elements will make it hard to ignore 것들 중 하나가 지방이나 액상 과당이 많이 첨가되어 패
Saturday! them. It’s no coincidence that so many fast food 스트푸드 메뉴에 나타난다면, 우리는 그 붕괴를 식별하
M: logos are red. Orange communicates creativity and 거나 막을 수 없을 것 같다. 우리는 아직 우리의 땅과
youth, which is perfect for a bold brand. If you’re 기후가 우리에게 주는 것을 음미하고 현명하게 소비하기
13. [출제의도] 긴 대화의 적절한 응답 고르기 trying to get your customers excited about 위해, 가계를 통해 전해져 내려오는 강력한 일반화된 규
M: Hello, is there anything I can do for you? something, go for orange. Green, on the other 범을 생각해내지 못했다. 대신, 우리는 전국적인 베스트
W: My umbrella is missing. I thought I might have hand, can communicate lots of different ideas, like 셀러의 규모로 서점과 배에 연이어 큰 소동을 일으키는
left it in the restroom, but when I checked, it luck, wealth, and freshness. Green is also a 일련의 유행하는 식단을 가지고 있다. 10명 중 9명의
wasn’t there. common choice to communicate environmentalism 영양학자들은 이것을 우리가 완전히 우리의 분별력을 잃
and sustainability, as well as comfort and 었다는 증거로 본다.
M: Which restroom did you use?
prosperity. Lastly, for a cool and calm feel,
W: The one on the third floor. [어구] corruption 붕괴
choose blue. Brands that supply ice or cooling
M: What does your umbrella look like? Could you solutions can communicate this literal coolness
describe it?

9
2023학년도 6월
전국연합학력평가 정답 및 해설 고2

의 길로 들어섰다. 1650년 병에서 회복하기 위해, 그


22. [출제의도] 글의 요지 파악하기 30. [출제의도] 문맥상 적절한 어휘 파악하기
는 자연을 산책하기 시작했고 식물학에 대한 관심을
아마도, 직장에서 인공 지능(AI)의 출현은 감성 지능 지난 수십 년 동안, 가난한 나라들의 부채를 줄이기
키웠다. 부유한 학생이자 후원자인 Francis Willughby
(EI)에 좋은 징조가 될 수 있다. AI가 추진력을 받고 위한 몇 가지 합의가 있었지만, 다른 경제적 과제(무역
와 함께 Ray는 1660년대에 영국과 유럽을 여행했고
모든 수준의 일자리에서 사람들을 대신함에 따라, 높은 장벽과 같은)는 남아 있다. 할당제, 보조금, 수출 제한과
식물과 동물을 연구하고 수집했다. 그는 1673년
EI 능력을 가진 사람들에게 프리미엄이 주어질 것이라 같은 비관세 무역 조치가 점점 더 널리 퍼지고 있으며
Margaret Oakley와 결혼했고, Willughby 집안을 떠난
는 전망이 있다. 사람들이 상호 작용하는 동안 보내고 무역과 무관한 정책적 이유로 제정될 수 있다. 그러나
후에는 Black Notley에서 77세까지 조용히 살았다. 그
반응하는 감정적인 메시지들은, 이러한 점에서, AI 프 그것들은 부유한 국가들에 의해 부과된 비관세 조치의
는 동식물 목록을 만들기 위해 표본을 연구하면서 말
로그램의 모방하는 능력을 훨씬 넘어선다. 우리가 스마 요건을 준수할 자원이 부족한 국가들의 수출에 차별적인
년을 보냈다. 그는 식물과 그 형태, 기능뿐만 아니라
트 기기의 시대로 접어들수록, 감정을 감지하고 관리하 효과를 가진다. 예를 들어, 가난한(→부유한) 국가들이
신학과 그의 여행에 관한 20편 이상의 저서를 썼다.
는 것은 AI를 당혹하게 하는 지능의 한 유형으로 남을 자국의 농부들에게 주는 막대한 보조금은 전 세계 나머
것이다. 이것은 EI와 관련된 사람들과 직업들이 기계에 [어구] blacksmith 대장장이 botany 식물학 지 국가들의 농부들이 그들과 경쟁하는 것을 매우 어렵
의해 점령되는 것으로부터 안전하다는 것을 의미한다. 게 만든다. 또 다른 예는 국내 보건 또는 안전 규제인데,
27. [출제의도] 내용 불일치 고르기(실용문)
한 설문 조사에서, 일상적인 업무의 자동화가 정서적 이것은, 구체적으로 수입을 목표로 삼진 않지만, 수입자
기술이 필요한 활동에 효과적인 AI를 만드는 것이 불가 평화 마라톤 축제
시장에 순응하고자 하는 외국 제조업체에 상당한 비용을
능하다는 점에 부딪히면서, 임원 네 명 중 세 명 가량 평화 마라톤 축제는 세계 평화를 장려하고 도움이 필
부과할 수 있다. 개발도상국 시장의 산업은 이러한 추가
이 EI를 향후 직장의 “필수” 기술로 보고 있다. 요한 사람들을 위한 온정을 나누기 위해 개최됩니다.
비용을 부담하는 데 더 많은 어려움을 겪을 수 있다.
달리기를 즐기는 데 동참하고 더 나은 세상을 만들어
23. [출제의도] 글의 주제 파악하기 주세요. 31. [출제의도] 빈칸 추론하기
교육은 나뭇가지, 잔가지, 잎이 모두 공통의 핵심에서 언제 & 어디서 비즈니스 전략과 환경을 연구하는 과정에서, Michael
나오는 방식을 밝히면서, 지식의 나무 줄기에 초점을 맞 · 2023년 9월 3일, 일요일 Porter는 기업이 규제로부터 이익을 얻는 것처럼 보인
춰야 한다. 다양한 분야의 실무자들이 혁신 과정에 대한 (출발 시각: 오전 10시) 다는 독특한 패턴을 발견했다. 그는 또한 더 엄격한 규
경험을 공유하고 그들의 창의적 활동 사이의 연결 고리 · 시민 스타디움 제가 느슨한 규제보다 더 많은 혁신을 유발하고 있다는
를 발견할 수 있는 공통 언어를 제공하면서, 사고를 위 참가비 & 자격 것을 발견했다. 네덜란드의 꽃 산업은 하나의 예시이다.
한 도구는 이 핵심에서 비롯된다. 교육과정 전반에 걸쳐 · 풀 & 하프: $30 (20세 이상) 수년 동안, 네덜란드의 세계적으로 유명한 튤립과 다른
동일한 용어가 사용될 때, 학생들은 서로 다른 과목들과 · 10km & 5km: $15 (나이 제한 없음) 꽃들을 생산하는 회사들은 또한 비료와 농약으로 그 나
수업들을 연결하기 시작한다. 글쓰기 수업에서 추상을 등록 라의 물과 토양을 오염시키고 있었다. 1991년, 네덜란
연습하고, 회화나 그림 그리기 수업에서 추상을 연습하 · 참가자는 1,000명으로 제한됩니다. 드 정부는 2000년까지 농약 사용을 절반으로 줄이도록
고, 그리고 모든 경우에 그들이 그것을 추상이라고 일컫 (선착순입니다.) 고안된 정책을 채택했는데, 이것은 그들이 궁극적으로
는다면, 그들은 학문의 경계를 넘어 사고하는 방법을 이 · ipmarathon.com에서 온라인으로만 달성한 목표였다. 점점 더 엄격한 규제에 직면하면서,
해하기 시작한다. 그들은 그들의 생각을 하나의 개념과 참고 온실 재배자들은 더 적은 양의 농약으로 상품의 품질을
표현 방식에서 다른 방식으로 바꾸는 방법을 알게 된다. · 기념품과 메달이 모든 참가자들에게 주어집니다. 유지하려면 새로운 방법을 개발해야만 한다는 것을 깨
용어들과 도구들이 보편적 상상력의 일부로 제시될 때 · 탈의실은 무료로 이용 가능합니다. 달았다. 이에 그들은 폐쇄 루프 방식으로 물을 순환시
학문들을 연결하는 것은 자연스럽게 이루어진다. · 물은 2.5km마다 그리고 결승선에서 제공됩니다. 키고 암모 배양판에서 꽃을 키우는 재배 방식으로 전환
[어구] twig 잔가지 disciplinary 학문의, 교과의 했다. 새로운 시스템은 환경에 배출되는 오염을 감소시
28. [출제의도] 내용 일치 고르기(실용문)
켰을 뿐만 아니라, 회사들이 재배 조건을 더 잘 통제할
24. [출제의도] 글의 제목 추론하기 Out to Lunch
수 있게 함으로써 이익을 증가시켰다.
새로운 상황, 생각, 감정에 반응하여 새로운 단어들과 맛있는 음식, 좋은 음악과 함께 오후를 즐기고 싶으
표현들이 계속해서 생겨난다. Oxford 영어 사전은 그 언 세요? ‘Out to Lunch’는 당신의 요구를 충족시켜 주는 [어구] fertilizer 비료 circulate 순환시키다
어에 등장한 새로운 단어들과 표현들의 추가분을 출판한 더할 나위 없는 행사입니다! 오셔서 Missoula 시내의
32. [출제의도] 빈칸 추론하기
다. 어떤 사람들은 이런 일을 한탄하고 그것을 올바른 Caras 공원에서 열리는 이 행사를 즐기세요!
빠르지만 고도로 숙련된 사람에게 더 많은 돈을 지불
영어에서 벗어난 것으로 본다. 그러나 영어의 철자와 구 날짜 & 시간
하기는 어려운데, 그 이유는 단순히 관찰되는 노력이
두법을 공식화하려는 시도는 18세기에 이르러서야 이루 · 6월 매주 수요일, 오후 12시 – 오후 3시
적기 때문이다. 두 명의 연구원이 사람들에게 데이터
어졌다. 21세기에 우리가 사용하는 언어는 Shakespeare 주요 특징
복구에 얼마를 지불할 것인지를 묻는 연구를 한 적이
에게는 사실상 이해되기 어려울 것이며, 우리에게도 그 · Diamond 아이스크림을 포함한 모든 푸드 트럭에서 10% 할인
있다. 그들은 사람들이 더 많은 양의 복구된 데이터에
의 말하는 방식은 마찬가지일 것이다. Alvin Toffler는 · 신인 그룹 Cello Brigade의 라이브 음악 공연
대해 조금 더 많은 돈을 지불할 것이라는 것을 발견했
Shakespeare가 현재 영어에서 일반적으로 사용되는 · 아이들을 위한 페이스 페인팅과 물풍선 놀이
지만, 사람들이 가장 민감하게 여기는 것은 기술자가
450,000개의 단어 중 약 250,000개만을 이해할 것이라 공지
일한 시간이었다. 데이터 복구에 몇 분밖에 걸리지 않
고 추정했다. 다시 말해서, 말하자면, 만약 Shakespeare · 개인 (야외용) 접의자와 담요를 가져오세요.
았을 때, 지불 의사가 낮았지만, 같은 양의 데이터를 복
가 오늘날 런던에 나타난다면, 그는 평균적으로 우리의 · 개인 쓰레기를 올바르게 처리해 주세요.
구하는 데 일주일 이상이 걸렸을 때, 훨씬 더 많은 비
어휘에 있는 9개의 단어당 5개만 이해할 것이다. · 주류를 마시는 것은 엄격하게 금지됩니다.
용을 지불할 의사가 있었다. 생각해 보라. 그들은 같은
[어구] unintelligible 이해하기 어려운 29. [출제의도] 문법성 판단하기 결과에 대해 더 느린 서비스에 더 많은 비용을 기꺼이
연구 심리학자들은 종종 자기 보고 데이터로 작업을 하 지불하고자 했다. 근본적으로, 우리가 결과보다 노력을
25. [출제의도] 내용 불일치 고르기(도표)
는데, 이는 참가자들의 행동에 대한 구두 설명으로 구성 중시할 때, 우리는 무능함에 비용을 지불하는 것이다.
위 그래프는 선정된 국가들의 2019년 공립 초·중등학 비록 그것이 실제로는 비합리적이지만, 우리는 무능함
되어 있다. 변인을 측정하기 위해 설문지, 면접 또는 성
교 교사 1인당 평균 학생 수를 보여준다. 벨기에는 공
격 목록이 사용될 때마다 이에 해당한다. 자기 보고 방법 에 지불하면서, 더 이성적이고, 더 편하다고 느낀다.
립 초등학교와 중등학교 모두에서 교사 1인당 학생 수
은 꽤 유용할 수 있다. 그것들은 사람들이 자신을 풀타임
가 OECD 평균보다 적은 유일한 나라였다. 공립 초등학 [어구] incompetence 무능함
으로 관찰할 수 있는 유일한 기회를 가진다는 사실을 이
교와 중등학교 모두에서, 교사 1인당 평균 학생 수는
용한다. 그러나, 자기 보고는 몇 가지 종류의 왜곡으로 33. [출제의도] 빈칸 추론하기
멕시코에서 가장 많았다. 공립 초등학교에서는 교사 1
인해 오염될 수 있다. 이러한 왜곡 중 가장 문제가 되는 청소년기에 우리 중 다수는 위대한 책이나 작가의 영향
인당 평균 학생 수가 일본보다 독일에서 적은 반면, 공
하나는 사회적 바람직성 편향인데, 이것은 사회적으로 승 을 받은 경험이 있다. 우리는 책 속의 참신한 아이디어에
립 중등학교에서는 그 반대였다. 독일에서 공립 중등학
인된 답을 자신에 관한 질문에 제공하는 경향이다. 이러 매료되었고, 영향에 매우 열려 있었기 때문에, 흥미로운 아
교의 교사 1인당 평균 학생 수는 영국의 절반보다 적었
한 편향에 영향을 받은 피실험자들은 특히 민감한 문제에 이디어와의 이러한 초기 만남은 우리의 마음속 깊이 가라
다. 5개국 중 멕시코는 공립 중등학교의 교사 1인당 학
대해 질문받을 때 호의적인 인상을 만들기 위해 추가적으 앉았고 우리 자신의 사고 과정의 일부가 되었고, 그것들을
생 수가 공립 초등학교보다 많은 유일한 나라였다.
로 노력한다. 예를 들어, 많은 설문 조사 응답자들은 사 흡수한 지 수십 년이 지난 후에 우리에게 영향을 미쳤다.
26. [출제의도] 내용 불일치 고르기 실은 하지 않았다고 결정하는 것이 가능할 때 선거에서 그러한 영향들은 우리의 정신적 풍경을 풍부하게 했고, 사
1627년 잉글랜드 Essex주 Black Notley에서 태어난 투표했다거나 자선 단체에 기부했다고 보고할 것이다. 실 우리의 지성은 더 나이가 많고 더 현명한 사람들의 교
John Ray는 마을 대장장이의 아들이었다. 16세에 그 훈과 생각을 흡수하는 능력에 달려 있다. 그러나, 나이가
[해설] ④ ask → are asked
는 Cambridge 대학교에 들어가서 폭넓게 공부하고 그 들면서 몸이 경직되는 것처럼 마음도 그러하다. 그리고 약
[어구] variable 변인 점과 취약성에 대한 우리의 깨달음이 학습 욕구를 자극했
리스어부터 수학까지 강의를 하다가 1660년에 성직자

10
2023학년도 6월
전국연합학력평가 정답 및 해설 고2

듯이, 슬며시 다가오는 우월감도 새로운 생각과 영향력에 보존된 Monteverde Cloud Forest Reserve에서 살았음에 일 때 컴퓨터 화면에 나오는 제품들에 호의적으로 반응
대해 서서히 우리를 닫는다. 어떤 사람들은 현대 세계에서 도 불구하고, 1989년쯤, 황금 두꺼비는 외관상으로 멸종 했는데, 이는 그들의 결정이 그들의 행동에 의해서 무
우리가 모두 더 회의적으로 된다고 주장할지도 모르지만, 하였다. 그것의 열대 우림 서식지를 지탱해 준 습기의 많 의식적으로 영향을 받는다는 것을 보여주었다.
사실 훨씬 더 큰 위험은 우리가 나이가 들수록 개인으로서 은 부분은 카리브해에서 불어 들어오는 습기를 실은 구름
[41~42]
우리에게 부담을 주고, 일반적으로 우리의 문화에 부담을 의 형태에서 왔다. (A) 하지만 세계적 기후 변화로 인한
당신의 뇌가 기억된 사건들의 창고에서 그것들에 접
주는 것처럼 보이는 점차적인 마음의 폐쇄에서 온다. 더 따뜻한 공기가 이러한 구름들을 상승하게 했고, 숲에서
근하려고 할 때, 평범하지 않은 사건들이나 경험들이
습기를 제거하였으며, 황금 두꺼비와 많은 다른 종들의
[어구] vulnerability 취약성 advocate 주장하다 더 잘 기억되는 경향이 있는데 그 이유는 그것들과 경
서식지가 완전히 말라 버렸다. 황금 두꺼비는 주로 지구
쟁하는 것이 없기 때문이다. 다시 말해, 2주 전 목요일
34. [출제의도] 빈칸 추론하기 온난화로 인한 기후 변화의 첫 희생양들 중 하나인 것 같다.
에 아침 식사로 무엇을 먹었는지 기억하는 것이 어려울
많은 사람이 대중적인 사고에서 안전과 안심을 찾는다. [어구] deprive 제거하다 수 있는 이유는 아마도 그 목요일이나 그 특정 아침 식
그들은 만약 많은 사람이 무언가를 하고 있다면, 그것은 사에 대해 특별한 것이 없었기 때문이다 — 그 결과, 당
틀림없이 옳을 것으로 생각한다. 그것은 좋은 생각임이 틀 38. [출제의도] 문장 삽입하기
신의 모든 아침 식사 기억은 일종의 일반적인 아침 식
림없다. 만약 대부분의 사람들이 그것을 받아들인다면, 그 실험 방법의 근본적인 본질은 조작과 통제이다. 과학 사에 대한 인상으로 합쳐진다. 여러분의 기억력은 유사
것은 아마도 공정함, 평등함, 동정심, 그리고 민감성을 상 자들은 관심 변인을 조작하고, 차이가 있는지 확인한다. 한 사건들을 병합하는데, 그것은 그렇게 하는 것이 더
징할 것이다, 그러한가? 꼭 그렇다고 할 수는 없다. 대중적인 동시에, 다른 모든 변인의 잠재적 영향을 통제하려고 시 효율적일 뿐만 아니라, 이것이 우리가 어떤 것들을 배
사고는 지구가 우주의 중심이라고 했지만, Copernicus는 도한다. 사건의 근본적인 원인을 식별하는 데 있어 통제 우는 방법의 기본이기 때문이다 — 우리의 뇌는 경험을
별과 행성을 연구했고 지구와 태양계의 다른 행성들이 태 된 실험의 중요성은 아무리 강조해도 지나치지 않다. 현 함께 묶는 추상적인 규칙들을 추출한다.
양 주위를 돈다는 것을 수학적으로 증명했다. 대중적인 사 실의 통제되지 않은 세계에서, 변인들은 종종 상관관계 이것은 일상적인 것들에 특히 해당된다. 만약 당신의
고는 수술이 깨끗한 도구를 필요로 하지 않는다고 말했지 가 있다. 예를 들어, 비타민 보충제를 섭취하는 사람들은 아침 식사가 항상 같다면 — 예를 들어, 우유를 곁들인
만, Joseph Lister는 병원에서의 높은 사망률을 연구했고 비타민을 섭취하지 않는 사람들과는 다른 식습관과 운동 시리얼, 오렌지 주스 한 잔, 커피 한 잔 — 당신의 뇌가
즉시 생명을 구하는 멸균법을 소개했다. 대중적인 사고는 습관을 지닐 수 있다. 그 결과, 만약 우리가 비타민의 건 특정한 한 아침 식사에서 그 세부 사항을 추출하는 것
여성들이 투표권을 가져서는 안 된다고 했지만, Emmeline 강에 미치는 효과를 연구하고 싶다면, 우리는 단지 현실 은 쉽지 않다. 아이러니하게도, 일상화된 행동의 경우,
Pankhurst와 Susan B. Anthony 같은 사람들은 그 권리를 세계만 관찰할 수 없는데, 왜냐하면 이러한 요소(비타민, 당신은 그 행동의 일반적인 내용(당신이 먹었던 것과
위해 싸웠고 쟁취했다. 우리는 항상 수용과 지성 사이에 식단, 운동) 중 어느 것이든 건강에 영향을 미칠 수 있기 같은, 당신이 항상 같은 것을 먹기 때문에)은 기억할
큰 차이가 있다는 것을 기억해야 한다. 사람들은 수가 많 때문이다. 오히려, 우리는 현실 세계에서 실제로 일어나 수 있지만, 그 한 가지 예의 세부 사항들(쓰레기 트럭
은 편이 더 안전하다고 말할지도 모르지만, 그것이 항상 지 않는 상황을 만들어야 한다. 그것이 바로 과학 실험 이 지나가는 소리 또는 당신의 창문을 지나치는 새소리
사실인 것은 아니다. 이 하는 일이다. 그것들은 그 밖의 다른 모든 것을 일정 와 같은)은 그것들이 매우 특이하지 않다면 상기하기가
하게 유지하면서, 한 번에 하나의 특정 변인을 조작하여 매우 어려울 수 있다. 반면에, 만약 당신이 당신의 일상
35. [출제의도] 무관한 문장 파악하기 세상에서 자연적으로 발생하는 관계를 분리하려고 애쓴다. 을 깨뜨리는 특이한 일을 했다면 — 아마도 당신은 아침
런던에서 택시 운전면허를 받기 전에, 사람은 “The
[어구] manipulate 조작하다 supplement 보충제 식사로 남은 피자를 먹고 드레스 셔츠에 토마토 소스를
Knowledge”라는 위협적인 이름의 매우 어려운 시험을 통과
쏟았다 — 당신은 그것을 덜(→더) 기억하기가 쉽다.
해야 한다. 이 시험은 Greater London 지역의 2만 개 이상 39. [출제의도] 문장 삽입하기
거리의 구획을 암기하는 것을 포함하는데, 이는 엄청난 양 [어구] generic 일반적인
왜 지중해 지역의 사람들은 더 오래 살고 질병 발생률
의 기억 자원을 포함하는 기술이다. 사실, 택시 운전사 훈련 이 더 낮을까? 몇몇의 사람들은 그것이 그들이 먹는 것 41. [출제의도] 1지문 2문항(글의 제목 추론하기)
에 등록한 사람 중 50% 미만이 시험을 통과하는데, 심지어 때문이라고 말한다. 그들의 식단은 신선한 과일, 생선,
그것을 위해 2, 3년을 공부한 후에도 말이다! 그리고 밝혀 채소, 통곡물, 견과류로 가득하다. 이러한 문화권의 사람 42. [출제의도] 1지문 2문항(문맥상 적절한 어휘 파악하기)
진 바에 따르면, 런던 택시 운전사들의 두뇌는 그들의 초인 들은 적포도주를 마시고 많은 양의 올리브유를 사용한다.
적인 기억 노력을 반영하는 방식에서 택시 운전을 하지 않 [43~45]
왜 그러한 음식 패턴이 건강에 좋은가? 한 가지 이유는 (A) Henrietta는 가장 위대한 “노래의 여왕” 중 한 명
는 사람들과 다르다. (즉, 그들은 운전 면허청에서 발급된
그들이 다양한 색깔을 먹고 있기 때문이다. 식품에 존재 이다. 그녀는 독일이 배출한 가장 위대한 가수로서 그녀
정식 운전면허증을 최소 1년 동안 소지해야 한다.) 사실, 공
하는 수천 가지의 다채로운 “생화학 물질”(phyto=식물) 가 부러워할 만한 위치에 도달하기 전에 혹독한 시련을
간 기억과 가장 자주 연관되어 온 뇌의 부분, 해마라 불리
의 이점을 보여주는 점점 더 많은 연구가 표면화되고 있 겪어야 했다. 그녀의 경력 초기에 그녀는 경쟁자
는 해마 모양을 한 뇌 영역의 꼬리 부분은 이들 택시 운전
다. 식물에 있는 건강에 좋고, 영양가 없는 이 화합물들 Amelia의 친구들에 의해 비엔나 무대에서 야유를 받고
사들에게서 평균보다 더 크다.
은 식물에 색과 기능을 제공하고 인체의 건강에 보탬이 쫒겨났다. 그러나 이 좌절에도 불구하고, Henrietta는
[어구] intimidating 위협적인 된다. 각각의 색깔은 몸에서 특정 기능을 하는 특정 화 모든 유럽이 그녀의 발아래에 있을 때까지 견뎠다.
합물과 연결된다. 예를 들어, 만약 당신이 보라색 음식을 (D) 수년 후, Henrietta의 명성이 절정에 달했을 때,
36. [출제의도] 글의 순서 파악하기
먹지 않는다면, 당신은 중요한 뇌 보호 화합물인 안토시 그녀는 어느 날 베를린의 거리를 차를 타고 지나가고
정책을 평가할 때, 사람들은 그것이 어떤 특정한 문제
아닌을 아마도 놓치고 있는 것이다. 그와 유사하게, 만약 있었다. 곧 그녀는 눈먼 여성을 데리고 가는 여자 아이
를 어떻게 해결할 것인가에 집중하는 경향이 있으며, 그
당신이 녹색 음식을 피한다면, 세포가 손상되는 것을 막 와 마주쳤다. 그녀는 여성의 무력함에 마음이 움직였고,
정책이 가질 수 있는 다른 효과는 무시하거나 경시한다.
아주는 식물 산화 방지제인 엽록소가 부족할 수도 있다. 충동적으로 아이를 그녀에게 오라고 손짓하며, “이리
경제학자들은 종종 이 상황을 의도하지 않은 결과의 법칙
와, 얘야. 네가 손을 잡고 데리고 가는 사람은 누구니?”
이라고 부른다. (B) 예를 들어, 국내 철강 노동자들의 일 [어구] anthocyanins 안토시아닌
라고 말했다.
자리를 보호하기 위해 수입된 철강에 관세를 부과한다고
40. [출제의도] 문단 요약하기 (B) 대답은, “저분은 제 어머니, Amelia Steininger 입
가정해 보자. 만약 당신이 충분히 높은 관세를 부과한다
사람들은 특정한 생각을 할 때 매우 예측 가능한 방식 니다. 그녀는 훌륭한 가수였지만, 목소리를 잃었고, 그
면, 그들의 일자리는 실제로 외국 철강 회사들과의 경쟁
으로 행동한다. 그들은 동의할 때, 고개를 끄덕인다. 여 일로 너무 많이 울어서 그녀는 이제 더 이상 앞을 볼 수
으로부터 보호될 것이다. (A) 그러나 하나의 의도하지 않
기까지는, 놀랄 일은 아니다, 하지만 “고유 수용 심리학” 없습니다.” 였다. Henrietta는 그들의 주소를 묻고 나서
은 결과는 일부 자동차 노동자들의 일자리가 외국 경쟁사
으로 알려진 한 연구 분야에 따르면, 그 과정은 역으로 아이에게 “어머니께 오래된 지인이 오늘 오후에 그녀를
에 빼앗기게 된다는 것이다. 왜일까? 철강 노동자들을 보
도 작용한다. 사람들을 특정한 방식으로 행동하게 하면 방문할 것이라고 말하렴.”이라고 말했다. 그녀는 그들의
호하는 관세는 국내 자동차 제조업체들이 자동차를 만드
당신은 그들이 특정한 생각을 갖도록 한다. 그 아이디어 거처를 찾아내서 모녀를 돌보았다. 그녀의 요청에 따라
는 데 필요한 철강의 가격을 높인다. (C) 그 결과, 국내
는 처음에는 논란의 여지가 있었지만, 다행히도 설득력 숙련된 의사가 Amelia의 시력을 회복시키려 했지만, 허사였다.
자동차 제조업체들은 자동차 가격을 인상해야 하고, 국산
있는 실험으로 뒷받침되었다. 한 연구에서 참가자들은 (C) 그러나 Henrietta가 그녀의 예전 경쟁자에게 베푼
차를 외제 차에 비해 상대적으로 덜 매력적이게 만든다.
큰 컴퓨터 화면을 가로질러 움직이는 다양한 제품들에 친절은 여기서 그치지 않았다. 그 다음 주에 그녀는 그
가격을 올리는 것은 국산 차 판매를 줄이는 경향이 있어
시선을 고정하고 그 제품들이 그들에게 매력적인지 아닌 불쌍한 여성을 위한 자선 콘서트를 열었고, 그 자리에서
서, 일부 국내 자동차 노동자들은 일자리를 잃는다.
지를 나타내도록 요청받았다. 일부 제품은 수직으로 움 Henrietta는 그녀가 전에 한번도 불러본 적이 없는 방식
37. [출제의도] 글의 순서 파악하기 직였고(참가자들이 보는 동안 고개를 끄덕이게 하면서), 으로 불렀다고 한다. 그리고 많은 청중의 박수와 함께
오직 한 지역에서만 발견되는 종들은 토착종이라고 불 다른 제품은 수평으로 움직였다(좌우로 머리를 움직이게 지상 사람들의 선행에 기뻐하는 천국에 있는 천사들의
리고 특히 멸종에 취약하다. (B) 그들은 섬들과 특히 대 하면서). 참가자들은 자신의 “예”와 “아니요”의 머리 움 박수가 섞여 있었다는 것을 누가 의심할 수 있겠는가?
부분의 종이 매우 특화된 열대 우림인 다른 독특한 작은 직임이 결정에 핵심적인 역할을 했다는 사실을 인지하지 43. [출제의도] 1지문 3문항(글의 순서 파악하기)
지역에 있다. 한가지 예는 코스타리카의 산악 지역에 있 못한 채 수직으로 움직이는 제품을 선호했다.
는 무성한 열대 우림의 작은 지역에서만 한 때 발견되었  44. [출제의도] 1지문 3문항(지칭 추론하기)
던 번쩍이는 색깔의 황금 두꺼비이다. (C) 그 나라의 잘 한 연구에서, 참가자들은 그들의 고개를 위아래로 움직
45. [출제의도] 1지문 3문항(세부 내용 파악하기)

11
2023학년도 6월
전국연합학력평가 정답 및 해설 고2

광종은 과거제를 실시하였고, 공신 및 호족 세력의 14. [출제의도] 조선 후기 경제 상황 파악하기


• 한국사 영역 • 경제력과 군사력을 약화시키고 국가의 기반을 강화 제시된 자료는 대도시 주변의 농촌에서 상품 작물을
하기 위해 노비안검법을 실시하였다. 재배하여 도시로 팔러 가는 상황과 독점적 도매 상인인
[오답풀이] ① 조선 후기 일부 실학자들을 중심으로 도고의 출현을 나타내고 있다. 조선 후기에는 농업
제기되었다. ② 고려 말 ④ 고려와 몽골의 전쟁 시기 생산력 증가를 바탕으로 상품 화폐 경제가 발달하여
정 답
⑤ 조선 고종(흥선 대원군 집권기)에 해당한다. 농업, 수공업, 상업, 광업 등 여러 분야에서 다양한
변화가 나타났다.
1 ④ 2 ③ 3 ② 4 ① 5 ② 8. [출제의도] 고려의 문벌 지배 체제 이해하기
6 ⑤ 7 ③ 8 ② 9 ① 10 ② 제시된 자료의 ‘특정 가문의 권력 독점’, ‘이자겸의 15. [출제의도] 임진왜란 이해하기
11 ④ 12 ④ 13 ② 14 ③ 15 ④ 난’, ‘서경 천도 운동’ 등을 통해 탐구 주제는 고려 제시된 자료는 임진왜란의 주요 해전이 일어난 ‘량’에
16 ① 17 ③ 18 ⑤ 19 ① 20 ③ 문벌 지배 체제의 동요 과정임을 알 수 있다. 이자겸의 대한 탐구 활동 보고서이다. 임진왜란 때 조선 수군은
난, 묘청의 서경 천도 운동이 일어나면서 문벌 지배 방향 선회력이 뛰어나며 거친 바람과 물결에 저항
세력이 동요되는 가운데 일어난 무신 정변으로 문벌 능력이 우수한 조선 배의 장점을 활용하여, 물살이
해 설
지배 체제는 무너지게 되었다. 거센 ‘량’에서 작전을 펼쳤다. 수군과 의병의 활약으로
조선은 역전의 발판을 마련하였고, 권율은 행주산성
9. [출제의도] 고려의 대외 관계 이해하기
1. [출제의도] 청동기 시대의 생활 이해하기 전투를 승리로 이끌었다.
제시된 지도에서 ‘서희의 외교 협상’, ‘강감찬의 귀주
제시된 자료는 청동기 시대의 고인돌이다. 청동기 시대 [오답풀이] ① 병인양요 ② 몽골의 침입 ③ 병자호란
대첩’ 등을 통해 이 전쟁은 거란과의 전쟁임을 알 수 ⑤ 살수 대첩에 해당한다.
에는 사유 재산이 생기고 계급이 발생하였으며, 대표
있다. 거란의 1차 침입 때 서희는 외교 담판으로 강동
적인 유물로 비파형 동검이 있다.
6주를 획득했으며, 거란의 3차 침입 때 강감찬이 이끄는 16. [출제의도] 개화 정책 이해하기
[오답풀이] ① 통일 신라 말 ② 조선 고종(흥선 대원군
고려군은 귀주에서 크게 승리하였다. 이후 고려, 송, 제시된 자료의 ‘개화 정책을 총괄’, ‘12사’를 통해
집권기) ③ 신석기 시대 ⑤ 고구려에 해당한다.
요(거란) 사이의 세력 균형으로 평화가 유지되었고, (가) 기구는 통리기무아문임을 알 수 있다. 1880년에
2. [출제의도] 신라 이해하기 고려는 압록강 입구에서 도련포에 이르는 천리장성을 설치된 통리기무아문은 개화 정책을 총괄하여, 별기군
제시된 자료의 ‘경주’, ‘토함산’, ‘불국사’, ‘석굴암’ 등을 쌓아 외적의 침입에 대비하였다. 창설, 조사시찰단ㆍ영선사 파견을 주도하였다. 임오
통해 (가) 국가가 신라임을 알 수 있다. 신라에는 통일 [오답풀이] ② 신미양요 ③ 임진왜란 등 ④ 청일 전쟁 군란 때 폐지되었다.
이전부터 골품제라는 신분 제도가 있었는데 이 제도 의 영향에 해당한다. ⑤ 원 간섭기에 권문세족은 고려 [오답풀이] ② 조선 중종 ③, ⑤ 고려 ④ 신라에 해당
는 골품에 따라 관직 승진에 제한을 두었고, 가옥이나 지배 세력으로 등장하였다. 한다.
수레의 크기, 의복과 그릇의 사용까지도 제한하였다.
10. [출제의도] 조선 정조의 업적 파악하기 17. [출제의도] 조ㆍ미 수호 통상 조약 이해하기
[오답풀이] ① 고려, 조선 ② 부여 ④ 발해 ⑤ 고려
제시된 자료는 북한의 첫 유네스코 세계기록유산으로 제시된 자료는 조선이 청의 알선으로 미국과 조ㆍ미
에 해당한다.
등재된 무예도보통지에 대한 내용이다. 정조는 수원 수호 통상 조약을 체결한 내용이다. 서양 국가와 최초로
3. [출제의도] 고려 태조의 정책 파악하기 화성을 만들고 친위 부대인 장용영을 두었으며, 규장각을 체결한 이 조약은 거중 조정 조항과 관세 조항, 최혜국
제시된 자료의 ‘호족 세력과 혼인’, ‘왕씨 성을 하사’, 설치해 강력한 정치 기구로 육성하였다. 대우 조항 등을 담고 있다.
‘후삼국 통일’ 등을 통해 왕이 고려 태조임을 알 수 있 [오답풀이] ① 통일 신라 신문왕 ③ 조선 고종 ④ [오답풀이] ① 조선 세종 때 부산포, 제포, 염포를
다. 고려 태조는 후대 왕들에게 훈요 10조를 남겨 고 조선 성종 ⑤ 고려 예종에 해당한다. 개방하였다. ② 조ㆍ일 수호 조규(강화도 조약)에 해
려 왕조가 나아갈 방향을 제시하였다. 당한다. ④ 조선과 일본은 광해군 때 관계를 회복하고
11. [출제의도] 독립 협회의 활동 이해하기 통신사를 다시 파견하였다. ⑤ 병자호란 때의 강화조약
[오답풀이] ① 고구려 소수림왕 ③ 조선 광해군 등
④ 조선 태조 ⑤ 고려 공민왕에 해당한다. 제시된 자료의 ‘종로’, ‘의정부 관리 참여’, ‘헌의 6조’ 에 해당한다.
를 통해 (가)는 관민공동회임을 알 수 있다. 관민공
4. [출제의도] 발해의 역사 이해하기 동회를 주도했던 독립 협회는 박정양 내각과 협상을 18. [출제의도] 근대 국민 국가의 수립 과정 이해하기
제시된 자료의 ‘대조영’, ‘고구려 유민 출신’을 통해 벌인 끝에 새로운 중추원 관제를 반포하게 하는 등 의 (가)는 갑신정변, (나)는 동학 농민 운동이다. 갑신
(가) 국가는 발해임을 알 수 있다. 발해는 중앙 행정 회 설립 운동을 전개하였다. 정변에서 제기된 국왕의 전제권 제한, 인민 평등권
조직으로 당의 3성 6부제를 받아들였지만, 운영에서 [오답풀이] ① 병자호란 이후 조선에서는 청에 당한 확립 등과 동학 농민 운동에서 제기된 신분제 폐지,
독자성을 발휘하였고, 중국에서 해동성국이라 불릴 치욕을 씻고자 청을 정벌하자는 북벌론이 송시열 과부의 재가 허용 등은 갑오개혁에 반영되었다.
정도로 국력이 융성하였다. 등을 중심으로 제기되었다. ② 15세기 중반 훈구는 [오답풀이] ① 임오군란 ② 인조반정 ③ 이차돈의 순교
[오답풀이] ②, ④ 조선 ③ 통일 신라 ⑤ 고려에 해당 세조의 즉위에 공을 세우고 권력을 장악하였다. ③ ④ 위화도 회군에 해당한다.
한다. 1880년대 조선책략이 유포되고 정부가 미국과 수교 19. [출제의도] 흥선 대원군의 정책 이해하기
하려 하자, 영남의 유생들은 만인소를 올리며 반대
5. [출제의도] 삼별초의 대몽 항쟁 이해하기 제시된 자료는 흥선 대원군의 집권 시기에 이루어진
하였다. ⑤ 고려 시기 무신 정변을 일으킨 무신들은
제시된 자료는 고려 정부의 개경 환도와 몽골 침략에 경복궁 중건과 서원 철폐의 내용이다. 흥선 대원군은
초기에 중방을 중심으로 국정을 운영하였다.
대항한 삼별초의 항쟁 내용이다. 1231년 몽골이 침략 정치 기강을 바로잡기 위해 세도 가문을 약화시키고,
하자 이듬해 강화도로 천도하여 항전하던 고려 정부는 12. [출제의도] 조선 세종의 업적 이해하기 비변사를 사실상 폐지하는 등 통치 체제를 재정비
몽골과 화의한 후 1270년 개경으로 환도하였다. 이에 제시된 자료에서 ‘집현전 설치’, ‘훈민정음 반포’를 통해 하였다. 또한 군정의 폐단을 바로잡기 위해 호포제를
삼별초는 개경 환도에 반대하며 진도, 제주도로 이동 (가) 왕은 조선 세종임을 알 수 있다. 세종은 4군 6진 실시하였다.
하면서 몽골에 항전하였다. 고려 건국(918), 무신 정변 지역을 개척하고 압록강과 두만강 지역까지 영토를 [오답풀이] ② 고려 무신 집권기 ③ 고려 태조 ④ 백제
(1170), 위화도 회군(1388), 인조반정(1623), 임술 농민 넓혔다. 무령왕 ⑤ 신라 진흥왕에 해당한다.
봉기(1862), 청일 전쟁(1894)이다. [오답풀이] ① 통일 신라 신문왕 ② 신라 지증왕 ③ 조선 20. [출제의도] 대한 제국의 정책 이해하기
고종 ⑤ 고려 공민왕에 해당한다. 제시된 자료는 대한 제국의 원수부 관제로 황제 중심의
6. [출제의도] 삼국의 역사 이해하기
제시된 (가) 자료는 ‘평양으로 도읍을 옮겼다.’, ‘한성을 13. [출제의도] 조선의 신분 제도 이해하기 강력한 군대 육성의 의지를 나타내고 있다. 대한 제국은
함락’을 통해 5세기 고구려 장수왕 때의 일이며, 이후 대한국 국제를 반포하여 황제 중심의 전제 정치
제시된 자료는 조선 중기 양반의 일기로, 밑줄 친
(나) 자료는 ‘삼국이 한 집안을 이루고’를 통해 삼국 실시를 명문화하였다.
‘그의 신분’은 노비이다. 조선의 신분 제도는 법적
통일 이후의 일임을 알 수 있다. 신라는 6세기 법흥왕 [오답풀이] ① 백제 성왕 ② 조선 철종 ④ 조선 인조
으로 양인과 천인으로 구분하는 양천제이고, 점차 양반,
때 금관가야를 복속하였다. ⑤ 고려에 해당한다.
중인, 상민, 천민의 네 신분으로 정착되었다. 천인에
[오답풀이] ① 조선 연산군, 중종, 명종 등 ② 고려 속한 노비는 매매ㆍ상속ㆍ증여의 대상이 되었다.
인종 ③ 조선 순조 ④ 고려 공민왕에 해당한다. [오답풀이] ① 중인 ③ 상민 ④ 양인 ⑤ 양반에 해당
7. [출제의도] 고려 광종의 업적 파악하기 한다.

제시된 자료에서 ‘과거제’, ‘쌍기’, ‘유교적 소양을 갖춘


관리’를 통해 (가) 왕이 고려 광종임을 알 수 있다.

12
2023학년도 6월
전국연합학력평가 정답 및 해설 고2

[오답풀이] ㄱ. 플라톤과 에피쿠로스 모두 죽음을 두려움의


16. [출제의도] 동물 권리에 대한 데카르트와 싱어의
• 생활과 윤리 • 대상으로 보지 않았다. ㄴ. 플라톤은 죽음 이후에 인간의
입장 비교하기
영혼은 소멸하지 않고 이데아 세계에 들어갈 수 있다고
갑은 데카르트, 을은 싱어이다. 데카르트는 동물은 움직
주장하였다.
이는 기계이며 도덕적 권리를 가지지 않는다고 보았다.
정 답 8. [출제의도] 뇌사에 대한 윤리적 쟁점 이해하기 싱어는 동물은 쾌고감수능력을 지닌 존재이므로 도덕적
갑은 뇌사자의 장기를 다른 환자에게 이식할 수 있고, 배려 대상에 포함되어야 한다고 보았다.
1 ③ 2 ③ 3 ② 4 ③ 5 ① 인공호흡기와 같은 의료 자원을 효율적으로 이용할
17. [출제의도] 사회 윤리에 대한 니부어의 입장 이해하기
6 ⑤ 7 ⑤ 8 ④ 9 ② 10 ⑤ 수 있으므로 뇌사를 죽음의 판정 기준으로 인정해야
제시문의 사상가는 니부어이다. 니부어는 개인의 도덕성
11 ③ 12 ⑤ 13 ④ 14 ① 15 ④ 한다고 주장한다.
16 ① 17 ④ 18 ⑤ 19 ④ 20 ② 보다 개인이 속한 집단의 도덕성이 현저하게 떨어진다고
9. [출제의도] 형벌에 대한 칸트와 벤담의 입장 비교하기 보았다. 또한 니부어는 개인이 속한 집단에서 인간의
제시문의 ‘나’는 칸트이고, ‘어떤 사상가’는 벤담이다. 이기적인 성향과 집단 간 힘의 불균등한 분배로 인해
해 설 칸트는 응보주의의 관점에서 범죄자는 오직 범죄를 부정의가 발생한다고 보았다.
저질렀다는 이유만으로 처벌을 받아야 한다고 주장 [오답풀이] ㄱ. 니부어는 개인이 지향하는 최고의 도덕적
하였다. 벤담은 공리주의의 관점에서 형벌은 근본적 이상은 이타심, 사회가 지향하는 최고의 도덕적 이상은
1. [출제의도] 규범 윤리학과 메타 윤리학의 입장 이해하기
으로 악이지만 형벌이 가져올 사회 전체의 유용성이라는 정의라고 보았다. ㄷ. 니부어는 집단 간의 갈등 해결을
갑은 규범 윤리학, 을은 메타 윤리학의 입장이다. 규범
측면에서 필요하다고 주장하였다. 위해 개인의 양심과 선의지뿐만 아니라 강제적인 수단이
윤리학은 도덕적 행위의 근거가 되는 도덕 원리나
[오답풀이] ①, ③, ④, ⑤ 벤담이 간과하는 입장이 필요하다고 보았다.
인간의 성품에 관해 탐구하고, 이를 바탕으로 도덕적
아니다.
으로 가치 있는 삶의 방향 제시를 핵심 과제로 삼는다. 18. [출제의도] 사랑에 대한 프롬의 입장 파악하기
메타 윤리학은 도덕적 용어들의 개념을 분석하고, 도덕 10. [출제의도] 국가의 역할에 대한 아리스토텔레스의 제시문의 사상가는 프롬이다. 프롬은 참다운 사랑의
판단의 타당성을 입증하거나 정당화하는 것을 핵심 입장 이해하기 실천을 위해서는 보호, 책임, 존경, 이해 등이 요구된다고
과제로 삼는다. 제시문의 사상가는 아리스토텔레스이다. 아리스토텔레스는 보았다.
[오답풀이] ① 메타 윤리학의 입장이다. ②, ⑤ 기술 국가가 인간의 본성에 따라 자연적으로 형성되었으며,
19. [출제의도] 유전자 치료의 윤리적 쟁점 파악하기
윤리학의 입장이다. ④ 규범 윤리학의 입장이다. 인간은 국가 안에서 행복한 삶을 살 수 있다고 보았다.
A는 수정란이나 발생 초기의 배아에 유전 물질을 삽입
2. [출제의도] 국가의 권위에 대한 로크의 입장 파악하기 11. [출제의도] 분배적 정의에 대한 롤스와 노직의 입장 하여 질병을 치료하는 방법인 생식 세포 유전자 치료
제시문의 사상가는 로크이다. 로크는 시민들의 국가에 비교하기 이다. 유전 질환을 물려주지 않으려는 부모의 선택권을
대한 복종의 의무가 명시적 동의뿐만 아니라 묵시적 갑은 롤스, 을은 노직이다. 롤스는 공정한 분배를 위하여 보장해야 한다는 주장은 생식 세포 유전자 치료를
동의에 의해서도 발생한다고 보았다. 또한 국가가 정의의 원칙을 제시하였으며, 정의의 원칙은 자연적· 찬성하는 근거이다.
개인의 기본권을 침해했을 경우, 개인은 그 국가에 사회적 우연성이 배제된 원초적 입장에서 도출되어야
20. [출제의도] 기업의 사회적 책임에 대한 입장 파악하기
대해 저항권을 행사할 수 있다고 보았다. 정당화된다고 보았다. 노직은 개인이 정당한 절차를
신문 칼럼에서는 기업은 사회의 핵심 기관이므로 사회적
통해 얻은 소유물에 대해 절대적 소유 권리를 가진다고
3. [출제의도] 윤리적 성찰의 중요성과 방법 파악하기 가치 실현에 적극적으로 참여해야 한다고 주장하며,
보았다. 한편 롤스와 노직은 분배 절차의 공정성이
그림의 강연자는 소크라테스이다. 소크라테스는 성찰의 이는 기업의 본질적 목적인 이윤 추구에도 도움이
분배 결과의 공정성을 보장한다고 주장하였다.
중요성을 강조하면서, 개인은 각자의 무지를 깨닫고 된다고 본다.
[오답풀이] ㄹ. 노직은 취득과 이전에서의 정의의 원리와
영혼을 최상의 상태로 가꾸어야 한다고 주장하였다.
불의에 대한 교정의 원리에 따라 소유물에 대한 권리가
4. [출제의도] 도가와 불교 사상의 입장 이해하기 부여된다고 보았다.
(가)는 도가 사상, (나)는 불교 사상의 입장이다. 도가 12. [출제의도] 성에 대한 자유주의와 중도주의 입장
사상에서는 사회 혼란의 원인이 인위적인 제도와 규범에 이해하기
있으므로 자연적 본성에 따라 소박한 삶을 살아야
갑은 자유주의, 을은 중도주의 입장이다. 갑은 타인
한다고 보았다. 불교 사상에서는 세상의 모든 존재와
에게 해악을 주지 않는 범위에서 성인들의 자발적
현상이 원인과 조건으로 발생한다는 연기설을 주장
동의를 전제로 한 성적 관계가 정당화될 수 있다고
하였다.
보고, 을은 자발적 동의와 사랑을 동반한 성적 관계가
[오답풀이] ① 유교 사상의 입장이다. ② 도가 사상
정당화될 수 있다고 본다.
에서는 선과 악을 명확히 분별해야 한다고 주장하지
않았다. ④ 불교 사상에서는 고정불변의 실체가 있음을 13. [출제의도] 시민 불복종에 대한 롤스의 입장 이해하기
주장하지 않았다. ⑤ 도가 사상에서는 인의의 덕을 롤스는 시민 불복종 행위가 공유된 정의관에 근거하여
인위적인 것으로 보았다. 공개적이고 비폭력적으로 이루어져야 하며, 법체계를
존중하여 최후의 수단으로 이루어져야 한다고 주장
5. [출제의도] 칸트의 의무론적 윤리 이해하기
하였다. 한편 롤스는 시민 불복종은 개인적 도덕 원칙과
제시문의 사상가는 칸트이다. 칸트는 실천 이성이 우리
종교적 신념이 아니라 사회적 다수의 공유된 정의관에
자신에게 부과한 자율적 명령인 도덕 법칙에 따라
근거해야 한다고 보았다.
행동해야 한다고 보았다. 또한 자연적 경향성이 아니라
선의지에 따른 행위가 도덕적 가치를 지닌다고 보았다. 14. [출제의도] 인간 배아 복제에 대한 입장 파악하기
갑은 배아가 단순한 세포 조직이 아니라 인간과 마찬
6. [출제의도] 형제간의 관계 이해하기
가지로 존엄한 존재라고 주장한다. 을은 배아가 인격체는
가상 편지에서는 형제간에 지켜야 할 덕목으로 우애를
아니지만, 그 가치를 인정하면서 인간의 질병 치료를
강조하고 있으며, 부모에 대한 사랑과 수족지의의 도리를
목적으로 사용될 수 있다고 주장한다.
실천해야 한다고 주장한다. 또한 형제는 장유의 구분이
있는 관계라고 본다. 15. [출제의도] 칼뱅과 공자의 직업관 이해하기
갑은 칼뱅, 을은 공자이다. 칼뱅은 직업을 신의 부르심,
7. [출제의도] 플라톤과 에피쿠로스의 죽음관 비교하기
즉 소명이라고 보며 직업적 성공으로 인한 부의 축적을
갑은 플라톤, 을은 에피쿠로스이다. 플라톤은 죽음을
긍정하였다. 공자는 자신의 직분에 충실하는 정명을
통해 인간의 영혼이 육체로부터 벗어날 때 참된 진리를
강조하였다.
파악할 수 있으므로 죽음을 두려워할 필요가 없다고
[오답풀이] ㄱ. 칼뱅은 모든 직업이 신의 부르심이므로
보았다. 에피쿠로스는 죽음 이후에 인간은 아무것도
직업에는 귀천이 없다고 보았다. ㄷ. 공자는 각자 자신의
인식할 수 없으므로 죽음을 두려워할 필요가 없다고
직분에 충실할 것을 강조하였다.
보았다.

13
2023학년도 6월
전국연합학력평가 정답 및 해설 고2

9. [출제의도] 원효의 사상 이해하기 15. [출제의도] 정약용이 이황에게 제기할 수 있는 비판


• 윤리와 사상 • 제시문의 사상가는 원효이다. 원효는 중생이 무지에서 파악하기
벗어나 본래 마음으로 돌아가면 부처가 된다고 보아 제시문의 사상가는 정약용이다. 이황은 사단과 사덕이
중생과 부처는 둘이 아니라고 주장하였다. 또한 당시의 모두 인간에게 선천적으로 내재된 완전하고 순선한 것
정 답 다양한 불교 이론과 종파의 주장이 지닌 가치를 인정 으로 보았다. 한편, 정약용은 사덕은 인간의 본성에
하면서, 그 바탕 위에서 이들을 더 높은 차원에서 하나로 선천적으로 내재해 있는 것이 아니라, 일상생활 속에서
1 ③ 2 ④ 3 ① 4 ④ 5 ③ 아우르는 화쟁을 주장하였고 실천과 수행에는 일정한 후천적인 노력을 통해 형성되는 것이라고 보았다.
6 ⑤ 7 ① 8 ⑤ 9 ⑤ 10 ② 형식이나 방법이 없다는 무애행을 주장하였다.
16. [출제의도] 맹자의 사상 이해하기
11 ① 12 ③ 13 ② 14 ④ 15 ④ [오답풀이] 원효는 당시 경전 해석을 두고 종파 간의 차이를
16 ② 17 ⑤ 18 ② 19 ② 20 ④ 제시문의 사상가는 맹자이다. 맹자는 누구나 불인인지심
부정하고 획일화시킬 것이 아니라 다양한 입장을 인정해야
(不忍人之心)을 지니고 있어 인간의 본성이 선하다는
함을 주장하였다.
성선설(性善說)을 주장하였다. 또한 옳은 일을 꾸준히
해 설 10. [출제의도] 주희와 왕수인의 사상 이해하기 행하여 호연지기(浩然之氣)를 기르면 누구나 이상적
(가)의 갑은 주희, 을은 왕수인이다. 주희는 성은 곧 이 인간이 될 수 있다고 보았다.
[性卽理]라고 주장하며, 마음과 개별 사물들 모두에
1. [출제의도] 인간의 특성 이해하기 17. [출제의도] 중국 도교 사상 이해하기
이치가 부여되어 있다고 주장하였다. 그리고 우리의
글쓴이는 인간이 동물과 달리 성찰을 통해 가치 있는 황로학파(黃老學派)는 전설상의 제왕인 황제와 노자를
앎을 극진히 하려면 개별 사물의 이치를 탐구해야
삶을 살고자 노력하는 존재라는 것을 강조한다. 숭상하여 무위(無爲)로 다스리는 제왕의 통치술을
한다고 주장하였다. 왕수인은 마음이 곧 이치[心卽理]
주장하였다. 오두미교(五斗米敎)는 교리를 믿고 따르면
2. [출제의도] 윤리 사상과 사회사상의 특징 및 관계 이므로 마음 밖에는 이치가 없고 마음과 무관한 사물도
반드시 병이 낫는다고 주장하였다. 현학(玄學)은 세속적
이해하기 없다고 주장하였다. 한편 주희와 왕수인 모두 천리를
주제와 거리를 두고, 형이상학적이고 예술적인 논의인
그림의 A는 윤리 사상, B는 사회사상이다. 윤리 사상과 보존하고 인욕을 제거할 것과 도덕적 앎과 실천을
청담(淸談)을 주장하였다.
사회사상은 상호 의존적이며 보완적인 관계이지만 각각 일치시키기 위한 수행을 강조하였다.
독립적인 영역을 가진다. [오답풀이] ㄹ. 왕수인은 누구나 양지를 선천적으로 18. [출제의도] 동학과 원불교 사상 이해하기
지니고 있다고 보았다. 갑은 동학 사상가 최시형, 을은 원불교의 창시자 박중빈
3. [출제의도] 공자의 사상 이해하기
이다. 동학은 서학(西學)에 대응하는 입장을 견지하며
제시문의 사상가는 공자이다. 공자는 존비친소(尊卑 11. [출제의도] 맹자와 순자의 정치 사상 이해하기
모든 사람은 자기 안에 한울님을 모시고 있으므로[侍天主],
親疏)의 구별을 전제로 하며 시비선악(是非善惡)을 갑은 맹자, 을은 순자이다. 맹자는 백성을 가장 귀하게
사람이 곧 하늘[人乃天]이라 하여 모든 사람이 평등하다고
분별하여 실천하는 사랑인 인(仁)을 주장하였다. 이러한 여겨 그들이 항심(恒心)을 되찾을 수 있도록 일정한
보았다. 원불교는 일원상(一圓相)을 신앙과 수행의
인을 실천하는 방법으로서 서(恕)를 제시하였으며, 사사 생업[恒産]을 보장해야 한다고 강조하였다. 또한 그는
표본으로 삼고 정신문명과 물질문명, 정신과 육체의
로운 욕구를 극복하여 예(禮)를 회복하는 극기복례 군주가 군주답지 못하면 교체할 수 있다는 민본주의적
조화를 강조하며 종교적 수행과 사회적 실천을 통하여
(克己復禮)로 인이 실현된다고 보았다. 또한 정명(正名) 역성혁명(易姓革命)을 주장하였다. 순자는 이기적
교리를 체계화하면서 일상생활 속에서 수행할 수 있는
을 덕치(德治) 실현의 기반으로 보았으며, 통치자의 본성을 후천적 노력과 교육으로 변화시키고 예를 바탕
다양한 방법을 제시하였다. 동학과 원불교는 모두 현세
수기안인(修己安人)을 주장하였다. 으로 국가를 다스리는 예치(禮治)를 주장하였다. 두
에서 이상 사회를 실현할 수 있다고 보았다.
사상가 모두 군주는 인의(仁義)의 실현을 통치의 목적
4. [출제의도] 동양의 자연관 이해하기 으로 삼아야 한다고 보았다. 19. [출제의도] 순자의 사상 이해하기
(가)는 유교, (나)는 불교, (다)는 도가 사상이다. 동양
제시문의 사상가는 순자이다. 순자는 인간의 본성이
에서는 세계를 만물이 상호 의존적으로 살아가는 하나의 12. [출제의도] 노자의 사상 이해하기
악하기 때문에 성인이 제정한 예의를 배우고 익혀야만
유기체로 보고 공생의 자연관을 주장한다. 제시문의 사상가는 노자이다. 노자는 영토가 작고
사람들이 선하게 되고, 세상이 평화롭게 된다고 보았다.
백성의 수가 적은 공동체인 소국과민(小國寡民)을
5. [출제의도] 석가모니의 사상 이해하기 이상적인 사회로 보았다. 이 사회에서 백성들은 무지 20. [출제의도] 혜능과 지눌의 사상 이해하기
제시문의 사상가는 석가모니이다. 석가모니는 연기에 (無知)하고, 인위적인 규범이나 가치를 초월하여 소박 갑은 혜능, 을은 지눌이다. 지눌은 점수의 구체적 방법인
근거하여 모든 현상은 원인[因]과 조건[緣]에 의해 하게 살아간다. 이를 위해 노자는 무위의 다스림을 선정과 지혜를 함께 닦아나가는 정혜쌍수(定慧雙修)를
생겨나기 때문에 어떤 존재와 현상도 독립적일 수 강조하였다. 주장하였다.
없다고 보았다.
13. [출제의도] 용수와 세친의 사상 이해하기 [오답풀이] ① 혜능은 진리가 언어와 문자에 의지하지
[오답풀이] ① 무명은 진리에 통달하지 못한 마음의
않아도 마음에서 마음으로 전해진다[以心傳心]고 보았다.
상태를 말한다. ② 석가모니는 팔정도를 실천하면 괴로움의 갑은 중관 사상을 대표하는 용수, 을은 유식 사상을
② 혜능이 아니라 지눌의 입장이다. ③ 지눌은 불성이
원인인 삼독을 제거할 수 있다고 보았다. ④ 석가모니는 대표하는 세친이다. 중관 사상은 중도에 따라 양극단에
후천적으로 형성되는 것이 아니라 누구나 선천적으로
연기의 원리에 따라 만물이 생멸, 변화한다고 보았다. 빠지지 않고 관찰하는 일 즉 중관(中觀)을 강조하였다.
가지고 있는 것으로 보았다. ⑤ 선종에서는 교리 공부
⑤ 석가모니는 열반에 이르려면 극단적인 고행이 아니라 [오답풀이] ① 중관 사상에서는 모든 것은 연기에
보다는 참선을 통해 깨달음을 얻어야 함을 강조하였다.
중도의 수행법인 팔정도를 실천해야 한다고 보았다. 의해 발생하기 때문에 자성이 없으므로 고정된 실체가
없다고 보았다. ③ 유식 사상은 불변의 본질을 가진
6. [출제의도] 이항로와 신기선의 사상 이해하기 객관적 현상은 존재하지 않고 오직 그것을 경험하는
갑은 이항로, 을은 신기선이다. 이항로는 유교적 가치를 우리의 의식만이 존재한다고 주장하였다. ④ 유식 사상은
지키고 서양의 문물은 나라에 해가 되기에 모두 찾아내 본래 의식(마음)에 불변의 본질이 없다는 진실을 깨달으면
불태워야 한다는 위정척사(衛正斥邪)를 주장하였다. 해탈할 수 있다고 보았다. ⑤ 중관 사상과 유식 사상은
신기선은 유교적 가치는 지키면서 주체적으로 서양의 모두 대승 불교이므로 중생의 구제와 자비의 실천을
과학기술을 수용해야 한다는 동도서기(東道西器)를 중시하였다.
주장하였다.
14. [출제의도] 이이와 이황의 사상 이해하기
7. [출제의도] 대승 불교의 이상적 인간상 이해하기 갑은 이이, 을은 이황이다. 이이는 작용이 없는 이는
㉠은 보살이다. 보살은 위로는 진리를 구하고 아래로는 발(發)하는 까닭이고, 작용이 있는 기는 발하는 것이
중생을 구제하는 사람이다. 라고 보았다. 그래서 사단과 칠정은 모두 기가 발하고
8. [출제의도] 장자의 사상 이해하기 이가 타는 것이라고 파악하였다. 이황은 이와 기가
모두 발할 수 있어서 이도 기처럼 운동성을 지닌다고
제시문의 사상가는 장자이다. 장자는 이상적인 인간
보았다. 그래서 사단은 이가 발하고 기가 이를 따른
으로 지인(至人)을 제시하였다. 지인은 어떠한 외물
것이며, 칠정은 기가 발하고 이가 기를 탄 것이라고
에도 얽매이지 않고 자유롭게 살아감으로써 일체의
주장하였다.
분별을 없애 도달하게 되는 절대적인 정신적 자유의
[오답풀이] ㄱ. 갑과 을의 공통점이다. ㄷ. 을의 입장이다.
경지[逍遙]에 이른 존재이다.

14
2023학년도 6월
전국연합학력평가 정답 및 해설 고2

[오답풀이] ㄷ은 통곗값을 일정 단위의 점으로 찍어 에서 형성된 주상 절리가 있다. 울릉도는 전체적으로


• 한국지리 • 지리 현상의 분포를 표현한 점묘도이고, ㄹ은 지역 경사가 급한 종 모양의 화산섬으로, 섬의 중앙부에
간 이동에 대한 통곗값을 화살표의 방향과 굵기로 화구 부근이 함몰되어 형성된 칼데라 분지(나리 분지)
표현한 유선도이다. 와 칼데라 분지 내부에서 용암이 분출하여 형성된
중앙 화구구(알봉)가 있는 이중 화산체이다.
정 답 8. [출제의도] 범람원의 특성 이해하기
[오답풀이] ㄹ. 나리 분지는 알봉보다 먼저 형성되
하천의 범람에 의해 운반된 물질이 쌓여 형성된 범람
었다.
1 ④ 2 ④ 3 ⑤ 4 ② 5 ⑤ 원은 자연 제방인 (나)와 배후 습지인 (가)로 구성
6 ② 7 ① 8 ① 9 ④ 10 ⑤ 된다. 자연 제방은 배후 습지에 비해 평균 해발 고도가 16. [출제의도] 지역 조사 방법과 지리 정보 유형 이해
11 ③ 12 ④ 13 ① 14 ④ 15 ③ 높고, 퇴적 물질의 평균 입자가 크며, 홍수 시 침수 하기
16 ② 17 ⑤ 18 ③ 19 ③ 20 ① 가능성이 낮다. 수치 지도는 지리 정보를 디지털화하여 표현하기 때문에
종이 지도보다 지리 정보의 추가와 수정이 용이하다.
9. [출제의도] 독도, 마라도, 이어도의 특성 이해하기
원격 탐사는 인공위성이나 항공기 등을 이용하여 관측
해 설 (가)는 마라도, (나)는 이어도, (다)는 독도이다. 독도
대상과의 접촉 없이 먼 거리에서 대상의 정보를 수집
는 우리나라 동쪽 끝에 위치하여 마라도보다 일출
하는 데 유리하다.
시각이 이르다.
1. [출제의도] 우리나라의 위치 특성 이해하기 [오답풀이] ㄴ. 주소는 지리적 현상의 위치를 나타
[오답풀이] ① 마라도는 영해 설정에 통상 기선을
우리나라는 북반구 중위도에 위치하여 사계절이 뚜렷 내는 것으로 공간 정보에 해당한다. ㄹ. 현지 방문
적용한다. ② 수중 암초인 이어도는 우리나라의 배타적
하고 냉ㆍ온대 기후가 나타나며, 동경 135°를 표준 및 면담은 지역 조사 단계 중 야외 조사에 해당한다.
경제 수역에 포함되나 영역에는 해당하지 않는다.
경선으로 사용하여 본초자오선이 지나는 영국보다 17. [출제의도] 우리나라 하천의 특성 이해하기
③ 독도는 행정구역상 경상북도에 속한다. ⑤ 한ㆍ일
표준시가 9시간 빠르다. 우리나라는 유라시아 대륙의
중간 수역은 어업 협정을 통해 양국이 공동으로 어족 낙동강은 강원도 태백시 함백산에서 발원하여 영남
동안에 위치하여 계절풍의 영향을 받고, 기온의 연교차
자원을 보존ㆍ관리하는 수역으로 마라도와 이어도는 지방의 중앙 저지대를 통해 남해로 유입하는 강으로
가 큰 대륙성 기후가 나타난다.
이에 포함되지 않는다. 남한에서 제일 길다.
2. [출제의도] 기후 요소와 기후 요인 이해하기 [오답풀이] A는 한강, B는 금강, C는 영산강, D는
10. [출제의도] 지역별 기후 특징 이해하기
비슷한 위도의 수원과 비교하여 태백의 8월 평균 기온 섬진강이다.
A는 삼지연, B는 서울, C는 울릉도, D는 대구, E는
이 낮은 이유는 해발 고도가 높기 때문이다. 광주와
제주이다. 18. [출제의도] 높새바람의 특성 이해하기
수원의 기온 차이는 위도로 인한 것이다.
[오답풀이] 서울, 대구에는 신생대 화산 지형이 분포 높새바람은 주로 늦봄에서 초여름 사이에 부는 북동풍
3. [출제의도] 산지의 특성 이해하기 하지 않는다. A ~ E 중에서 기온의 연교차가 가장 으로, 푄 현상에 의해 영서ㆍ경기 지방에 영향을
지리산은 시ㆍ원생대의 변성암이 오랜 시간에 걸쳐 큰 지역은 삼지연이고, 겨울 강수 집중률이 가장 높은 미치는 고온 건조한 바람이다. 이로 인해 이상 고온
풍화와 침식을 받아 형성된 토양층이 두꺼운 흙산이다. 지역은 울릉도이다. 현상이 나타나고 가뭄으로 인한 피해가 발생한다.
설악산은 중생대에 땅 속에서 형성된 화강암이 오랫 [오답풀이] ㄱ. 꽃샘추위는 이른 봄 시베리아 고기압
11. [출제의도] 카르스트 지형의 특성 이해하기
동안 침식 작용을 받아 지표면에 드러나면서 형성된 의 일시적 확장으로 나타난다. ㄹ. 황사는 봄철에
카르스트 지형은 기반암인 석회암이 빗물이나 지하수
돌산이다. 중국 내륙이나 몽골의 건조 지역에서 발생한 흙먼지
에 용식되어 형성된 지형이다. 돌리네는 기반암이
[오답풀이] 지리산은 소백산맥에 위치한다. 가 편서풍을 타고 날아오는 현상이다.
용식을 받아 움푹 파인 깔대기 모양의 지형으로 배수
4. [출제의도] 조선 시대 지리지의 특징 이해하기 가 양호하여 주로 밭으로 이용된다. 석회암 풍화토는 19. [출제의도] 지리 정보 체계를 활용한 최적 입지 분
(가)는 조선 후기에 제작된 택리지로, 저자의 주관적 기반암이 용식된 후 철분 등이 산화되어 붉은색을 석하기
견해가 많이 반영된 사찬 지리지이다. (나)는 조선 띤다. 어린이 박물관의 최적 입지로 선정된 A 지역은 지표면
전기에 제작된 신증동국여지승람으로, 국가 통치의 경사도는 5° 미만, 지가 3백만 원/ km 미만, 인구
12. [출제의도] 해안 지형의 특성 이해하기
기초 자료를 확보하기 위해 제작된 관찬 지리지이며, 밀도 백 명/ km 미만인 지역이다. 또한 기존 박물관
올레길에서 바라본 시 스택은 파랑의 침식 작용으로
백과사전식으로 서술되었다. 에서 1 km 이상, 가장 가까운 도로로부터 1 km 이내에
주변부가 제거되고 남은 돌기둥 혹은 바위섬이다.
[오답풀이] ⑤ ㉠은 가거지의 조건 중 지리(地理)에 위치한다.
용머리 해안의 파식대는 파랑의 침식 작용으로 형성된
해당한다.
평탄한 지형이다. 20. [출제의도] 해안 지형의 특성 이해하기
5. [출제의도] 계절의 특성 이해하기 해안 단구는 과거의 파식대나 해안 퇴적 지형이 지반
13. [출제의도] 침식 분지의 특성 이해하기
장마, 소나기, 폭염이 나타나는 (가) 계절은 여름, 삼한 의 융기 또는 해수면 하강으로 형성된 계단 모양의
침식 분지는 주위가 산지로 둘러싸인 평지로 암석의
사온, 폭설, 한파가 나타나는 (나) 계절은 겨울이다. 지형이다. 단구면에서는 과거 바닷가에 퇴적되어 있던
차별적인 풍화와 침식 작용으로 형성된다. 변성암이
시베리아 기단의 영향이 큰 계절은 겨울이다. 모래나 둥근 자갈을 볼 수 있다.
기반암을 이루는 곳에 화강암이 관입한 지역이나 하천
[오답풀이] ㄷ. 석호는 후빙기 해수면 상승으로 형성
6. [출제의도] 대동여지도의 특징 이해하기 중ㆍ상류의 하천이 합류하는 지점에서 주로 발달한다.
된 만의 입구에 사주가 발달하여 형성된 호수로, 시간
대동여지도는 김정호가 조선 후기(1861년)에 제작한 [오답풀이] ㉢ A의 주된 기반암은 중생대에 마그마
이 지남에 따라 하천에 의한 토사의 유입 등으로 인해
지도로, 지도표를 활용하여 각종 지리 정보를 표현 관입으로 형성된 화강암이고, B의 주된 기반암은
규모가 축소된다. ㄹ. 사빈은 하천 또는 주변의 암석
하였다. 도로에는 10리마다 방점을 찍어 대략적인 시ㆍ원생대에 형성된 변성암이다. ㉣ 등고선 간격이
해안으로부터 공급되어 온 모래가 파랑 및 연안류의
거리 파악이 가능하고, 하천은 쌍선(배가 다닐 수 넓은 A는 완만한 평지, 등고선 간격이 좁은 B는 경사
퇴적 작용을 받아 형성되었으며, 주로 해수욕장으로
있는 하천)과 단선(배가 다닐 수 없는 하천)으로 구분 가 급한 산지이다.
이용된다.
하였다. A는 쌍선으로 표현되어 있어 선박의 운항이 14. [출제의도] 한반도의 지체 구조 이해하기
가능하며, C는 봉수로 불이나 연기를 피워 연락을
A는 신생대 제3기에 형성된 두만 지괴, 길주ㆍ명천
하는 통신 시설이다.
지괴, B는 시ㆍ원생대의 평북ㆍ개마 지괴, 경기 지괴,
[오답풀이] ㄴ. B에서 D까지의 최단 거리는 약 50리
영남 지괴, C는 고생대에 형성된 평남 분지, 옥천
이다. ㄹ. 대동여지도에서 산줄기는 굵은 선으로 연결
습곡대, D는 경상 분지, E는 신생대 화산 활동으로
하여 표현하였으나, 정확한 해발 고도는 알 수 없다.
형성된 제주도이다. 중생대의 경상 분지는 넓은 호수
7. [출제의도] 통계 지도의 특징 이해하기 (또는 습지)로 이곳에 오랜 시간 퇴적물이 두껍게
ㄱ. 단계구분도는 통곗값을 몇 단계로 구분하고 음영 쌓이면서 경상 누층군이 형성되었다. 이 퇴적층의
이나 패턴을 달리하여 표현한 지도로, (가) 시ㆍ도별 일부 지역에서는 공룡 발자국 화석이 다수 발견된다.
경지 이용률을 표현하기에 적합하다. ㄴ. 등치선도는 15. [출제의도] 화산 지형의 특성 이해하기
같은 통곗값을 갖는 지점을 선으로 연결하여 표현한
한탄강 양안에는 유동성이 큰 용암의 열하 분출로
지도로, (나) 지역별 개나리꽃 개화일을 표현하기에
형성된 용암 대지와 용암이 냉각 및 수축되는 과정
적합하다.

15
2023학년도 6월
전국연합학력평가 정답 및 해설 고2

건조하고, 겨울철에 편서풍의 영향을 받아 온난 습윤 와 프톨레마이오스의 세계지도이다. 혼일강리역대국도


• 세계지리 • 하다. 이러한 기후 특성에 의해 포도와 올리브와 같은 지도는 경위선의 개념은 사용되지 않았으며, 중국에서
수목 농업이 발달하였다. ①은 지중해성 기후, ②는 들여온 여러 지도와 우리나라 지도를 합치고 일본을
냉대 습윤 기후, ③은 사막 기후, ④는 열대 몬순 첨부하여 그린 세계지도이다. 프톨레마이오스의 세계
기후, ⑤는 사바나 기후의 그래프이다. 지도는 150년 경 로마 시대에 제작된 세계지도로
정 답
지구를 구체(球體)로 인식하여 경위선의 개념을 사용
7. [출제의도] 지리 정보 시스템을 이용한 최적 입지
하였다. 하지만 당시에는 신대륙에 대한 지리적 지식이
1 ③ 2 ② 3 ② 4 ⑤ 5 ③ 선정하기
없었기 때문에 아메리카 대륙은 표현되어 있지 않다.
6 ① 7 ③ 8 ③ 9 ④ 10 ② 코로나 백신 연구소를 설립하기에 적합한 후보지 다섯
11 ④ 12 ⑤ 13 ④ 14 ③ 15 ① 개 국가를 조건별로 살펴보면 다음과 같다. 15. [출제의도] 주요 건조 지형의 특징 파악하기
16 ⑤ 17 ② 18 ② 19 ⑤ 20 ① 코로나 백신 접종 인구 백만 명당 정부 대응 엄격성 (가)는 버섯바위에 대한 설명으로 A이고 (나)는
평가항목
국가 완료 인구 비율 사망자 수 지수
선상지에 대한 설명으로 B이고, (다)는 메사에 대한
① 칠레 적합 적합 부적합
설명으로 C이다.
해 설 ② 브라질 부적합 부적합 부적합
③ 우루과이 적합 적합 적합 16. [출제의도] 열대 고산 기후와 사바나 기후 지역의
④ 에콰도르 적합 적합 부적합
분포 파악하기
1. [출제의도] 지역화 전략 이해하기 ⑤ 아르헨티나 적합 부적합 적합
(가)는 1년 내내 봄과 같은 기온이 나타나는 열대
㉠ ‘I amsterdam’은 암스테르담의 지역 브랜드화, ㉡ 따라서 코로나 백신 연구소를 설립하기에 가장 적합한 고산 기후이고, (나)는 사바나 기후이다. 열대 고산
‘Münchner Bier’는 뮌헨 맥주의 지리적 표시제 사례 국가는 ‘우루과이’이다. 기후 지역에서 위도가 같은 사바나 기후 지역으로
이다. 지역 브랜드화는 지역이나 지역의 상품을 특별한
8. [출제의도] 화산 및 카르스트 지형의 특성 이해하기 이주한 것은 지도의 E이다.
브랜드로 인식시켜 지역 가치를 높이고 지역의 경쟁력
을 강화하는 전략이고, 지리적 표시제는 특정 지역의 (가)는 간헐천으로 지열에 의해 데워진 지하수가 17. [출제의도] 사막 기후 지역의 분포 파악하기
지리적 특성을 반영한 우수 상품이 그 지역에서 생산 주기적으로 분출하는 지형이다. (나)는 석회 동굴로 (가)는 대륙 서안에 분포하며 1년 내내 한류의 영향
ㆍ가공되었음을 증명하고 표시하는 제도이다. 내부에서 종유석, 석순, 석주 등을 관찰할 수 있다. 을 받아 형성된 나미브 사막(A)이다. (나)는 바다와
[오답풀이] ㄱ. 간헐천은 판의 경계 지역에 잘 나타 멀리 떨어져 수분을 공급받지 못해 형성된 타클라마칸
2. [출제의도] 세계 대지형의 특성 파악하기 난다. ㄹ. (가)는 화산 활동과 관련한 지형이고, (나)는 사막(C)이다. B는 아열대 고압대의 영향을 받아 형성
광산이 세워진 우랄산맥, 애팔래치아산맥, 그레이트 지하수의 용식 작용과 관련한 지형이다. 된 룹알할리 사막이다.
디바이딩산맥은 모두 고기 조산대에 위치한다. 고기
조산대는 고생대에 조산 운동으로 만들어졌으며, 석탄 9. [출제의도] 이슬람교의 특징 이해하기 18. [출제의도] 주요 해안 지형의 특성 이해하기
의 매장량이 풍부하다. 율법에 따라 할랄 인증이 된 음식만 허용된 종교는 시 스택은 해식애가 파랑의 침식에 의해 육지로 후퇴
[오답풀이] ㄴ. 판의 경계에 위치하여 지진과 화산 이슬람교이다. 이슬람교는 하나의 신만을 인정하는 할 때 단단한 암석 부분(경암층)이 남아 형성된
활동이 빈번한 지역은 신기 조산대이다. ㄹ. 고기 조산대 유일신교이며, 신자들은 율법에 따라 하루에 다섯 바위섬이나 돌기둥으로 파랑의 침식 작용이 활발한
에 발달한 산맥은 고생대에 형성된 이후 오랜 시간 번씩 성지(메카)를 향해 기도한다. 곶(串)에서 주로 발달한다.
풍화와 침식을 받아 신기 조산대에 발달한 산맥에 비해 [오답풀이] 갑. 이슬람교는 보편종교에 해당한다. 병.
살생을 금하며 윤회 사상을 중시하는 종교는 불교이다. 19. [출제의도] 온대 서안 기후의 특성 파악하기
해발고도가 낮고 경사가 완만하다.
퀴즈 쇼의 정답자인 ‘갑’이 고른 파리, 밴쿠버는 모두
3. [출제의도] 스텝 기후와 툰드라 기후 지역의 특성 10. [출제의도] 냉대 겨울 건조 기후와 빙설 기후의 분포 서안 해양성 기후로 중위도 대륙 서안에 위치한다.
이해하기 파악하기 따라서 바다에서 불어오는 편서풍의 영향으로 해양성
(가)의 게르(ger)는 스텝 기후 지역, (나)의 춤(chum) (가)는 최한월 평균 기온 –3℃ 미만이고, 최난월 평균 기후가 나타나 연중 강수량이 고르다.
은 툰드라 기후 지역의 전통 가옥이다. 스텝 기후 지역 기온이 10℃ 이상인 냉대 겨울 건조 기후가 나타나는 [오답풀이] ① 여름에 열대 저기압의 영향을 주로
은 짧은 우기에 키가 작은 풀이 자라 넓은 초원을 이르쿠츠크의 기후 그래프이다. (나)는 최난월 평균 받는 지역은 대륙 동안의 온대 기후 지역이다.
이룬다. 툰드라 기후가 주로 나타나는 고위도 지역에 기온 0℃ 미만인 빙설 기후가 나타나는 맥머도의 기후 ②, ③ 개방적인 구조의 고상 가옥이나 카사바 등을
서는 오로라를 볼 수 있다. 그래프이다. 지도에 표시된 A는 냉대 겨울 건조 기후가 재배하는 이동식 화전 농업은 열대 기후에서 나타나
[오답풀이] ① 스텝 기후는 연 강수량보다 연 증발량 나타나는 이르쿠츠크, B는 서안 해양성 기후가 나타 는 특징이다. ④ 서안 해양성 기후 지역은 비슷한 위도
이 많은 건조 기후이다. ③ 툰드라 기후 지역에서는 나는 웰링턴, C는 빙설 기후가 나타나는 맥머도이다. 의 다른 지역에 비해 여름이 서늘하고 겨울이 온화
짧은 여름을 제외하고 월평균 기온이 0℃ 미만으로 11. [출제의도] 냉․한대 기후 지역의 지형 특성 이해하기 하여 기온의 연교차가 작은 편이다.
벼농사가 이루어지기 어렵다. ④ 툰드라 기후 지역이
④ ⓐ 빙식곡이 바닷물에 잠기면 피오르 해안이 발달 20. [출제의도] 세계의 주요 종교 경관의 특성 이해하기
스텝 기후 지역보다 주로 고위도에 위치한다. ⑤
한다. 리아스 해안은 하천에 의해 침식된 V자곡이 자료에 제시된 ‘예수’와 ‘성당’을 통해 ‘이 종교’가
(가), (나) 모두 최난월 평균 기온이 0℃ 이상이다.
침수되어 만들어진 해안이다. 크리스트교임을 알 수 있다. 크리스트교의 대표적인
4. [출제의도] 갯벌 해안의 특성 이해하기 종교 경관은 십자가와 종탑이다.
12. [출제의도] 열대 우림 기후의 특성 파악하기
조수 간만의 차가 큰 해안에서 점토, 모래 등의 물질 [오답풀이] ②는 불교, ③은 이슬람교, ④, ⑤는 힌두교
지도에 표시된 아프리카 콩고 분지, 동남아시아의
이 조류에 의해 퇴적되어 형성된 지형은 갯벌이다. 와 관련이 깊은 종교 경관이다.
적도 부근, 남아메리카의 아마존 분지 등의 지역에서는
[오답풀이] ㄱ. 산호충의 유해가 퇴적되어 만들어진
열대 우림 기후가 나타난다.
지형은 산호초 해안으로 수온이 높고 수심이 얕은
[오답풀이] ㄱ. 열대 기후 지역은 연중 기온이 높아
도서 및 연안 지역에 발달한다. ㄴ. 곶(串)에서는 파랑
기온의 일교차보다 기온의 연교차가 작다. ㄴ. 회백색의
의 에너지가 집중되어 침식 지형이 발달한다.
포드졸 토양은 냉대 기후, 붉은색의 라테라이트 토양은
5. [출제의도] 대기 대순환의 특성 이해하기 열대 기후 지역에 주로 분포한다.
A는 극 고압대, B는 고위도 저압대, C는 아열대 고압 13. [출제의도] 판의 경계 유형 분포 파악하기
대가 형성되는 위도 30° 부근, D는 적도 수렴대이다.
B는 두 개의 판이 어긋나서 미끄러지는 경계 유형으로
B의 60° 부근에서는 편서풍과 극동풍이 만나 한대
사례 지역은 샌안드레아스 단층이다. D는 해양판과
전선이 형성되어 전선성 강수가 자주 내린다.
대륙판이 서로 만나는 경계 유형으로 사례 지역은
[오답풀이] A에서는 연중 극동풍이 탁월하게 분다.
안데스산맥이다.
D는 북동 무역풍과 남동 무역풍이 만나 적도 수렴대가
[오답풀이] A는 대륙판이 갈라지는 경계 유형으로 사례
형성된다.
지역은 동아프리카 지구대이다. C는 두 대륙판이 충돌
6. [출제의도] 지중해성 기후 지역의 기후 특성 파악하기 하는 경계 유형으로 사례 지역은 히말라야산맥이다.
자료의 이 지역은 지중해성 기후 지역이다. 지중해성 14. [출제의도] 세계 고지도의 특징 파악하기
기후는 여름철에 아열대 고압대의 영향을 받아 고온
자료에 제시된 세계지도는 각각 혼일강리역대국도지도

16
2023학년도 6월
전국연합학력평가 정답 및 해설 고2

흉노, 선비 등 5호가 화북 지역에서 세력을 확대하자, 14. [출제의도] 흉노 제국 파악하기


• 동아시아사 • 진(晉)의 귀족과 한족 집단은 창장강 이남으로 이주 (가) 제국은 흉노이다. 흉노의 최고 통치자를 선우라
하여 동진(東晉)을 건국하였다. 하며, 그 아래에 좌현왕과 우현왕 등을 두었다. 흉노
는 진ㆍ한 대에 몽골 고원을 중심으로 초원 지대를
8. [출제의도] 명의 대외 관계 이해하기
장악하여 강력한 세력을 형성하였다. 특히 묵특 선우
정 답 (가) 국가는 명이다. 명은 건국 후 주변국과 조공ㆍ
시기에는 한 고조를 굴복시킬 정도로 강성하였다.
책봉 관계를 맺고 동아시아 국제 질서를 주도하였다.
[오답풀이] ① 수, 당 등 ② 고조선 ③ 진시황제 ⑤ 일
1 ① 2 ① 3 ⑤ 4 ③ 5 ④ 하지만 15세기 중반 북쪽의 오이라트부가 남하하여
본 열도에서는 야요이 시대에 정치 집단이 등장하였다.
6 ⑤ 7 ③ 8 ⑤ 9 ② 10 ① 명의 황제를 포로로 잡았고(토목보의 변), 16세기 중반
11 ② 12 ③ 13 ② 14 ④ 15 ④ 에는 타타르부가 만리장성을 넘어 수도인 베이징을 15. [출제의도] 몽골 침입과 일본의 변화 이해하기
16 ④ 17 ① 18 ④ 19 ② 20 ③ 포위하기도 하였다. 또한 이 무렵에는 왜구가 중국의 「몽고습래회사」는 몽골 및 고려의 2차 일본 원정군
동남 해안 지방을 자주 침입하였다. 15세기 초 명과 과 가마쿠라 막부 사이의 전투(1281)를 묘사한 그림
조공ㆍ책봉 관계를 맺고 감합 무역을 추진하던 일본의 이다. 가마쿠라 막부는 두 차례에 걸친 몽골의 침략
해 설 무로마치 막부의 힘이 약해지면서 왜구의 활동이 다시 을 격퇴하였으나 이후 쇠퇴의 길로 접어들었다. 전쟁
활발해졌다. 이들은 동중국해와 동남아시아 지역까지 과 혼란으로 궁핍해진 무사들은 막부에 불만을 품고
영역을 확장하여 약탈을 일삼고 밀무역에 간여하기도 천황의 명에 따라 가마쿠라 막부를 무너뜨렸다. 이후
1. [출제의도] 만주와 한반도 지역의 청동기 문화 이해하기
하였다. 이렇게 명은 북쪽의 몽골과 동남쪽의 왜구, 천황 권력에 반발한 무사들의 지지를 받은 아시카가
(가) 유물은 비파형 동검이다. 비파형 동검은 고인돌
이른바 ‘북로남왜’에 시달리며 큰 어려움을 겪었다. 다카우지가 교토에 무로마치 막부를 세웠으나
등과 함께 만주와 한반도 지역 청동기 문화를 대표
[오답풀이] ① 조선 ③ 후한 ④ 7 ~ 9세기 일본이다. (1336), 곧이어 천황이 둘로 나뉘어 대립하는 남북조
하는 유물이다.
의 분열기가 14세기 말까지 계속되었다.
[오답풀이] ② 중국 상 왕조 시기의 네 발 달린 청동 ② 화번공주는 한이나 수ㆍ당과 같은 중원 왕조가
[오답풀이] 한 멸망은 220년, 당 건국은 618년, 발해
솥 ③ 한반도 신석기 시대 유물인 빗살무늬 토기 ④ 평화를 위해 북방 민족 등 이민족 군주에게 출가시킨
멸망은 926년, 가마쿠라 막부 수립은 1185년, 조선
공주(황족의 딸)를 일컫는다. 한은 왕소군을 흉노,
일본 열도의 신석기 시대 유물인 조몬 토기 ⑤ 일본 건국은 1392년, 명 멸망은 1644년이다.
수는 의성 공주를 돌궐, 당은 문성 공주를 토번, 함안
열도의 야요이 시대 유물인 동탁이다.
공주를 위구르로 출가시켰다. 16. [출제의도] 일본 율령 체제의 특징 이해하기
2. [출제의도] 허무두 문화 파악하기 ‘태정관’, ‘영의해’, ‘요로령(養老令)’, ‘다이호 율령’을
9. [출제의도] 병자호란의 영향 파악하기
자료는 창장강 하류 지역에서 발달한 신석기 문화인 통해 자료의 율령을 반포한 국가는 일본이라는 것을
‘남한산성 전투’, ‘인조의 항복’, ‘삼전도의 홍타이지
허무두 문화에 대한 것이다. 허무두 문화의 대표적인 알 수 있다. 일본은 중앙 관제를 2관 8성으로 운영하
송덕비 건립’ 등을 볼 때 (가) 전쟁은 병자호란(1636
유물에는 돼지 무늬 토기, 뼈보습 농기구 등이 있다. 였다.
~ 1637)이다. 조선의 인조는 남한산성에서 청에 항전
[오답풀이] (나) 황허강 하류 (다) 랴오허강 유역 [오답풀이] ① 고려 ② 진 ③ 중국의 춘추 전국 시대
하였으나, 결국 청의 홍타이지에게 항복 의식을 치렀다.
(라) 한반도 (마) 일본 열도이다. ⑤ 몽골이다.
이후 조선에서는 명과의 의리를 지키고 청에 당한
3. [출제의도] 상 왕조 이해하기 치욕을 씻자는 북벌 운동이 추진되었는데, 효종 때 17. [출제의도] 몽골 제국 파악하기
밑줄 친 ‘이 나라’는 중국의 상 왕조이다. 상 왕조의 가장 왕성하게 전개되었다. 밑줄 친 ‘이 나라’는 몽골 제국이다. 몽골 제국은 도로
수도였던 은의 유적지에서 다량의 갑골문이 발굴되 [오답풀이] ① 936년 ③ 4세기경 ④ 13세기 후반 망을 정비하고 일정한 간격으로 역참을 설치하였다.
었는데, 갑골문은 상의 왕들이 제사장을 겸하며 종교 ⑤ 정강의 변(1126, 1127)이다. 또한 주요 항구에 시박사를 설치하고, 교초를 발행하여
적 권위에 의지하여 국가를 통치하였음을 보여준다. 제국 전역의 경제적 통합을 추진하였다. 쿠빌라이 칸은
10. [출제의도] 동아시아 불교의 특징 파악하기 수도를 카라코룸에서 대도(베이징)로 옮기고 국호를
[오답풀이] ① 원 ② 일본의 막부 ③ 명 ④ 발해이다.
동아시아 각국은 군주권을 강화하고 사회를 안정시 원으로 정하였다.
4. [출제의도] 조공ㆍ책봉 관계의 변화 이해하기 키기 위한 이데올로기로 불교를 수용하였기 때문에 [오답풀이] ② 한 ③ 고려 ④ 신라 ⑤ 일본 야마토 정권
조공과 책봉은 주 왕실과 제후 간의 국내적 관계에서 호국 불교의 성격을 강하게 지녔다. 북위의 윈강 석굴 (645)이다.
시작하였다. 한(漢) 이후에는 화이관에 입각하여 사원 대불, 일본의 도다이사, 신라의 황룡사 9층 목탑
18. [출제의도] 전쟁을 통한 문물의 교류 이해하기
주변국과의 외교 관계에 확대 적용되었다. 하지만 한 등이 대표적인 사례이다.
정유재란 당시 일본에 포로로 끌려간 강항은 후지와라
멸망 이후 동아시아 각국이 자신에게 유리한 국제 정세
11. [출제의도] 거란(요) 파악하기 세이카 등과 교류하면서 일본 성리학 발전에 영향을
를 만들기 위한 다원적 실리 외교, 각국 간의 우호와
(가) 국가는 거란(요)이다. 고려의 서희는 요와 외교 주었다. 병자호란 이후 청에 끌려갔던 소현세자는 독일인
상호 견제를 위한 장치로 사용하였다. 다원적 실리
담판을 벌여 강동 6주를 확보하였다. 요는 거란족 등 신부 아담 샬 등과 교류하였고 서양 서적을 가져왔다.
외교의 대표적인 사례로는 고구려와 남북조 간의 외교,
유목민은 북면관제, 한족 등 농경민은 남면관제로 이는 임진왜란(정유재란)과 병자호란이라는 전쟁을 통해
북주ㆍ북제와 돌궐 간의 외교 등이 있다.
분리하여 통치하는 이중 지배 체제를 시행하였다. 인적 교류와 문화 전파가 이루어진 사례라고 할 수
5. [출제의도] 명 왕조 파악하기 [오답풀이] ① 한 ③ 몽골 ④ 당 ⑤ 송이다. 있다.
밑줄 친 ‘항해’를 추진한 왕조는 명이다. 명의 영락제 19. [출제의도] 성리학 이해하기
12. [출제의도] 당 시기 동아시아 상황 이해하기
는 자금성을 건설하고 수도를 베이징으로 옮겼으며, (가) 사상은 남송의 주희가 집대성한 성리학이다. 주희
자료에 ‘수도 장안’, ‘승려 혜초, 현장’ 등을 통해
정화의 항해를 통해 명 중심의 조공 질서를 확대하 는 거경궁리, 격물치지, 성즉리 등을 강조하였고, 오경
(가) 국가는 당임을 알 수 있다. 선유사는 신라 승려
였다. 또한, 일본 무로마치 막부와 조공ㆍ책봉 관계 보다 사서를 중시하여 사서에 주석을 붙인 집주를
혜초가 인도를 순례한 후 기우제를 지낸 곳으로, 혜초
를 맺고 감합 무역을 전개하였다. 저술하였다. 고려 말 등장한 신진 사대부는 성리학을
를 기리는 기념비가 남아있다. 대안탑은 당의 승려
[오답풀이] ① 몽골 ② 당 ③ 여진(금) ⑤ 고려이다. 사상적 기반으로 삼아 사회 개혁을 추진하였다.
현장이 인도에서 가져온 불경과 불상을 보관하기 위해
[오답풀이] ㄴ. 불교 ㄹ. 법가 사상에 해당한다.
6. [출제의도] 한 무제 이해하기 조성되었다. 9세기경 신라의 장보고는 완도에 청해진을
‘동중서의 건의로 유교를 통치 이념으로 채택’, ‘남비 설치하고 당 – 신라 – 일본 사이의 해상 무역을 주도하 20. [출제의도] 임진왜란의 영향 파악하기
엣 정복, 군현 설치’ 등의 내용을 통해 (가) 황제는 였다. 밑줄 친 ‘전쟁’은 임진왜란이다. 일본의 침략으로 시작
한 무제임을 알 수 있다. 이 시기에 한은 흉노를 견 [오답풀이] ① 양명학은 명 대 왕수인에 의해 집대성 된 전쟁 초반에는 조선이 열세를 면치 못했으나, 수군
제하기 위해 장건을 서역의 (대)월지에 파견하였고, 되었다. ② 고려 ④ 3세기경 위는 야마타이국의 조공을 과 의병, 조ㆍ명 연합군의 활약으로 조선은 전세를
고조선과 남비엣을 정복하고 군현을 설치하였다. 받고 히미코에게 친위왜왕의 칭호를 주었다. ⑤ 임진 역전시켰다. 도요토미 히데요시가 사망하면서 일본군
[오답풀이] ① 남송 고종 ② 명 만력제 ③ 일본 왜란 이후이다. 은 철수하였다. 임진왜란의 영향으로 명은 재정이
겐메이 천황 ④ 조선 세종이다. 악화되었으며, 조선은 인구가 크게 줄고 국토가 황폐화
13. [출제의도] 북위 이해하기
되었다. 일본에서는 포로로 끌려간 조선인 기술자들
7. [출제의도] 인구 이동에 따른 국가의 수립 이해하기 (가) 국가는 북위이다. 5호 16국 시대의 분열을 수습
을 통해 도자기 기술이 발달하였다. 또한 도쿠가와
인구 이동은 문화의 전파와 교류를 촉진할 뿐만 아 한 북위는 효문제 시기 뤄양으로 천도하고 한족의
이에야스에 의해 에도 막부가 수립되었다. 한편 전란
니라 새로운 정권이나 국가를 성립시키기도 하였다. 언어와 풍습을 적극적으로 받아들이는 한화 정책을
으로 조선과 명의 국력이 약화된 틈을 타 누르하치가
기원전 1세기경 고구려에서 남하한 비류와 온조 실시하였다. 여진족을 통일하고 후금을 건국하였다. ③ 전연의
세력이 백제를 건국하였다. 4세기 초 북방 민족인 [오답풀이] ① 몽골 ③ 수 ④ 왜 ⑤ 통일 신라이다. 맹약은 1004년 송과 요 사이에 체결된 것이다.

17
2023학년도 6월
전국연합학력평가 정답 및 해설 고2

전매제를 시행하여 재정을 확충하고자 하였다. 화되면서 제철, 자기, 칠기, 견직업 등 수공업이 크게
• 세계사 • [오답풀이] ①, ④ 당 ③ 송 ⑤ 명에 해당한다. 발달하였으며 상업 또한 비약적으로 발달하였다. 송
대에는 도시의 상공업자가 행ㆍ작과 같은 동업 조합
7. [출제의도] 당 파악하기
을 결성하였으며, 교자 등의 지폐가 사용되었다.
(가) 왕조는 당이다. 8세기 안사의 난을 계기로 당은
[오답풀이] ②, ③ 청 ④ 거란(요) ⑤ 당에 해당한다.
정 답 급격히 약화되었고, 지방의 절도사들이 독자적인
지배력을 강화하였다. 대다수 농민이 소작농으로 14. [출제의도] 명 파악하기
1 ① 2 ⑤ 3 ④ 4 ③ 5 ⑤ 몰락하자 당은 재정난을 해결하기 위해 조용조 대신 (가) 왕조는 명이다. 명을 건국한 주원장은 황제권을
6 ② 7 ② 8 ④ 9 ① 10 ③ 양세법을 시행하였다. 양세법은 가구별로 자산에 따라 강화하기 위해 재상제를 폐지하고 6부를 황제의 직속
11 ③ 12 ③ 13 ① 14 ② 15 ⑤ 차등을 두어 여름과 가을에 세금을 징수하는 제도이다. 에 두어 국정을 운영하였다. 또한 백성들에게 유교
16 ③ 17 ④ 18 ④ 19 ② 20 ④ 이념을 보급하고 교화하기 위해 육유를 반포하였다.
8. [출제의도] 서하 파악하기
그리고 토지 대장(어린도책)과 호적 대장(부역황책)을
(가) 국가는 서하이다. 11세기 탕구트족이 건국한
마련하고 이를 바탕으로 세금과 요역을 징수하였으며,
해 설 서하는 동서 무역으로 번영하였으며, 고유 문자를
관리의 수탈을 줄이고자 농민이 직접 조세 징수와
제정하여 사용하였다. 또한, 송과 대립하기도 하였으나
치안 유지를 담당하는 이갑제를 시행하였다.
평화 조약을 체결하여 송으로부터 많은 물자를 제공
1. [출제의도] 이집트 문명 파악하기 [오답풀이] ①, ③ 청 ④ 위진 남북조 시대 ⑤ 진
받았다.
(가) 문명은 이집트 문명이다. 이집트 문명은 기원전 (秦)에 해당한다.
[오답풀이] ①, ② 거란(요) ③ 진(秦) ⑤ 당에 해당
3000년경 나일강 유역에서 발생하였다. 파라오라 15. [출제의도] 청 파악하기
한다.
불리는 통치자는 태양신 ‘라’의 아들이자, 살아 있는
(가) 왕조는 청이다. 청은 서양 선박의 출입을 광저우
최고신으로 여겨져 절대 권력을 가지고 신권 정치를 9. [출제의도] 몽골 제국 파악하기
로 제한하고 공행이 무역을 관리하도록 하였다. 명ㆍ청
행하였다. 이집트 문명의 사람들은 영혼 불멸과 사후 밑줄 친 ‘제국’은 몽골 제국이다. 칭기즈 칸은 기존의
대에는 옥수수, 감자, 고구마 등 새로운 작물이 도입
세계를 믿어 시신을 미라로 만들었고, 피라미드를 건설 부족들을 재편성하여 천호제라는 체계적인 군사ㆍ행정
되었으며, 차, 사탕수수, 담배 등의 상품 작물이 전국
하였다. 또한 죽은 사람을 위한 안내서인 「사자의 서」 조직을 정비하였으며, 이를 토대로 정복 전쟁을 벌여
각지에서 활발하게 재배되었다. 또 막대한 은이 중국
를 남겼다. 지배 영역을 확장하였다. 몽골 제국은 광대한 제국을
에 들어오게 되면서 조세를 은으로 납부하게 되었다.
[오답풀이] ②, ③ 메소포타미아 문명 ④ 중국 문명 원활하게 통치하기 위해 역참을 설치하였다. 이러한
그리고 산시 상인, 휘저우(후이저우) 상인 같은 대상인
⑤ 인도 문명에 해당한다. 역참망의 설치로 사절이나 관리 등 공무 인원의
이 활동하였고, 각지에 회관(동향 조합), 공소(동업
여행과 물자 수송이 체계적으로 이루어졌고, 신속한
2. [출제의도] 아케메네스 왕조 페르시아 파악하기 조합) 등이 세워졌다.
공문서 전달도 가능해져 중앙의 지배력이 멀리 뻗칠
(가) 왕조는 아케메네스 왕조 페르시아이다. 아케메네스
수 있었다. 16. [출제의도] 마테오 리치 파악하기
왕조 페르시아의 왕 다리우스 1세는 도로와 역참제를
밑줄 친 ‘그’는 마테오 리치이다. 명 말 중국에 들어온
정비하고, 페르세폴리스를 건설하였다. 또한 여러 행정 10. [출제의도] 아바스 왕조 파악하기
예수회 선교사들은 선교 활동의 일환으로 천문, 역법
구역에 총독을 파견하였으며, 왕의 눈, 왕의 귀로 밑줄 친 ‘이 왕조’는 아바스 왕조이다. 아바스 왕조는
등 서양의 학문을 소개하였다. 특히 마테오 리치는
불리는 감찰관을 파견하였다. 아랍어를 공용어로 보급하고, 이슬람교를 전파하여
천주실의를 저술하고, 서광계와 함께 기하원본을
[오답풀이] ① 오스만 제국 ② 사산 왕조 페르시아 일정 비율 이상의 주민을 개종시키는 한편, 상업
간행하였다.
③ 우마이야 왕조 ④ 페니키아에 해당한다. 활동을 확대하였다. 또한 당과의 탈라스 전투에서
승리하고 동서 교역로를 장악하여 경제적으로 번영 17. [출제의도] 오스만 제국 파악하기
3. [출제의도] 춘추 전국 시대 파악하기
하였다. 밑줄 친 ‘이 제국’은 오스만 제국이다. 오스만 제국은
(가) 시대는 전국 시대이다. 춘추 시대에는 춘추 5패가
[오답풀이] ① 당 ~ 청 ② 티무르 왕조 ④ 수 ⑤ 정통 데브시르메 제도를 통해 크리스트교도 청소년을 징집
존왕양이를 명분으로 제후국을 통솔하였다. 이후 전국
칼리프 시대의 이슬람 세력에 해당한다. 하여 이슬람교도로 개종시킨 후 술탄의 친위 부대인
시대에는 전국 7웅이 약소 제후국들을 병합하면서
예니체리나 관료로 육성하였다. 한편, 오스만 제국은
약육강식의 치열한 경쟁을 벌였다. 이 시기에는 각국 11. [출제의도] 진시황제 파악하기
메(흐)메트 2세 때 콘스탄티노폴리스를 정복하여
의 제후들이 부국강병을 위해 능력 있는 인재들을 밑줄 친 ‘폐하’는 진시황제이다. 진시황제는 전국
비잔티움 제국을 멸망시켰다.
경쟁적으로 모집함으로써 제자백가라 불리는 사상가 시대를 통일하고, 중국에서 황제의 칭호를 처음으로
[오답풀이] ① 상 ② 인도 문명 ③ 사산 왕조 페르
들이 등장하였다. 사용하였다. 그는 군현제를 시행하고, 수도와 지방을
시아 ⑤ 아케메네스 왕조 페르시아에 해당한다.
잇는 도로망을 건설하였으며, 화폐ㆍ도량형ㆍ문자를
4. [출제의도] 금 이해하기
통일하였다. 또한 법가 사상을 바탕으로 분서갱유를 18. [출제의도] 왕안석의 신법 파악하기
(가) 국가는 금이다. 금은 거란(요)을 멸망시킨 후,
단행하여 반대 세력을 탄압하였다. 밖으로는 흉노를 (가)에 들어갈 탐구 주제로 가장 적절한 것은 ‘왕안석
송의 수도 카이펑을 함락하여 화북 지방을 통치하였다.
몰아내고 만리장성을 쌓았다. 의 신법’이다. 문치주의 정책으로 국방력이 약화된
금은 여진 문자를 만들어 사용하였으며, 여진족은
송은 거란(요)과 서하에 세폐를 제공하였고, 이로
맹안 모극제, 한족은 주현제를 통해 다스렸다. 12. [출제의도] 에도 막부 파악하기
인해 재정 부담이 증가하였다. 이에 신종은 왕안석을
밑줄 친 ‘막부’는 에도 막부이다. 에도 막부는 쇄국
5. [출제의도] 이슬람교 파악하기 등용하여, 부국강병을 위한 개혁을 추진하였다. 하지만
정책을 강화하여 크리스트교 포교를 금지하고 사무역
밑줄 친 ‘이 종교’는 이슬람교이다. 메카의 상인이었던 사마광을 대표로 하는 보수파 관료와 대지주 등의
을 엄격히 통제하였다. 다만 나가사키를 개방하여
무함마드에 의해 유일신 알라를 섬기는 이슬람교가 반대로 개혁은 실패하였다.
중국, 네덜란드 상인과 교역하였다. 이 과정에서
성립하였다. 메디나에서 무슬림 공동체를 만들어 세력 19. [출제의도] 북위 파악하기
네덜란드인으로부터 서양의 학문과 지식이 전해졌는데,
을 키운 무함마드는 메카를 장악하고 아라비아반도
이를 토대로 난학(란가쿠)이 발달하였다. 에도 막부는 지도에서 빗금으로 표시된 국가는 북위이다. 수 문제는
각지로 세력을 확장하였다. 무함마드가 알라로부터
쇼군이 지방 다이묘들을 통제하기 위해 정기적으로 9품중정제를 폐지하고 과거제를 실시하였다. 이후 송
받은 계시를 모아 놓은 이슬람교의 경전이 쿠란이다.
다이묘를 에도에 머물다 가도록 하는 산킨코타이 제도 태조는 과거제에 전시라는 최종 시험 단계를 정례화
6. [출제의도] 한 파악하기 를 실시하였다. 에도 막부 시기에는 가부키, 우키요에 하여 황제권을 강화하였다.
(가) 왕조는 한이다. 한대 사마천은 무제 때까지의 가 유행하는 등 조닌 문화가 발달하였다.
20. [출제의도] 강희제 파악하기
역사를 정리한 사기를 편찬하였다. 한의 전성기를 [오답풀이] ① 명 ② 헤이안 시대 ④ 나라 시대 ⑤
(가) 황제는 청의 강희제이다. 강희제는 오삼계 등이
이끈 무제는 군현제를 전국적으로 시행하여 중앙 집권 가마쿠라 막부에 해당한다.
일으킨 삼번의 난을 진압하고 타이완의 정씨 세력을
체제를 확립하고, 동중서의 건의를 수용하여 유교를 13. [출제의도] 송 파악하기 제압하였으며, 시베리아로 진출한 러시아와 네르친스크
통치 이념으로 채택하였다. 한편 무제는 대규모 군대
밑줄 친 ‘이 나라’는 송이다. 송은 960년 조광윤이 조약을 맺어 국경을 안정시켰다.
를 파견하여 남월과 고조선을 멸망시켰다. 북으로는
건국하였다. 송대에 들어와 이앙법이 보편화되었고,
흉노를 공격하고 서역의 대월지에 장건을 파견하여
새로운 품종인 참파벼가 남방에서 도입되었다. 새로운
흉노를 저지할 동맹군을 얻으려 하였다. 또한 무제는
농법의 도입으로 생산력이 향상되면서 강남의 경제력
잦은 대외 원정으로 재정이 어려워지자 소금과 철의
이 화북을 크게 앞지르게 되었다. 석탄의 사용이 보편

18
2023학년도 6월
전국연합학력평가 정답 및 해설 고2

감소의 원인이다.
7. [출제의도] 경제 주체 파악하기
• 경제 • A는 기업, B는 가계, C는 정부이다. 가계는 생산 요소 14. [출제의도] 자금의 수요와 공급 이해하기
시장에서 공급자이고, 생산물 시장에서 수요자이다. ㉠은 자금 수요가 감소하는 요인이고 ㉡은 자금 공급
[오답풀이] ① ㉡은 ‘예’, ㉠과 ㉢은 ‘아니요’이다. ② 이 증가하는 요인이다.
정 답 기업은 생산물 시장에서 공급자이다. ④ 정부는 사회적 [오답풀이] ㄴ. 자금 수요의 감소는 균형 이자율이
후생의 증대를 추구하는 경제 주체이다. ⑤ 가계와 하락하는 요인이다. ㄹ. 자금 공급의 증가는 자금
1 ① 2 ⑤ 3 ⑤ 4 ⑤ 5 ② 기업은 모두 민간 부문의 경제 주체이므로 해당 질문 거래량이 증가하는 요인이다.
6 ① 7 ③ 8 ③ 9 ③ 10 ④ 은 (가)에 들어갈 수 없다.
15. [출제의도] 시장 실패의 원인 파악하기
11 ① 12 ④ 13 ③ 14 ② 15 ②
8. [출제의도] 기본적인 경제 문제 이해하기 시장에서 거래 당사자들이 가진 거래에 필요한 정보의
16 ③ 17 ⑤ 18 ③ 19 ① 20 ④
(가)에는 ‘무엇을 얼마나 생산할 것인가’, (나)에는 양이 서로 다른 상태를 정보의 비대칭성이라고 한다.
‘어떻게 생산할 것인가’, (다)에는 ‘누구에게 어떻게 이로 인해 시장의 자원 배분 기능이 왜곡되는 시장
해 설 분배할 것인가’를 결정하는 문제가 나타난다. 기본적인 실패가 나타날 수 있다.
경제 문제는 모두 자원의 희소성 때문에 발생하며,
16. [출제의도] 가격 규제 정책 이해하기
기업은 기본적인 경제 문제를 해결할 때에 효율성을
1. [출제의도] 경제 활동의 유형과 객체 이해하기 정부의 가격 규제로 초과 공급이 발생하였으므로 최저
고려한다.
학생이 책가방을 구입하는 것은 재화를 소비하는 활동 가격제가 시행되었음을 알 수 있다. 가격 규제 이후
에 해당한다. 9. [출제의도] 경제 체제 이해하기 수요량은 Q1, 공급량은 Q3이므로 시장 거래량은 Q1
[오답풀이] ② 의사가 환자에게 약을 처방하는 것은 A는 계획 경제 체제, B는 시장 경제 체제이다. 시장 이다.
㉣에 해당한다. ③ 강사가 유료 인터넷 강의를 제작 경제 체제에서는 계획 경제 체제에서보다 개인의 이윤 [오답풀이] ② 최저 가격제는 생산자 보호를 목적으로
하는 것은 ㉣에 해당한다. ④ 관객들이 극장에서 영화 추구 동기가 강하게 나타난다. 한다. ④ 가격 규제 이후 총잉여는 감소한다. ⑤ 가격
를 관람하는 것은 ㉢에 해당한다. ⑤ 부가가치를 창출 [오답풀이] ② ‘보이지 않는 손’에 의한 자원 배분을 규제 이후 Q1Q3만큼의 초과 공급이 발생하였다.
하는 경제 활동은 ㉡, ㉣의 사례가 해당된다. 강조하는 것은 시장 경제 체제의 특징이다. ④ 경제
17. [출제의도] 경합성과 배제성에 따른 재화의 분류
주체 간 자유로운 경쟁을 강조하는 것은 시장 경제
2. [출제의도] 정부의 경제적 역할 이해하기 이해하기
체제의 특징이다. ⑤ 시장 경제 체제에서는 생산 수단
제시된 자료를 통해 정부가 시장에서 충분히 공급되지 A는 사적 재화, B는 공유 자원, C는 공공재이다. 공유
의 사적 소유가 인정된다.
않는 공공재나 사회 간접 자본을 직접 생산하여 공급 자원은 배제성이 없고 경합성이 있으므로 공공재에
함으로써 시장 기능을 보완함을 알 수 있다. 10. [출제의도] 시장 변동과 잉여 이해하기 비해 남용으로 인한 고갈 문제가 발생할 가능성이
X재의 소비자 잉여 변동을 통해 X재의 수요가 감소 높다.
3. [출제의도] 직접세와 간접세 이해하기
했음을 알 수 있다. X재의 수요 감소로 X재 균형 [오답풀이] ② 국방 서비스는 C에 해당한다. ③ 공해
A는 직접세이고, B는 간접세이다. 누진세율이 적용
가격이 하락하였고, X재 균형 거래량은 감소하였다. 상의 어족 자원은 B에 해당한다. ④ 무임승차자 문제
되는 직접세는 비례세율이 적용되는 간접세에 비해
총잉여는 (A+B+C+D)에서 (B+D)로 변동하였으므로 는 배제성을 갖는 A에서는 나타나지 않는다.
소득 재분배 효과가 크고, 조세 부담의 역진성은
(A+C)만큼 감소하였다.
작다. 18. [출제의도] 시장 실패와 정부 실패 이해하기
[오답풀이] ㄱ. X재의 수요가 감소했으므로 X재에
[오답풀이] ㄴ. 간접세는 주로 소비 지출에 부과된다. (가)는 정부의 시장 개입을 통해 시장 실패를 개선한
대한 선호도는 감소하였다. ㄷ. 생산자 잉여는
사례이고, (나)는 정부의 과도한 규제로 인해 정부
4. [출제의도] 합리적 선택 이해하기 (B+C+D)에서 D로 변동하였으므로 (B+C)만큼 감소
실패가 발생한 사례이다.
(가)는 암묵적 비용이다. 기회비용은 명시적 비용과 하였다.
암묵적 비용을 합한 값이고, 순편익은 편익에서 기회 19. [출제의도] 시장 균형 이해하기
11. [출제의도] 기업의 합리적 선택 이해하기
비용을 뺀 값이다. B재 가격이 10% 상승하더라도 가격이 300원일 때 초과 수요량이 0개이므로 균형
표는 갑 기업의 X재 생산량에 따른 이윤, 평균 비용을
B재 선택의 순편익은 10만 원이므로 A재 선택의 가격은 300원이고 균형 거래량은 100개이다. 가격이
나타낸 것이다. X재 가격은 5만 원이고, X재 생산량이
순편익인 –10만원보다 크다. 그러므로 갑의 선택은 200원일 때 초과 수요가 발생하므로 가격 상승 압력
3개일 때 이윤은 7만 원으로 가장 크다.
변함이 없다. 이 발생한다.
(단위: 개, 만 원)
[오답풀이] ① B재를 선택할 때의 암묵적 비용은 [오답풀이] ㄷ. 제시된 자료에서 가격이 500원일 때
생산량 1 2 3 4 5
A재를 선택할 때의 편익에서 명시적 비용을 뺀 값이 초과 공급량은 40개로 가장 많다. ㄹ. 모든 가격 수
이윤 3 6 7 6 3
므로 ㉠은 ‘100’이다. ③ A재 선택의 기회비용은 준에서 수요량이 10개씩 감소하고 공급량이 10개씩
평균 비용 2 2 약 2.67 3.5 4.4
260만 원이고, B재 선택의 기회비용은 240만 원이므로 증가하면, 균형 가격은 200원이 된다.
[오답풀이] ㄹ. X재 생산량을 3개에서 4개로 늘릴
A재 선택의 기회비용이 더 크다. ④ A재의 가격이
때 추가적으로 얻는 수입은 5만 원이고 추가적으로 20. [출제의도] 외부 효과 이해하기
10% 하락하면 A재 선택의 순편익은 –10만 원으로
발생하는 비용은 6만 원이다. X재 시장은 생산 측면에서 외부 불경제가 발생하였다.
음(-)의 값을 갖는다.
따라서 S1은 사적 비용만을 반영한 공급 곡선이고,
12. [출제의도] 대체재와 보완재 이해하기
5. [출제의도] 무상재와 경제재 이해하기 S2는 사회적 비용을 반영한 공급 곡선이다. X재 가격
B재는 A재의 보완재이고, C재는 A재의 대체재이다.
(가)는 경제재이고, (나)는 무상재이다. 무상재에 대한 은 정책 시행 전 15달러에서 정책 시행 후 20달러로
A재의 공급자 수가 감소하면 A재의 가격이 상승한다.
수요가 증가하거나 공급이 감소하면 수요 곡선과 공급 5달러 상승하였다.
따라서 C재의 수요가 증가하여 C재의 가격이 상승
곡선의 이동으로 시장 거래가 이루어질 수 있으므로, [오답풀이] ② 사회적 최적 거래량은 10만 개이다.
한다.
무상재가 경제재로 변하기도 한다. ③ X재 1개당 10달러의 세금이 부과되었다. ⑤ X재
[오답풀이] ② A재의 가격이 하락하면 B재의 수요는
[오답풀이] ㄴ. 무상재는 모든 가격 수준에서 수요량이 판매 수입은 정책 시행 전 225만 달러에서 정책 시행
증가한다. ③ A재의 생산비가 상승하면 A재의 가격이
공급량보다 적다. ㄹ. 무상재는 희소성을 갖지 않는다. 후 200만 달러로 25만 달러 감소하였다.
상승한다. 따라서 B재의 수요가 감소하여 B재의 판매
6. [출제의도] 경제적 유인 이해하기 수입이 감소한다. ⑤ A재의 공급이 증가하면 A재의
경제적 유인이란 편익이나 비용에 변화를 주어 경제 가격이 하락한다. 따라서 C재의 수요가 감소하여 C재
주체가 특정 방식으로 행동하도록 하는 것이다. 의 균형 거래량이 감소한다.
㉠은 일회용 컵 사용에 대한 부정적 유인, ㉡은 사용한
13. [출제의도] 공급량과 공급의 변동 이해하기
일회용 컵 반납에 대한 긍정적 유인이다. ㉠은 일회용
㉠은 공급량의 증가, ㉡은 공급의 증가이다. 공급자
컵 사용자의 비용을 증가시킨다.
수가 증가하면 공급이 증가한다.
[오답풀이] ③ ㉠과 ㉡ 모두 경제적 유인이다. ④ 성적
[오답풀이] ① 가격의 하락은 공급량 감소의 원인
이 우수한 학생에게 장학금을 지급하는 것은 긍정적
이다. ② 생산 기술의 발전은 공급 증가의 원인이다.
유인에 해당한다. ⑤ 제한 속도를 위반한 운전자에게
④ 공급자의 미래 가격 상승 예상은 공급 감소의 원인
범칙금을 부과하는 것은 부정적 유인에 해당한다.
이다. ⑤ 생산에 지급되는 정부 보조금의 감소는 공급

19
2023학년도 6월
전국연합학력평가 정답 및 해설 고2

다당제에 비해 다수당의 횡포가 나타날 가능성이 높다. 동시에 가지는 것은 대통령제의 특징이다.
• 정치와 법 • 7. [출제의도] 심급 제도 이해하기 15. [출제의도] 기본권의 제한 이해하기
갑이 대법원에 소를 제기한 것으로 보아 2심의 판결에 ○○ 법원이 헌법 재판소에 위헌 법률 심판을 제청한
불복하였음을 알 수 있다. 것으로 보아 해당 법률 조항들이 헌법에 위반된다고
정 답 [오답풀이] ① 갑은 형사 재판의 당사자이다. ② ○○ 판단하였음을 알 수 있다.
지방 법원 단독 판사가 1심을 담당하였으므로 2심은 [오답풀이] ① 직업의 자유는 소극적 성격의 권리이다.
1 ④ 2 ① 3 ③ 4 ⑤ 5 ② ○○ 지방 법원 본원 합의부가 담당하였다. ④ 대법원은 ③ 헌법 재판소는 해당 법률 조항들에 대해 위헌 법률
6 ③ 7 ③ 8 ④ 9 ⑤ 10 ③ 갑의 상고 사건을 담당하였다. ⑤ 형사 재판은 3심제 심판을 하였다. ④ 헌법 재판소는 해당 법률 조항들이
11 ④ 12 ⑤ 13 ⑤ 14 ④ 15 ② 이므로 갑은 원칙적으로 대법원의 판결에 대해 상소 과잉 금지의 원칙에 위반되지 않는다고 결정하였으므로
16 ① 17 ② 18 ① 19 ③ 20 ④ 할 수 없다. 목적의 정당성을 충족한다고 판단하였다. ⑤ 헌법
재판소의 합헌 결정으로 인해 해당 법률 조항들의 효력이
8. [출제의도] 국가 기관 간 견제 수단 이해하기
유지되므로 안경사가 아닌 자의 안경업소 개설은 금지
해 설 국무총리 임명 동의권은 입법부가 행정부를, 명령ㆍ
된다.
규칙ㆍ처분 심사권은 사법부가 행정부를 견제하는
수단이다. 16. [출제의도] 법률의 제ㆍ개정 과정 이해하기
1. [출제의도] 정치를 바라보는 관점 이해하기
[오답풀이] 사면권은 행정부가 사법부를 견제하는 ㄱ. 상임 위원회는 국회의 효율적인 의사 진행을 위해
갑은 좁은 의미로, 을은 넓은 의미로 정치를 바라보고
수단이다. 국정 감사권은 입법부가 행정부를 견제하는 운영된다. ㄴ. 법률 개정안의 본회의 의결 정족수는
있다. 넓은 의미로 정치를 바라보는 관점은 좁은 의미로
수단이다. 국회 재적 의원 과반수의 출석과 출석 의원 과반수의
정치를 바라보는 관점에 비해 복잡하고 다원화된 현대
찬성이다.
사회의 정치 현상을 설명하기에 적합하다. 9. [출제의도] 민주 정치의 발전 과정 이해하기
[오답풀이] ㄷ. 법률안 제ㆍ개정 과정에서는 국민
[오답풀이] ① 국가 형성 이전에 나타난 정치 현상을 A는 현대 민주 정치, B는 근대 민주 정치, C는 고대
투표를 거치지 않는다. ㄹ. 대통령이 법률안 거부권을
설명하기에 용이한 것은 넓은 의미로 정치를 바라보는 아테네 민주 정치이다. 근대 민주 정치는 입헌주의
행사하면 국회 재의결 과정을 거친다.
관점이다. ② 다른 사회 집단의 활동과 구별되는 국가 원리를 통한 민주 정치를 지향한다.
활동의 특수성을 강조하는 것은 좁은 의미로 정치를 [오답풀이] ① 추첨, 윤번제를 통해 공직자를 선출하는 17. [출제의도] 정치 과정과 정치 참여 이해하기
바라보는 관점이다. ③ 넓은 의미로 정치를 바라보는 것은 고대 아테네 민주 정치이다. ② 모든 사회 구성원의 정부가 저출산 대책을 수립하는 것은 정치 과정에서
관점은 좁은 의미로 정치를 바라보는 관점에 비해 정치 참여가 보장되는 것은 현대 민주 정치이다. ③ 산출에 해당한다.
정치 과정에 참여하는 주체가 다양하다고 본다. ⑤ 천부 인권 사상을 바탕으로 통치가 이루어지는 것은 근대 [오답풀이] ① 언론은 정책 결정 기구에 해당하지
시민 단체 내의 의사 결정 과정을 정치라고 보는 것은 민주 정치와 현대 민주 정치이다. ④ 근대 민주 정치와 않는다. ③ 언론과 시민 단체 모두 여론을 형성하는
넓은 의미로 정치를 바라보는 관점이다. 현대 민주 정치 모두 국민 주권의 원리에 기반한다. 기능을 한다. ④ ㉢, ㉣ 모두 정치 참여 주체의 정치적
효능감 향상에 기여할 수 있다. ⑤ ㉣은 정치 참여의
2. [출제의도] 우리나라 헌법의 기본 원리 이해하기 10. [출제의도] 우리나라 선거 제도의 특징 이해하기
시ㆍ공간적 제약을 완화할 수 있는 방법이다.
A는 국제 평화주의이다. 국제 평화주의는 국가가 국제 대통령 선거는 5년마다 실시되므로 순서 1은 옳지 않은
질서를 존중하고 평화를 추구하여야 한다는 원리이다. 설명이다. 비례 대표 국회 의원 선거는 정당 명부식 18. [출제의도] 정치 참여 집단 이해하기
[오답풀이] ② 국민 주권주의, ③ 문화 국가의 원리, 비례 대표제를 채택하고 있으므로 순서 2는 옳은 설명 A는 정당, B는 시민 단체, C는 이익 집단이다. 정당은
④ 복지 국가의 원리, ⑤ 자유 민주주의이다. 이다. 기초 의회 지역구 의원 선거는 중ㆍ대선거구제가 정부와 의회를 매개하는 역할을 한다.
적용되므로 순서 3은 옳은 설명이다. [오답풀이] ② 자신들의 활동에 대해 정치적 책임을
3. [출제의도] 전형적인 정부 형태 이해하기
지는 것은 정당이다. ③ 정당, 이익 집단, 시민 단체
갑국의 정부 형태는 전형적인 대통령제, 을국의 정부 11. [출제의도] 주민 참여 제도 이해하기
모두 정치 사회화 기능을 수행한다. ④ 정권 획득을
형태는 전형적인 의원 내각제이다. 전형적인 의원 내각제 주민 참여 예산 제도의 실시로 인한 기대 효과로
목적으로 하는 것은 정당이다. ⑤ 정당, 이익 집단,
에서 의회 의원은 각료를 겸직할 수 있다. 대의제의 한계 보완, 주민의 자치 의식 신장, 정책
시민 단체 모두 정부 정책에 대한 감시와 비판의 기능을
[오답풀이] ① 권력 융합에 충실한 정부 형태는 전형 결정의 정당성 확보, 지방 재정 운영의 투명성 제고를
수행한다.
적인 의원 내각제이다. ② 행정부 수반의 임기가 엄격히 들 수 있다.
보장되는 것은 전형적인 대통령제이다. ④ 의회가 내각 19. [출제의도] 민주 선거의 원칙 이해하기
12. [출제의도] 법치주의의 유형 이해하기
불신임권을 가지는 것은 전형적인 의원 내각제이다. A는 평등 선거, B는 보통 선거이다. 평등 선거는 모든
A는 실질적 법치주의, B는 형식적 법치주의이다. 형식적
⑤ 행정부가 의회에 대해 정치적 책임을 지는 것은 유권자가 평등하게 같은 수의 표를 행사하는 것으로
법치주의와 실질적 법치주의 모두 국민의 기본권을
전형적인 의원 내각제이다. 표의 등가성을 보장하는 원칙이다. 보통 선거는 재산이나
제한할 때 법적 근거를 강조한다.
성별 등에 따른 제한 없이 일정 연령 이상의 모든
4. [출제의도] 기본권의 유형 이해하기 [오답풀이] ① 독재를 정당화하는 근거로 악용될
국민에게 선거권을 부여하는 원칙이다.
밑줄 친 ㉠, ㉡이 공통적으로 해당되는 기본권의 유형은 우려가 있는 것은 형식적 법치주의이다. ② 법치주의는
청구권이다. 청구권은 다른 기본권이 침해되었을 때 통치자를 포함한 모든 사람이 법에 구속되어야 한다고 20. [출제의도] 선거 자료 분석하기
이를 구제하기 위한 수단적 권리이다. 본다. ③ 법치주의는 국가 권력의 자의적인 행사를 구분 A당 B당 C당 D당
[오답풀이] ① 평등권, ② 사회권, ③ 자유권, ④ 참정권 경계한다. ④ 실질적 법치주의는 위헌 법률 심사제의 지역구 의석수(석) 5 4 1 0
이다. 필요성을 강조한다. 지역구 의석률(%) 50 40 10 0
지역구 득표율(%) 40 30 20 10
5. [출제의도] 우리나라 헌법 기관 이해하기 13. [출제의도] 지방 자치 이해하기 비례 대표 의석수(석) 4 3 1 2
A는 대법원, B는 대통령, C는 국회이다. 대통령은 긴급 지방 자치 단체장은 조례의 범위에서 지방 자치 단체의 비례 대표 의석률(%) 40 30 10 20
재정ㆍ경제 처분 및 명령권을 가진다. 사무에 관한 규칙을 제정할 수 있다. 총의석수(석) 9 7 2 2
[오답풀이] ① 헌법 재판소의 권한이다. ③ 감사원의 [오답풀이] ① ㉠은 광역 자치 단체, ㉣은 기초 자치 총의석률(%) 45 35 10 10
권한이다. ④ 대통령의 조약 체결ㆍ비준에 대한 동의권은 단체이다. ② 지방 자치 단체를 대표하고 지방 행정 C당은 지역구 득표율(20%)이 지역구 의석률(10%)에
국회의 권한이다. ⑤ 대통령은 헌법 개정안을 발의할 사무를 총괄하는 것은 지방 자치 단체장이다. ③ ○○도 비해 높다.
수 있다. 주민에게는 조례의 개정 및 폐지 청구권이 인정된다. [오답풀이] ① 지역구 의원 선거구 제도는 중ㆍ대
④ 지방 의회는 지방 자치 단체장이 편성한 예산을 선거구제이다. ② 지역구 의원 선거에서 A당은 사표가
6. [출제의도] 정당 제도의 유형 이해하기
심의하고 확정하는 권한을 가진다. 발생하지 않았다. ③ B당은 지역구 의석률(40%)이
A는 다당제, B는 양당제이다. 양당제는 다당제에 비해
비례 대표 의석률(30%)에 비해 높다. ⑤ C당과 D당의
정당 간 대립 시 중재가 어렵다. 14. [출제의도] 우리나라 정부 형태 이해하기
총의석률은 10%로 같다.
[오답풀이] ① 양당제가 다당제에 비해 강력한 정책 (가)에는 우리나라 정부 형태의 의원 내각제 요소가
추진에 유리하다. ② 양당제가 다당제에 비해 정책 들어가야 한다. 국무총리 제도, 행정부의 법률안 제출권,
실패에 대한 책임 소재가 분명하다. ④ 다당제가 양당제에 국회의 국무 위원 해임 건의권은 의원 내각제 요소이다.
비해 군소 정당의 난립 가능성이 높다. ⑤ 양당제가 [오답풀이] ㄹ. 행정부 수반이 국가 원수의 지위를

20
2023학년도 6월
전국연합학력평가 정답 및 해설 고2

집단은 친밀한 대면 접촉을 중심으로 전인격적 인간


14. [출제의도] 대중매체 관련 자료 분석하기
• 사회∙문화 • 관계가 주로 나타난다. 2차 집단은 특정 목적 달성을
ㄱ. 텔레비전은 뉴 미디어에 비해 정보 생산자와
위한 수단적이고 형식적인 인간관계가 주로 나타난다.
소비자 간의 경계가 명확하다. ㄷ. 일방향 매체를
[오답풀이] ① 회사는 이익 사회에 해당한다. ② 공동
이용한다고 응답한 사람의 수는 최소 740명, 최대
사회는 결합 자체가 목적인 집단이다. ④ 1차 집단은
정 답 940명이다.
구성원에 대한 비공식적 통제가, 2차 집단은 공식적인
[오답풀이] ㄴ. 뉴 미디어는 텔레비전에 비해 정보
통제가 주로 나타난다.
1 ④ 2 ① 3 ④ 4 ② 5 ④ 전달과 수용의 시ㆍ공간적 제약이 작다. ㄹ. 뉴 미디어의
6 ⑤ 7 ③ 8 ① 9 ② 10 ③ 8. [출제의도] 사회ㆍ문화 현상의 탐구 태도 이해하기 신뢰도가 가장 낮다.
11 ③ 12 ⑤ 13 ① 14 ② 15 ④ 제시문에서 강조하는 사회ㆍ문화 현상의 탐구 태도는
15. [출제의도] 문화의 속성 파악하기
16 ③ 17 ⑤ 18 ⑤ 19 ① 20 ⑤ 객관적 태도이다. 객관적 태도는 현상을 있는 그대로
갑국과 을국에 공통적으로 나타난 문화의 속성은
관찰하고 사실과 가치를 엄격히 분리해야 함을 강조
공유성, 총체성, 변동성이다. 문화의 공유성으로 다른
한다.
해 설 사람의 행동을 예측할 수 있다.
[오답풀이] ② 개방적 태도에 해당한다. ③ 성찰적
[오답풀이] ㄷ. 축적성에 관한 설명이다.
태도에 해당한다. ⑤ 상대주의적 태도에 해당한다.
1. [출제의도] 사회ㆍ문화 현상의 특징 이해하기 16. [출제의도] 사회 집단과 사회 조직 자료 분석하기
9. [출제의도] 자료 수집 방법의 특징 이해하기
㉠은 자연 현상, ㉡, ㉢은 사회ㆍ문화 현상이다. 자연 제시된 자료에서 갑은 가족, 을은 ◇◇ 시민 단체와
A는 질문지법, B는 참여 관찰법이다. 참여 관찰법은
현상은 동일한 조건에 동일한 결과가 나타나므로 인과 대학교, 병은 △△ 회사와 △△ 회사 노동조합, 정은 마을
실제성 높은 생생한 자료를 수집하기에 용이하다.
관계가 분명하다. 봉사단에 소속되어 있음을 알 수 있다. △△ 회사,
[오답풀이] ① 문헌 연구법에 해당한다. ③ 참여 관찰
[오답풀이] ① 자연 현상은 확실성의 원리가 적용된다. △△ 회사 노동조합은 모두 공식 조직이다.
법은 예상치 못한 상황에 대한 통제가 어렵다. ④ 참여
② 자연 현상은 존재 법칙, 사회ㆍ문화 현상은 당위 [오답풀이] ① 가족은 1차적 사회화 기관이다. ② ◇◇
관찰법은 시간과 비용이 많이 든다. ⑤ 조사 대상자
법칙이 적용된다. ③ 사회ㆍ문화 현상은 가치 함축적 시민 단체는 자발적 결사체이다. 대학교는 자발적 결사
와의 언어적 상호 작용이 필수적인 자료 수집 방법은
이다. ⑤ 사회ㆍ문화 현상과 자연 현상은 모두 보편성이 체가 아니다. ④ 비공식 조직은 공식 조직 내에서 공동의
질문지법과 면접법이다.
나타난다. 관심이나 취미를 가진 사람들로 구성된다. ⑤ 대학교를
10. [출제의도] 문화 이해 태도 이해하기 제외한 자료에 제시된 모든 집단은 비공식적 사회화
2. [출제의도] 일탈 이론 이해하기
A는 자문화 중심주의, B는 문화 사대주의, C는 문화 기관이다.
A는 머튼의 아노미 이론, B는 차별 교제 이론이다.
상대주의이다.
[오답풀이] ② 차별 교제 이론에 대한 설명이다. 17. [출제의도] 문화를 바라보는 관점 파악하기
[오답풀이] ① 자문화 중심주의는 자문화의 우수성을
③ 낙인 이론에 대한 설명이다. ④ 차별 교제 이론은 A는 총체론적 관점, B는 상대론적 관점이다. 총체론적
지나치게 강조한 나머지 타문화를 부정적으로 여기
일탈 행동 발생 과정에서 나타나는 상호 작용에 주목 관점은 문화 요소 간의 관계를 전체적으로 살펴
므로 문화의 다양성을 보존하는 데 기여할 수 없다.
한다. ⑤ 머튼의 아노미 이론과 차별 교제 이론은 봄으로써 문화에 대한 편협한 이해를 방지하는 데
② 자문화 중심주의는 국수주의를 초래할 가능성이
모두 일탈을 규정하는 객관적 기준이 존재한다고 본다. 기여한다.
높다. ④ 문화 사대주의는 다른 문화 수용에 적극적
3. [출제의도] 사회 실재론과 사회 명목론 이해하기 이다. ⑤ 문화 사대주의는 자문화의 정체성을 상실할 18. [출제의도] 연구 윤리 평가하기
A는 사회 명목론, B는 사회 실재론이다. ㄹ. (가)에는 우려가 크다. 갑은 심층 면접의 대상자에게 충분한 정보를 제공하지
옳은 설명이 들어가야 한다. 사회 명목론은 사회 문제 11. [출제의도] 인간의 사회화 이해하기 않았다.
해결을 위해 의식 개선을 강조한다. [오답풀이] ④ 결과 발표 과정에서 익명성을 보장하
역할 행동이 사회적 기대에 부합하면 칭찬과 보상이
[오답풀이] ㄱ. 사회 실재론에 해당하는 설명이다. 였다.
따른다.
ㄷ. 옳은 설명이므로 ㉠은 ‘1점’이다.
[오답풀이] ① ㉠, ㉢은 모두 성취 지위이다. ② 재사회화 19. [출제의도] 문화의 의미 이해하기
4. [출제의도] 사회ㆍ문화 현상을 바라보는 관점 이해하기 는 사회 변화나 새로운 환경에 적응하기 위해 이전 ㉠은 넓은 의미의 문화, ㉡은 좁은 의미의 문화이다.
갑의 관점은 상징적 상호 작용론, 을의 관점은 갈등론에 과는 다른 규범이나 가치, 기능 등을 학습하는 것을 [오답풀이] ② ‘노인 문화’의 ‘문화’는 넓은 의미의
해당한다. 갈등론은 집단 간의 대립과 갈등을 사회의 의미한다. ④ 갑은 ㉥에 소속되어 있지 않다. ⑤ 역할 문화이다. ③ 좁은 의미의 문화는 후천적으로 학습된
필연적 속성으로 파악한다. 갈등이 아니다. 행위 중 세련되고 교양 있는 것을 의미한다. ④ 평가적
[오답풀이] ① 기능론에 해당한다. ③ 기능론에 해당 12. [출제의도] 사회ㆍ문화 현상의 연구 방법 이해하기 의미가 내포된 것은 좁은 의미의 문화이다. ⑤ ㉠, ㉡은
한다. ④ 상징적 상호 작용론에 해당한다. ⑤ 갈등론은 모두 물질문화를 구성 요소로 한다.
A는 양적 연구, B는 질적 연구이다. 양적 연구는 변인
거시적 관점이고, 상징적 상호 작용론은 미시적 관점
간의 관계를 파악하여 일반화나 법칙을 정립하려는 20. [출제의도] 하위문화의 특징 이해하기
이다.
목적으로 계량화된 자료 수집과 통계 분석을 통해 (가)는 하위문화이다. 다원화된 현대 사회에서는
5. [출제의도] 양적 연구 사례 분석하기 결론을 도출하는 연구 방법이다. 질적 연구는 사회ㆍ 다양한 하위문화가 나타난다. 하위문화는 구성원의
사례에서 연구자 갑이 활용한 자료 수집 방법은 실험 문화 현상에 담긴 인간 행위의 동기나 목적 파악을 욕구를 실현할 수 있게 해주고 같은 문화를 공유하고
법이다. A 집단은 독립 변인을 처치하는 실험 집단이고, 중시하고 직관적 통찰, 감정 이입적 이해 등을 통해 있는 구성원 간의 소속감과 결속력을 다져 주기도
B 집단은 통제 집단이다. 행위에 대한 의미 해석을 시도한다. 양적 연구, 질적 한다. 또한 다양한 하위문화는 역동적이고 지속 가능한
[오답풀이] ① ㉠은 독립 변인, ㉡은 종속 변인이다. 연구는 모두 경험적 자료를 바탕으로 사회ㆍ문화 사회를 만드는 데 기여하기도 하고 시대의 변화에
② 갑의 연구에서 모집단은 청소년이고, ㉢은 표본 현상을 연구한다. 따라 주류 문화로 변화되기도 한다.
집단이다. ③ ㉣은 종속 변인에 대한 조작적 정의에 [오답풀이] ① 질적 연구에 해당한다. ② 양적 연구에 [오답풀이] ⑤ 주류 문화는 한 사회 구성원이 전반적
해당한다. ⑤ ㉦은 1차 자료이다. 해당한다. ③ 질적 연구는 편지, 일기와 같은 비공식적 으로 공유하는 문화를 의미하는데 하위문화의 총합이
자료를 중시한다. ④ 양적 연구는 객관적이고 정밀한 주류 문화는 아니다.
6. [출제의도] 양적 연구 사례 분석하기 연구에 적합하다.
갑이 활용한 연구 방법은 양적 연구이다. ㄹ. 분석
결과는 통계적으로 유의미하므로 갑의 가설은 수용될 13. [출제의도] 관료제와 탈관료제의 특징 이해하기
것이다. A는 관료제, B는 탈관료제이다. 관료제는 복잡한
[오답풀이] ㄱ. 양적 연구는 방법론적 일원론을 전제 업무를 효율적으로 처리하기 위해 업무가 세분화,
한다. ㄴ. B 집단은 통제 집단으로 재난 대응 훈련 전문화 되어 있다.
프로그램의 적용을 받지 않았다. [오답풀이] ②, ③ 관료제는 위계의 서열화, 연공
서열에 따른 보상이 이루어진다. ④ 관료제는 정해진
7. [출제의도] 사회 집단의 유형 파악하기 규칙과 절차에 따라 업무를 처리하므로 업무 처리
A는 공동 사회, B는 이익 사회, C는 1차 집단, D는 절차의 예측 가능성이 높다. ⑤ 관료제와 탈관료제
2차 집단이다. 공동 사회는 구성원의 본질 의지, 이익 모두 효율적 과업 수행을 추구한다.
사회는 구성원의 선택 의지에 기초하여 형성된다. 1차

21
2023학년도 6월
전국연합학력평가 정답 및 해설 고2

[오답풀이] ㄱ. A와 B가 같은 빗면에서 운동하므로


14. [출제의도] 역학적 에너지 이해하기
• 물리학Ⅰ • 가속도의 크기는 같다.
ㄴ. Q가 P보다 더 높은 곳이므로 P에서 Q로 올라가는
ㄷ. p에서 A의 속력을 , 만날 때까지 A와 B의 속도
동안 제기의 중력 퍼텐셜 에너지는 증가한다.
       
변화량을 라 하면,       이 ㄷ. 제기의 속력은 P를 지나 올라갈 때가 P를 지나
 
정 답 므로   이다. 따라서 만나는 순간 A의 속력은 내려올 때보다 크므로 운동하는 동안 역학적 에너지는
, B의 속력은 이므로 만나는 순간 속력은 A가 공기 저항에 의해 손실된다. 따라서 Q에서 P까지 내려
1 ② 2 ③ 3 ⑤ 4 ② 5 ③  오는 동안 제기의 역학적 에너지는 감소한다.
B의  배이다. [오답풀이] ㄱ. 제기가 Q에 도달한 후 아래로 운동
6 ① 7 ② 8 ⑤ 9 ③ 10 ① 
11 ⑤ 12 ④ 13 ⑤ 14 ④ 15 ④ 하므로 제기에 작용하는 알짜힘은 이 아니다.
16 ② 17 ① 18 ③ 19 ③ 20 ④ 8. [출제의도] 운동량 보존 법칙 이해하기
충돌 전과 후 운동량이 보존되므로 A  B  15. [출제의도] 역학적 에너지 보존 이해하기

A ㄴ. A와 B의 운동 에너지 변화량이 같으므로 A와 B의



해 설    A    B  에서 A  B이므로    질량은 같다.
 B
ㄷ. p에서 q까지 이동하는 동안, A와 B의 역학적 에너지의
이다.
합은 보존된다. A의 중력 퍼텐셜 에너지 증가량과 A,
1. [출제의도] 운동의 종류 분류하기
9. [출제의도] 뉴턴 운동 법칙 이해하기 B의 운동 에너지의 증가량의 합은 B의 중력 퍼텐셜
B의 운동은 속력과 운동 방향이 모두 변하는 운동이다.
ㄱ. (나)에서 A의 가속도의 크기가  ms 이므로 A에 에너지의 감소량과 같다. 따라서 B의 중력 퍼텐셜 에너지는
[오답풀이] A의 운동은 속력은 일정하지만 운동 방향이
작용하는 알짜힘의 크기는  N 이다. (가)에서 A가  J 만큼 감소한다. ㉠은  J 이다.
변하는 운동이다.
정지해 있으므로 실이 A를 당기는 힘의 크기는  N 이다. [오답풀이] ㄱ. A의 운동 에너지와 중력 퍼텐셜 에
C의 운동은 운동 방향은 일정하지만 속력이 변하는
ㄷ. (가)에서 실이 B를 당기는 힘의 크기는  N 이고, 너지가 증가하므로 A의 역학적 에너지는 증가한다.
운동이다.
저울이 B를 떠받치는 힘의 크기는  N 이므로 B의
16. [출제의도] 역학적 에너지 보존과 충격량 이해하기
2. [출제의도] 위치 - 시간 그래프 분석하기 무게는  N 이다. 따라서 (나)에서 저울에 측정된
역학적 에너지 보존 법칙을 이용하여 벽에 충돌하기
ㄱ. 초부터 초까지 운동 방향은 변하지 않고 위치만 힘의 크기는  N 이다.

 m 변하였으므로 이동 거리는  m 이다. [오답풀이] ㄴ. (가)에서 지구가 B를 당기는 힘의 크기는 직전 물체의 속력 를 구하면,  ×  ×    ×  × 

ㄷ. 초부터 초까지 그래프의 기울기가 일정하므로 저울이 B를 떠받치는 힘의 크기와 실이 B를 당기는 
  ×  ×  이므로    ms 이고, 역학적 에너지
등속도 운동을 한다. 힘의 크기의 합과 같다. 
[오답풀이] ㄴ. 위치- 시간 그래프에서 기울기의 크기는 보존을 이용하여 벽에 충돌한 직후 물체의 속력  ′ 을
10. [출제의도] 충돌에서 평균 힘 이해하기  
속력을 나타낸다. 속력은 초일 때가 초일 때보다 작다. 구하면,  ×  ×    ×  ×    ×  ×  ′  이므로
A와 B가 용수철에서 분리된 직후 A와 B의 운동량의  
3. [출제의도] 힘의 상호 작용 이해하기 크기는 같고, 속력은 A가 B보다 작으므로 질량은  ′   ms 이다. 따라서 속도 변화량의 크기가  ms
ㄱ. B는 정지해 있으므로 알짜힘은 이다. A  B이다. 벽에 충돌하기 전후, 물체의 운동량 이므로, 충격량의 크기(=운동량 변화량의 크기)는
ㄴ. 수평면이 A를 떠받치는 힘과 A가 수평면을 누르는  N·s 이다.
변화량의 크기는 A와 B가 같고, 벽과의 충돌 시간은
힘은 크기가 같고 방향이 반대이므로 작용과 반작용 A가 B보다 작으므로 벽으로부터 받은 평균 힘의 크기는 17. [출제의도] 열역학 제1법칙 이해하기
관계이다. A  B이다. ㄱ. (가) → (나)에서 압력이 일정하고 부피가 증가하므로
ㄷ. 수평면이 A를 떠받치는 힘의 크기는 A와 B에 작용 온도는 (나)에서가 (가)에서보다 높다.
하는 중력의 크기의 합과 같다. 11. [출제의도] 운동량과 충격량의 관계 이해하기
[오답풀이] ㄴ. (가) → (나) 과정에서 기체의 온도는
초일 때 속력은  ms 이므로 운동량의 크기는
4. [출제의도] 힘의 상호 작용 이해하기 높아지므로 내부 에너지는 증가한다.
 kg ×  ms   kg ·ms 이고, 초부터 초까지 받은
ㄷ. (가) → (나) 과정에서 기체가 흡수한 열은 기체의
ㄷ. 지구가 인공위성을 당기는 힘과 인공위성이 지구를 
충격량의 크기는  N ×  s   ×  N   N ×  s 내부 에너지 증가량과 기체가 외부에 한 일의 합과 같다.
당기는 힘은 작용 반작용 관계이므로 두 힘의 크기는 같다. 
[오답풀이] ㄱ. 인공위성의 운동은 속력은 일정하지만,   N·s 이다. 따라서 초일 때 운동량의 크기가 18. [출제의도] 열역학 제1법칙 이해하기
운동 방향이 변하기 때문에 인공위성의 속도는 변한다.  kg ·ms ㄱ. 기체의 압력은 C에서가 A에서보다 크므로 기체의
 kg ·ms 이므로 초일 때 A의 속력은 
ㄴ. 인공위성의 운동 방향은 원의 접선 방향이고, 인  kg 온도는 C에서가 A에서보다 높다.
공위성에 작용하는 알짜힘의 방향은 인공위성의 운동  ㄴ. A → B 과정에서 기체의 부피가 증가하므로 기체는
  ms 이다.
방향과 수직이다. 
외부에 일을 한다.
5. [출제의도] 뉴턴 운동 법칙 탐구하기 12. [출제의도] 운동량 보존과 충격량 이해하기 [오답풀이] ㄷ. A → C 과정에서 기체의 온도는 높아
A가 B로부터 받은 충격량의 크기가  이므로 B가 지므로 기체는 열을 흡수한다.
ㄱ. 수레에 작용하는 힘이 일정하므로 수레는 등가속도
운동한다. 따라서 속도가 일정하게 증가하기 때문에 A로부터 받은 충격량의 크기도  이다. 한 덩어리가 19. [출제의도] 열역학 제1법칙과 열효율 이해하기
㉠은  ms 이다. 되어 운동하는 B의 속력은  이므로 충돌 후 A와 B의
ㄱ. D → A 과정에서 기체가 흡수한 열량은 이고,
ㄴ. (가), (나)에서 수레의 질량은 같고, 초 동안에 운동량의 합은  이다. 따라서 충돌 직전 A의
기체는 외부로부터 일을 받았으므로 기체의 온도는
속도 변화량의 크기는 (나)에서가 (가)에서보다 크므로 운동량의 크기는  이고, P를 지나기 전 A의
A에서가 D에서보다 높다.
 은  N 보다 크다. 운동량의 크기는 이므로 P에서 A가 받은 충격량의
ㄴ. C → D 과정에서 기체가 방출한 열량은  J 이므로
[오답풀이] ㄷ. (가), (다)에서 수레에 작용하는 힘의 크기는  이다.
내부 에너지 감소량은  J 이다. 따라서 ㉠은 이다.
크기는 같고, 초 동안에 속도 변화량의 크기는 13. [출제의도] 역학적 에너지 보존 이해하기 [오답풀이] ㄷ. A → B → C → D → A를 따라 순환
(가)에서가 (다)에서의 배이므로 질량은 (다)에서가 ㄱ. 용수철의 늘어난 길이가 r에서가 q에서보다 크므로 하는 동안 흡수한 열량은  J 이고, 한 일이  J
(가)에서의 배이다. 따라서 추의 질량은 수레의 용수철에 저장된 탄성 퍼텐셜 에너지는 r에서가 q에 
이므로 열기관의 열효율은  이다.
질량의 배이다. 서보다 크다. 

6. [출제의도] 힘과 운동의 법칙 이해하기 ㄴ. p와 q에서 역학적 에너지는 같다. 운동 에너지는 20. [출제의도] 역학적 에너지 이해하기
ㄱ. A와 B는 함께 운동하므로 A와 B의 가속도의 크기는 같다. q에서가 p에서보다 크므로, 물체의 중력 퍼텐셜 에너지와 A와 B의 역학적 에너지 합의 감소량은 마찰에 의해
용수철에 저장된 탄성 퍼텐셜 에너지의 합은 p에서가 손실된 역학적 에너지의 양과 같다. 마찰에 의한
[오답풀이] ㄴ, ㄷ. A와 B의 가속도의 크기는 같고,
q에서보다 크다. 역학적 에너지 감소량(-  ) = B의 중력 퍼텐셜
질량의 비가   이므로 작용하는 알짜힘의 비는   
 ㄷ. p에서 r까지 중력 퍼텐셜 에너지 감소량과 탄성 퍼텐셜
에너지 감소량(-4 ) + A의 운동 에너지 증가량(  )
이다. 따라서 A가 B에 작용하는 힘의 크기는   이다. 
 에너지 증가량이 같으므로       이다.
 + B의 운동 에너지 증가량(  )이다. 따라서 B의 운동
7. [출제의도] 등가속도 직선 운동 이해하기 에너지 증가량    이다. 운동 에너지의 증가량은

따라서 용수철 상수는    이다.
ㄴ. 같은 시간 동안 이동 거리가 A가 B의 배이므로  B
B가 A의 배이므로  는 이다.
평균 속력은 A가 B의 배이다. A

22
2023학년도 6월
전국연합학력평가 정답 및 해설 고2

9. [출제의도] 동위 원소와 평균 원자량 이해하기 16. [출제의도] 용액의 몰 농도( M ) 구하기


• 화학Ⅰ • 
ㄱ. X 와  
X 는 양성자수가 같고 질량수가 
X>  M A  mL 에  M A  mL 를

X 이므로 중성자수는  
X > X 이다. 첨가하여 만든 A의 몰 농도는
    ×    ×  mol
ㄷ. 존재 비율이 X > X 이므로 자연계에서 분자의
   M 이므로   
정 답   
 L
존재 비율은 X > X 이다.
이다.  M A   mL 에 물  mL 를 추가하여
[오답풀이] ㄴ. X 의 평균 원자량은  ×  mol
1 ② 2 ⑤ 3 ④ 4 ④ 5 ⑤ 만든 A 의 몰 농도( M )는 
 ×      ×     
6
11


7
12


8
13


9
14


10
15


 
    이다.
 
L  
16 ④ 17 ② 18 ① 19 ① 20 ④ 
10. [출제의도] 바닥상태 원자의 전자 배치 이해하기   M 이므로   이다. 따라서  ×    이다.

He 과 Ne 의 바닥상태 전자 배치는 각각  ,
   이므로   ,    이다. 따라서      17. [출제의도] 기체의 양( m o l ) 이해하기
해 설
이다. (나)에서 기체의 몰비는 A g  : B g  =  : 이고,
질량비는 A g  : B g  =  : 이므로 분자량비는
1. [출제의도] 화학의 유용성 이해하기 11. [출제의도] 기체의 양( m o l )과 분자량 이해하기 A : B =  : 이다. (나)에 추가된 B g   g 의
하버는 질소 기체와 수소 기체로 암모니아를 대량 같은 온도와 압력에서 기체의 양( mol )은 부피에 부피는  L 이고, (다)의 전체 부피가  L 이므로
비례한다. A g   g 의 부피는  L 이다. A g   g 의 부피가
합성하는 방법을 개발하였다. 따라서 ㉠은 질소이다.
ㄱ. 기체의 분자 수는 (나)에서의 B g 가 (가)에서의 B 의 분자량
2. [출제의도] 탄소 화합물 이해하기 A g  의 배이다.  L 이므로    이다. 따라서 ×
A 의 분자량
ㄱ. CHCOOH은 산성이다. A g 의 밀도 A g 의 부피
[오답풀이] ㄴ.  ,  이다.
ㄴ. CHOH 은 살균 효과가 있어 손 소독제를 만드 B g 의 밀도 B g 의 부피
는데 이용된다.  18. [출제의도] 화학식량과 몰 이해하기
  이므로 기체의 질량은 (가)에서의 A g 와
ㄷ. (가) ~ (다)는 모두 탄소를 포함하고 있으므로  ㄱ.  g 에 들어 있는 분자 수는 분자량에 반비례하므로
탄소 화합물이다. (나)에서의 B g 가 같다. 분자량비는 (가) : (나) =  : 이다. 따라서 (가)는
ㄷ. 같은 부피에 들어 있는 기체의 질량은 A 가 B 의 BA 이고, (나)는 A 이다.
3. [출제의도] 물질의 양( mo l ) 이해하기
배이므로 기체의 분자량은 A 가 B 의 배이다. [오답풀이] ㄴ. A 와 B 의 원자량을 각각 , 라고
CH 의 분자량이 이므로 CH  g 의 양은  mol
하면, 분자량비는 (가) : (나) =    :  =  : 
이고, CH  mol 에 포함된 H 원자의 양은  mol 12. [출제의도] 몰 농도( M ) 비교하기
A 의 원자량 
이다. 따라서   ,   이다. ㄱ. (가)에서 A 의 양은  M ×  L   mol 이다. 이므로   이다. 따라서   이다.
B 의 원자량 
ㄷ. (가)에서 A 의 질량은  g 이므로 (다)에서 B
4. [출제의도] 화학 반응식의 반응 계수 구하기  mol 의 질량도  g 이다. 따라서 B 의 화학식량은 ㄷ.  g 에 들어 있는 전체 원자 수는  g 에 들어 있는
아세톤 연소 반응의 화학 반응식은 CHO +  O  →  이다. 분자 수 × 분자당 원자 수이므로 (가) : (나) 
  [오답풀이] ㄴ. (가)와 (나)에서 A 의 양( mol )이 같으  ×    ×      이다.
 CO  +  HO 이고,   ,   이므로    이다.
  므로  M ×  L   mol 이다. 따라서   이다. 19. [출제의도] 원자의 구성 입자 이해하기
5. [출제의도] 바닥상태 전자 배치 이해하기 13. [출제의도] 수소 원자의 오비탈 이해하기 용기 (가) (나)
ㄴ. (나)는  오비탈의 전자 개가 모두 홀전자가  분자 
C O  
HO 
HO
 는 , , 가 각각 , , 이므로 (가)는
되도록 배치했으므로 훈트 규칙을 만족한다. 
질량( g )    
ㄷ. (다)는 쌓음 원리, 파울리 배타 원리, 훈트 규칙을 , (나)는 , (다)는 이다.
분자량   
모두 만족하므로 바닥상태 전자 배치이다. ㄴ. (가)는 이므로   이다.
물질의 양( mol )    
[오답풀이] ㄱ. (가)는  오비탈에 전자가 개 모두 [오답풀이] ㄱ. (다)는 이다. 
O 원자의 양( mol )   
채워지지 않은 채 전자를  오비탈에 배치했으므로 ㄷ. 수소 원자에서 가 같으면 오비탈의 종류와
중성자의 양( mol )   
쌓음 원리에 어긋난다. 관계없이 오비탈의 에너지 준위가 같으므로 에너지
따라서    ,   이므로     이다.
준위는 (가)와 (나)가 같다.
6. [출제의도] 오비탈 이해하기
(가)와 (다)는 구형이고, 가 각각 , 이므로 (가)는 20. [출제의도] 화학 반응에서 양적 관계 이해하기
14. [출제의도] 바닥상태 원자의 전자 배치 이해하기
, (다)는 이며, (나)는  이다. 기체  mol 의 부피를  L 라고 하면 반응 전 Ⅰ과
바닥상태 Li , B , O 의 전자 배치는 각각  ,
Ⅱ에서 전체 기체의 양은 각각  mol ,  mol 이다.
ㄱ. (가)는 이다. 원자가 전자 수
   ,  이다. 는 Li , Ⅰ에서 B  g 의 양을  mol 이라 하면, 반응의 양적
ㄴ.  오비탈은 구형이므로 원자핵으로부터 거리가 홀전자 수
관계는 다음과 같다.
같으면 방향에 관계없이 전자가 발견될 확률이 같다.
B , O 가 각각 , , 이므로 X 는 Li 이다.  오비탈에
[오답풀이] ㄷ. 최대로 들어갈 수 있는 전자 수는  A g  + B g → C g 
들어 있는 전자 수는 B 와 O 가 각각 , 이므로 Y 는
(나)와 (다)가 로 같다. 반응 전( mol )    ( g )
O 이고, Z는 B이다.
7. [출제의도] 몰 농도( M ) 이해하기 ㄱ. X 는 Li 이다. 반응( mol ) -(  ) -  +

(나)에서 만든 수용액에 녹아 있는 A 의 양은 ㄴ. YO  는  오비탈에 들어 있는 전자 수가 이므로 반응 후( mol )  ( g ) 


 M ×  L   mol 이고, (가)에서 만든 수용액에   이다. 반응 후 전체 기체의 양( mol )은   이므로
녹아 있는 A 의 양은 (나)에서 만든 수용액의 배이므로 [오답풀이] ㄷ. ZB 에서 전자가 개 들어 있는   이다. 반응 몰비는 A g  : B g  =  : 이므로
 mol 이다. 따라서  g   mol ×  gmol 오비탈 수는 이다.   이고, Ⅱ에서 반응의 양적 관계는 다음과 같다.
  g 이다.
15. [출제의도] 화학 반응에서 양적 관계 이해하기 A g  + B g  → C g 
8. [출제의도] 원자와 이온의 구성 입자 이해하기 ㄱ. 화학 반응식은 Mg  + HCl  → 반응 전( mol )  ( g )
ㄱ. X 는 원자이므로 양성자수와 전자 수가 로 같다. MgCl   + H  g 이므로 X 는 H 이다. 반응( mol ) - -  +
[오답풀이] ㄴ. Y  은 전자 수가 이므로 Y 의 양성자 ㄴ. 발생한 X g  의 부피( L )는 반응한 Mg   의 반응 후( mol ) ( g )  
수는 이고, 중성자수는 이다. 따라서 Y 의 질량 질량(g)에 비례하므로 ‘반응한 Mg 의 질량에 비례
Ⅱ에서 반응한 B 의 질량은  g 이고, 남은 A
수는 이다. 한다.’는 ㉠으로 적절하다.
 mol 의 질량이  g 이므로 반응한 A  mol 의
ㄷ. Z  은 전자 수가 이므로 Z 의 양성자수는 이다. ㄷ.  ℃, 기압에서 X g   mol 의 부피를  L 라고
질량은  g 이며, 생성된 C 의 질량은  g 이다.
원자 번호는 양성자수와 같고, X ~ Z 의 양성자수가  g  L
하면,    이므로   이다. Ⅱ에서 생성된 C 의 질량( g ) 
 gmol  Lmol 따라서    이다.
각각 , , 이므로 원자 번호는 Y 가 가장 크다.
 

23
2023학년도 6월
전국연합학력평가 정답 및 해설 고2

세포체는 중간뇌에 있다.


17. [출제의도] 내분비샘과 호르몬의 특성 이해하기
• 생명과학Ⅰ • [오답풀이] ㄱ. ㉠과 ㉡은 교감 신경을 이루는 뉴런
시상 하부에서 분비된 TRH가 뇌하수체 전엽을 자극
이므로 ㉠의 길이는 ㉡의 길이보다 짧다.
하면 TSH가 분비되고, 분비된 TSH가 갑상샘을 자극
ㄷ. ㉠과 ㉣의 말단에서 분비되는 신경 전달 물질은
하면 티록신이 분비된다. 교감 신경이 부신 속질을
모두 아세틸콜린으로 같다.
정 답 자극하면 에피네프린이 분비된다. 따라서 ㉠은 뇌하
9. [출제의도] 흥분의 전도 이해하기 수체 전엽, ㉡은 갑상샘, A는 티록신이다. ㉢은 부신
1 ① 2 ③ 3 ④ 4 ① 5 ⑤ ㉠이 5 ms일 때 A의  에서의 막전위는 –80 mV이므로 속질, B는 에피네프린이다.
6 ① 7 ⑤ 8 ② 9 ① 10 ⑤ 자극을 준 지점 X는  이다. 따라서 ㉠이 3 ms일 때 ㄱ. A(티록신)의 분비가 촉진되면 물질대사가 활발
11 ⑤ 12 ② 13 ③ 14 ④ 15 ② A의  에서의 막전위는 약 –60 mV이므로 B의  에 하게 일어난다.
16 ① 17 ⑤ 18 ④ 19 ③ 20 ③ ㄴ. 부신 속질은 ㉢에 해당한다.
서의 막전위(+10 mV)와 비교하면 속도는 B가 A보다
ㄷ. ⓑ(신경에 의한 신호 전달 경로)에서가 ⓐ(호르몬에
빠르다. 흥분 전도 속도는 A가 1 cm/ms이고, B는
의한 신호 전달 경로)에서보다 빠르다.
해 설 2 cm/ms이다.
ㄱ. X는  이다. 18. [출제의도] 혈장 삼투압 조절 이해하기
[오답풀이] ㄴ. A의 흥분 전도 속도는 1 cm/ms이다. 항이뇨 호르몬(ADH)은 콩팥에서 수분 재흡수를 촉진
1. [출제의도] 생물의 특성 이해하기
ㄷ. ㉠이 4 ms일 때 B의  에서는 재분극이 일어나고 한다.
‘짝짓기를 하고 알을 낳는다.’는 생물의 특성 중 생식과
있다. ㄱ. 시상 하부는 ADH의 분비를 조절한다.
유전에 해당한다.
ㄷ. 단위 시간당 오줌 생성량은 Ⅰ에서가 Ⅱ에서보다
①은 생식과 유전, ②는 항상성, ③은 물질대사, ④는 10. [출제의도] 생명 과학의 통합적 특성 이해하기
적다.
세포로 구성, ⑤는 적응과 진화이다. 학생이 조사한 내용인 거북 로봇, 유전체 분석 결과를
[오답풀이] ㄴ. ㉠은 혈장 삼투압이다.
활용한 백신과 항체 개발, 고대 인류의 모습 재현은
2. [출제의도] 요소 분해 실험 이해하기
다른 분야의 학문과 생명 과학이 연계된 사례이다. 19. [출제의도] 흥분의 전도 이해하기
ㄱ. 생콩즙에는 요소를 분해하는 효소가 들어 있다.
㉠은 K  , ㉡은 Na , Ⅰ은 세포 안, Ⅱ는 세포 밖이다.
ㄴ. 생콩즙에 들어 있는 효소에 의해 요소가 분해되어 11. [출제의도] 세포 호흡 이해하기
ㄱ. ㉠은 K  이다.
암모니아가 생성되고, 생성된 암모니아에 의해 용액의 ㉠은 O  , ㉡은 CO  , ⓐ는 세포 호흡 과정에서 발생
pH가 높아진다. ㉠의 농도
하는 에너지이다. (나)는 ATP이다. ㄷ.  일 때 는 Ⅰ에서가 Ⅱ에서보다 높다.
[오답풀이] ㄷ. 용액의 색깔 변화는 종속변인에 해당 ㄴ. ⓐ의 일부는 ATP를 합성하는데 이용된다. ㉡의 농도
한다. ㄷ. ATP가 ADP로 분해될 때 발생하는 에너지는 생명 [오답풀이] ㄴ. Ⅰ은 세포 안이다.

3. [출제의도] 생물과 비생물의 차이점 이해하기 활동에 이용된다.


20. [출제의도] 골격근의 수축 이해하기
[오답풀이] ㄱ. ㉠은 O  이다.
병원체 A는 바이러스이다.
길이(m )
ㄴ. 바이러스는 단백질을 갖는다. 시점
12. [출제의도] 호르몬의 분비 조절 이해하기 X ㉠ ㉡ ㉢(H대) A대
ㄷ. 바이러스는 스스로 물질대사를 하지 못한다. ㉠은 뇌하수체 전엽, A는 티록신이다.  3.0 0.7 0.3 1.0 1.6
[오답풀이] ㄱ. 바이러스는 세포 구조가 아니다. ㄴ. A는 혈액을 통해 표적 기관으로 이동한다.
 2.4 0.4 0.6 0.4 1.6
4. [출제의도] 뉴런의 구조 이해하기 [오답풀이] ㄱ. ㉠은 뇌하수체 전엽이다.
(가)는 연합 뉴런, (나)는 감각 뉴런이다. ㉠은 가지 ㄷ. 혈중 A의 농도가 증가하면 TSH의 분비는 억제된다.  에서  로 될 때 골격근이 수축한다.
돌기의 일부, ㉡은 말이집, ㉢은 랑비에 결절이다. ㄱ.  일 때 H대의 길이는 1.0 m 이다.
13. [출제의도] 뇌의 구조와 기능 이해하기
ㄱ. (가)는 연합 뉴런이다. ㄴ.  일 때 ㉡의 길이는 0.6 m 이다.
A는 소뇌, B는 중간뇌, C는 연수이다. ㉠은 ‘중추
[오답풀이] ㄴ. ㉠은 가지 돌기의 일부이다. 신경계에 속한다.’, ㉡은 ‘동공 반사의 조절 중추 [오답풀이] ㄷ. X의 길이는  일 때가  일 때보다
ㄷ. ㉢에 역치 이상의 자극을 주면 ㉡에서 활동 전위가 이다.’, ㉢은 ‘뇌줄기에 속한다.’이다. 길다.
발생하지 않는다.

5. [출제의도] 대사성 질환 이해하기


ㄱ. 고지혈증은 물질대사 이상으로 생기는 대사성 질환
B A
이다.
C
ㄴ. 콜레스테롤은 ㉠에 포함된다.
ㄷ. ㉠이 혈관 내벽에 계속 쌓이면 심혈관계 질환이
14. [출제의도] 혈당량 조절 이해하기
나타날 수 있다.
정상인이 운동을 하는 동안 혈중 인슐린 농도는 감소
6. [출제의도] 생명 과학의 탐구 방법 이해하기 하고, 혈중 글루카곤 농도는 증가한다. 따라서 A는
ㄱ. 연역적 탐구 방법이 이용되었다. 인슐린이고, B는 글루카곤이다.
[오답풀이] ㄴ. (나)는 가설 설정 단계이다. ㄱ. A(인슐린)는 이자의 세포에서 분비된다.
ㄷ. 이 탐구를 통해 ‘수컷의 소드 길이가 다를 때 암 ㄷ. A(인슐린)와 B(글루카곤)는 혈중 포도당 농도
컷이 L과 보내는 시간은 S와 보내는 시간보다 길다.’는 조절에 길항적으로 작용한다.
것을 알 수 있다. [오답풀이] ㄴ. 혈액에서 간세포로 포도당 흡수를 촉진
하는 호르몬은 A(인슐린)이다.
7. [출제의도] 기관계의 통합적 작용 이해하기
A는 순환계, B는 호흡계, C는 배설계이다. 15. [출제의도] 체온 조절 이해하기
ㄴ. 간에서 암모니아가 요소로 전환된다. ㄴ. ㉠은 교감 신경이고, 교감 신경에 의해 피부 근처
ㄷ. 물( HO )이 몸 밖으로 나갈 때 호흡계(B)와 배 혈관이 수축된다.
설계(C)가 관여한다. [오답풀이] ㄱ. (가)는 저온이다.
[오답풀이] ㄱ. 폐는 호흡계(B)에 속한다. ㄷ. ⓐ는 열 발생량(열 생산량) 증가이다.

8. [출제의도] 자율 신경계 이해하기 16. [출제의도] 신경계 이해하기

동공은 교감 신경(A)에 의해 확대되고, 부교감 신경 ㉠은 운동 신경, ㉡은 연합 신경, ㉢은 감각 신경이다.


(B)에 의해 축소된다. ㉠에 역치 이상의 자극을 주었을 A : ㉠은 척수의 전근을 이룬다.
때 시간에 따라 동공이 확대되므로 ㉡은 교감 신경의 [오답풀이] B : ㉡은 연합 신경이다.
신경절 이후 뉴런이다. C : 자극에 의한 반사가 일어날 때 ㉢에서 ㉠으로 흥
ㄴ. ㉢은 부교감 신경의 신경절 이전 뉴런이므로 신경 분의 전달이 일어난다.

24
2023학년도 6월
전국연합학력평가 정답 및 해설 고2

[오답풀이] ㄴ. 판게아(초대륙)는 고생대 말에 형성 B


 × (%) 값이 가장 큰 태평양판의 평균 이동 속도가
A
• 지구과학Ⅰ • 되었다. ㄷ. 히말라야산맥은 인도판과 유라시아판이
가장 빠르다.
충돌하여 형성되었으므로 (가)와 (나) 시기 사이에
형성되기 시작하였다.
17. [출제의도] 퇴적 구조 이해하기
정 답 9. [출제의도] 암석의 용융 곡선 이해하기 ㄱ. 건열은 점토와 같이 입자가 매우 작은 퇴적물이
ㄱ. A는 물이 포함된 화강암의 용융 곡선, B는 물이 수면 위의 건조한 환경에 노출되어 퇴적물의 표면이
1 ② 2 ④ 3 ② 4 ① 5 ⑤ 포함되지 않은 맨틀의 용융 곡선이다. 갈라진 구조이다. ㄷ. ㉠은 지층의 하부에서 상부로
6 ④ 7 ② 8 ① 9 ① 10 ③ [오답풀이] ㄴ. 해령 하부의 마그마는 상승하는 맨틀 갈수록 주요 입자 평균 크기가 작아지고 있으므로
11 ① 12 ② 13 ⑤ 14 ⑤ 15 ④ 물질에 작용하는 압력이 감소함에 따라 생성되므로 ㉡ 점이 층리의 주요 입자 평균 크기 변화를 나타낸 것
16 ⑤ 17 ③ 18 ③ 19 ① 20 ④
과정으로 설명할 수 있다. ㄷ. ㉡ 과정으로 생성된 이다.
마그마가 굳어서 만들어진 대표적인 암석으로는 반려암, [오답풀이] ㄴ. 유수나 바람의 방향을 알 수 있는 대표
해 설 현무암이 있다. 적인 퇴적 구조에는 사층리가 있다.

10. [출제의도] 판 경계와 화산 활동 이해하기 18. [출제의도] 방사성 동위 원소의 붕괴 곡선 이해하기


1. [출제의도] 판구조론 정립 과정 이해하기 ㄱ. A는 태평양판과 나즈카판의 경계에서 화산 활동이 ㄱ. T일 때 X와 Y의 함량이 각각 50%이므로 X의
ㄴ. 맨틀대류설은 방사성 원소가 붕괴하여 생성된 열과 일어나고 있는 지역으로 판의 발산형 경계가 존재한다. 반감기는 T이다. ㄷ. 3T일 때 X : Y = 2 : 14이므로
고온의 지구 중심부에서 맨틀로 공급되는 열에 의해 ㄴ. B 부근에는 판의 섭입에 의한 습곡 산맥이 발달 Y 의 함량
한다. 의 값은 7이다.
맨틀이 대류한다고 설명하는 이론이다. X 의 함량
[오답풀이] ㄱ. 맨틀대류설은 홈스가 주장하였다. ㄷ. [오답풀이] ㄷ. A에서는 주로 현무암질 용암이, B에 [오답풀이] ㄴ. A는 25이다.
A는 맨틀 대류의 상승부이다. 서는 주로 안산암질 용암이 분출한다. 따라서 용암의
SiO  평균 함량은 A가 B보다 낮다. 19. [출제의도] 고기압과 저기압 이해하기
2. [출제의도] 음향 측심법 이해하기 ㄱ. 대기권에서 기압은 상층으로 갈수록 낮아진다.
A 해역에는 해구가 존재하고, B 해역에는 해령이 존재 11. [출제의도] 지층 대비 이해하기 [오답풀이] ㄴ. A는 주위보다 기압이 낮은 저기압이다.
한다. ㄴ. 해양 지각의 평균 나이는 해구가 존재하는 ㄱ. (가)와 (나)의 응회암은 동일한 시기에 형성된 ㄷ. B는 고기압으로 하강 기류가 나타난다.
해역 A가 해령이 존재하는 해역 B보다 많다. ㄷ. 음파의 것으로 열쇠층(건층)으로 적절하다.
20. [출제의도] 온대 저기압 이해하기
왕복 시간이 8초인 지점의 수심은 6km이다. 따라서 [오답풀이] ㄴ. (나)의 셰일층은 응회암층보다 먼저
형성되었으므로 신생대의 표준 화석인 화폐석은 산출 ㄴ. A에는 북서풍, B에는 남서풍, C에는 남동풍이 우세
A 해역에는 음파의 왕복 시간이 8초보다 긴 지점이
될 수 없다. ㄷ. (가)의 사암층은 (나)의 사암층보다 하다. ㄷ. 한랭 전선 후면에 있는 A가 온난 전선 전면에
존재한다.
나중에 퇴적된 것이다. 있는 C보다 소나기가 내릴 가능성이 높다.
[오답풀이] ㄱ. ㉠은 A 해역에서 측정한 자료이다.
[오답풀이] ㄱ. A ~ C 중 기압이 가장 높은 곳은 C이다.
3. [출제의도] 고지자기 줄무늬 이해하기 12. [출제의도] 단층 이해하기
ㄷ. 해령을 축으로 양쪽으로 해양 지각이 확장된다. ㄴ. 상반이 하반에 대해 상대적으로 아래로 내려간
따라서 고지자기 줄무늬는 해령을 축으로 대칭적으로 정단층이므로 단층에 작용한 힘은 장력이다.
분포한다. [오답풀이] ㄱ. ㉠은 단층면보다 위에 있으므로 상반
[오답풀이] ㄱ. 해령을 중심으로 멀리 떨어질수록 해양 이다. ㄷ. 단층이 형성되기 전에는 A가 B보다 상부에
지각의 나이가 많아진다. 따라서 B가 A보다 먼저 생성된 위치하였으므로 A는 B보다 나중에 퇴적되었다.
것이다. ㄴ. A는 정자극기, B는 역자극기에 생성되었다.
13. [출제의도] 포획암 이해하기
4. [출제의도] 지질 시대의 특징 이해하기 마그마가 관입할 때 기존 암석에서 떨어져 나온 암석
A는 선캄브리아 시대, B는 고생대, C는 신생대이다. 조각이 화성암에 포함되어 나타날 수 있는데 이를
따라서 A, B, C의 시기를 시간 순서대로 옳게 나열한 포획암이라고 한다. B. 마그마가 관입할 때 주변 암석의
것은 A → B → C이다. 일부가 떨어져 나와 마그마 속으로 유입되는 것을
포획이라 하고, 포획된 암석을 포획암이라고 한다.
5. [출제의도] 퇴적암 분류하기
따라서 포획암을 둘러싸고 있는 주변 암석은 화성암
ㄱ. 역암, 사암, 셰일은 쇄설성 퇴적암에 속한다. ㄴ.
이다. C. 포획암은 주변 암석에 해당하는 화성암보다
(나)는 유기적 퇴적암으로 생물의 유해나 골격의 일부가
먼저 생성된 것이다.
쌓여서 만들어진다. ㄷ. 암염은 해수가 증발하면서
[오답풀이] A. 낙엽은 물이 얼기 전부터 물속에 존재
소금이 침전하여 생성된 암석으로 화학적 퇴적암에
하였다.
속한다.
14. [출제의도] 지질 구조 형성 과정 이해하기
6. [출제의도] 고지자기극 겉보기 이동 경로 이해하기
ㄱ. ㉠은 횡압력이다. ㄴ. 지점토 판의 위쪽을 수평으로
ㄴ. 복각의 크기는 지자기 북극에 가까울수록 크다.
자르는 것은 지층이 해수면 위로 융기된 후 일어나는
따라서 A 지점에서 측정한 복각의 크기는 1억 년 전이
침식 작용에 해당한다. ㄷ. (다)에서 X-X′를 따라
2억 년 전보다 크다. ㄷ. 고지자기극의 겉보기 이동
지점토 판은 C-B-A-B-C 순서로 관찰된다.
경로를 통해 대륙이 이동했음을 알 수 있다.
[오답풀이] ㄱ. 지자기 북극은 하나이다. 15. [출제의도] 지질 단면 해석하기
ㄴ. 삼엽충은 고생대의 표준 화석으로서 지층의 나이가
7. [출제의도] 플룸구조론 이해하기
6억 년보다 많은 A, B에서는 발견될 수 없다. ㄷ.
맨틀의 온도가 높은 곳에서는 지진파의 속도가 느리기
관입암 Y와 지층 C의 관계는 난정합에 해당한다.
때문에 S파의 속도 편차가 음의 값(-)을 가진다.
[오답풀이] ㄱ. 관입의 법칙에 따르면 관입당한 암석이
ㄴ. B는 상승하는 뜨거운 플룸으로 A보다 밀도가 작다.
관입한 암석보다 먼저 생성된 것이다. 따라서 X는
[오답풀이] ㄱ. A는 하강하는 차가운 플룸이다. ㄷ.
Y보다 나중에 생성된 것이다.
㉠에서 분출하는 용암은 섭입대에서 생성된 것이다.
16. [출제의도] 판의 이동 속도에 미치는 요인 이해하기
8. [출제의도] 지질 시대 대륙 분포 이해하기
ㄱ. ㉠은 약 18.9, ㉡은 약 28.5이다. ㄴ. 인도판의
(가)는 중생대, (나)는 현재, (다)는 고생대 말의 대륙 B
분포를 나타낸 것이다. ㄱ. 대륙의 분포는 (다) →  × (%) 값은 약 8.2이므로 a는 인도판이다. ㄷ.
A
(가) → (나) 순으로 변하였다.

25

You might also like